You are on page 1of 204

QUIZ : ALTERATIONS IN OXYGENATION

1. Which of the following infants is least probable to develop sudden infant death syndrome
(SIDS)?

A. Baby Angela who was premature.


B. A sibling of Baby Angie who died of SIDS.
C. Baby Gabriel with prenatal drug exposure.
D. Baby Gabby who sleeps on his back.

Option D: Infants who sleep on their back are least likely to develop SIDS. However, SIDS has
been associated with infants who sleep on their abdomens.
Options A, B, C: Being premature, having a sibling who died of SIDS, and being prenatally
exposed to drugs all place the infant at high risk for developing SIDS

2. Which of the following actions is NOT appropriate in the care of a 2-month-old infant?

A. Place the infant on her back for naps and bedtime.


B. Allow the infant to cry for 5 minutes before responding if she wakes during the night as
she may fall back asleep.
C. Talk to the infant frequently and make eye contact to encourage language development.
D. Wait until at least 4 months to add infant cereals and strained fruits to the diet.

Feedback

Answer: B. Allow the infant to cry for 5 minutes before responding if she wakes during the night as
she may fall back asleep.

Option B: Infants under 6 months may not be able to sleep for long periods because their
stomachs are too small to hold adequate nourishment to take them through the night.

Option A: Infants should always be placed on their backs to sleep. Research has shown a dramatic
decrease in sudden infant death syndrome (SIDS) with back sleeping.

Option C: Eye contact and verbal engagement with infants are important to language development.

Option D: The best diet for infants under 4 months of age is breast milk or infant formula.

3. Sudden infant death syndrome (SIDS) is one of the most common causes of death in infants. At
what age is the diagnosis of SIDS most likely?

A. At 1 to 2 years of age.
B. At 1 week to 1 year of age, peaking at 2 to 4 months.
C. At 6 months to 1 year of age, peaking at 10 months.
D. At 6 to 8 weeks of age.

Feedback

Answer: B. At 1 week to 1 year of age, peaking at 2 to 4 months.


Options B: SIDS can occur anytime between 1 week and 1 year of age.

Options A, C, D: The incidence peaks at 2 to 4 months of age.

4. During a community visit, volunteer nurses teach stress management to the participants. The
nurses will most likely advocate which belief as a method of coping with stressful life events?

A. Avoidance of stress is an important goal for living.


B. Control over one’s response to stress is possible.
C. Most people have no control over their level of stress.
D. Significant others are important to provide care and concern.

5. The nurse teaches the mother of a newborn that in order to prevent sudden infant death
syndrome (SIDS) the best position to place the baby after nursing is

A. Prone.
B. Trendelenburg
C. Supine.
D. Fowler’s.

Options B and C. Research demonstrates that the occurrence of SIDS is reduced with these two
positions.
Options A and D: Both are inappropriate positions for infants.

6. The nurse is caring for a pediatric patient with chronic obstructive pulmonary disease (COPD).
Which intervention for airway management should you delegate to a nursing assistant?

A. Assisting the patient to sit up on the side of the bed.


B. Instructing the patient to cough effectively.
C. Teaching the patient to use incentive spirometry.
D. Auscultation of breath sounds every 4 hours.

7. A 7-year-old client is brought to the E.R. He’s tachypneic and afebrile and has a respiratory rate
of 36 breaths/minute and a nonproductive cough. He recently had a cold. From his history, the
client may have which of the following?

A. Acute asthma
B. Bronchial pneumonia
C. Chronic obstructive pulmonary disease (COPD)
D. Emphysema

Correct Answer: A. Acute asthma

Based on the client’s history and symptoms, acute asthma is the most likely diagnosis. Patients will
usually give a history of a wheeze or a cough, exacerbated by allergies, exercise, and cold. There
is often diurnal variation, with symptoms being worse at night. Many asthmatics have nocturnal
coughing spells but appear normal in the daytime. He’s unlikely to have bronchial pneumonia
without a productive cough and fever and he’s too young to have developed COPD or
emphysema.

8. Which of the following assessment findings would help confirm a diagnosis of asthma in a client
suspected of having the disorder?

A. Circumoral cyanosis
B. Increased forced expiratory volume
C. Inspiratory and expiratory wheezing
D. Normal breath sounds

Correct Answer: C. Inspiratory and expiratory wheezing

Inspiratory and expiratory wheezes are typical findings in asthma. Patients will show some
respiratory distress, often sitting forward to splint open their airways. On auscultation, a bilateral,
expiratory wheeze will be heard. In life-threatening asthma, the chest may be silent, as air cannot
enter or leave the lungs, and there may be signs of systemic hypoxia.

9. A client with acute asthma showing inspiratory and expiratory wheezes and a decreased
expiratory volume should be treated with which of the following classes of medication right away?

A. Beta-adrenergic blockers
B. Bronchodilators
C. Inhaled steroids
D. Oral steroids

Correct Answer: B. Bronchodilators

Bronchodilators are the first line of treatment for asthma because bronchoconstriction is the cause
of reduced airflow. Bronchodilators are indicated for individuals that have lower than optimal airflow
through the lungs. The mainstay of treatment is beta-2 agonists that target the smooth muscles in
the bronchioles of the lung. Various respiratory conditions may require bronchodilators, including
asthma and chronic obstructive pulmonary disease.

10. Emergency treatment of a client in status asthmaticus includes which of the following
medications?

A. Inhaled beta-adrenergic agents


B. Inhaled corticosteroids
C. I.V. beta-adrenergic agents
D. Oral corticosteroids

Correct Answer: A. Inhaled beta-adrenergic agents

Inhaled beta-adrenergic agents help promote bronchodilation, which improves oxygenation.


Albuterol is preferred over metaproterenol in that class because of its higher beta 2 selectivities
and longer duration of action. The dose-response curve and duration of action of these
medications are adversely affected by a combination of patient factors, including pre-existing
bronchoconstriction, airway inflammation, mucus plugging, poor patient effort, and coordination.
QUIZ: LABOR AND DELIVERY
COMPLICATIONS

1. A nurse in a labor room is assisting with the vaginal delivery of a newborn infant. The nurse
would monitor the client closely for the risk of uterine rupture if which of the following occurred?

A. Hypotonic contractions
B. Forceps delivery
C. Schultz delivery
D. Weak bearing down efforts

Correct Answer: B. Forceps delivery. Excessive fundal pressure, forceps delivery, violent bearing
down efforts, tumultuous labor, and shoulder dystocia can place a woman at risk for traumatic
uterine rupture. Hypotonic contractions and weak bearing down efforts do not alone add to the risk
of rupture because they do not add to the stress on the uterine wall.

2. A client who is gravida 1, para 0 is admitted in labor. Her cervix is 100% effaced, and she is
dilated to 3 cm. Her fetus is at +1 station. The nurse is aware that the fetus’ head is:

A. Not yet engaged


B. Entering the pelvic inlet
C. Below the ischial spines
D. Visible at the vaginal opening

Correct Answer: C. Below the ischial spines A station of +1 indicates that the fetal head is 1 cm
below the ischial spines. Positive numbers are used when a baby has descended beyond the
ischial spines. During birth, a baby is at the +4 to +5 station.

3. When monitoring the fetal heart rate of a client in labor, the nurse identifies an elevation of 15
beats above the baseline rate of 135 beats per minute lasting for 15 seconds. This should be
documented as:

A. An acceleration
B. An early elevation
C. A sonographic motion
D. A tachycardia

Correct Answer: A. An acceleration is an abrupt elevation above the baseline of 15 beats per
minute for 15 seconds; if the acceleration persists for more than 10 minutes it is considered a
change in baseline rate. A tachycardic FHR is above 160 beats per minute.

4. A client arrives at the hospital in the second stage of labor. The fetus’ head is crowning, the
client is bearing down, and the birth appears imminent. The nurse should:

A. Transfer her immediately by stretcher to the birthing unit.


B. Tell her to breathe through her mouth and not to bear down.
C. Instruct the client to pant during contractions and to breathe through her mouth.
D. Support the perineum with the hand to prevent tearing and tell the client to pant.

5. A client is admitted in labor with spontaneous ROM 24hr earlier. The fluid is clear and the FHR
is 124 w/ moderate variability. Which assessment is most important for the nurse to make at this
time?

A) Maternal Temperature
B) Cervical Dilation & effacement
C) maternal pulse rate
D) Contraction frequency and duration

6. The nurse is to intervene when caring for a laboring client whose baby is exhibiting signs of fetal
distress. Which of the following actions should the nurse take?

A) administer O2 via nasal cannula


B) place the client in high Fowler's position
C) Remove the internal fetal monitor electrode
D) increase the IV infusion rate

7. A woman, 39 weeks gestation, is admitted to the delivery unit w/ vaginal warts from human
papillomavirus. Which of the following actions by the RN is appropriate?

A) notify MD for a surgical delivery


B) follow standard infectious disease precautions
C) notify the nursery of the imminent delivery of an infected neonate
D) wear a mask whenever the perineum is exposed

8. A patient just spontaneously ruptured membranes. Which of the following factors makes her
especially at high risk for having a prolapsed cord? Select all

A) breech presentation
B) station -3
C) oligohydramnios
D) dilation 2cm
E) transverse lie

Feedback
A) breech
B) -3 station
E) transverse lie

9. A woman is 6cm dilated. The fetal monitor tracing shows recurring deep late decelerations. The
woman's doctor informs her that the baby must be delivered by C-sect. The woman refuses to sign
the informed consent. Which of the following actions by the RN is appropriate?

A) strongly encourage the woman to sign consent


B) prepare the woman for c-sect
C) inform the woman that the baby will likely die w/out surgery
D) provide the woman with ongoing labor support

10. Assessment on mother Estella's reveal that she is 4cm dilated and 80% effaced w/ a fetal heart
tracing showing frequent late decelerations and strong contractions every 3 minutes, each lasting
90 seconds. The nursing management should be directed toward which of the following goals?

A) completion of the first stage of labor


B) delivery of a healthy baby
C) safe pain medication
D) prevention of a vaginal lacerations

11. A nurse is caring for a client in labor who is receiving Pitocin by IV infusion to stimulate uterine
contractions. Which assessment finding would indicate to the nurse that the infusion needs to be
discontinued?

A. Three contractions occurring within a 10-minute period


B. Increased urinary output
C. Adequate resting tone of the uterus palpated between contractions
D. A fetal heart rate of 90 beats per minute

Correct Answer: D. A fetal heart rate of 90 beats per minute A normal fetal heart rate is 120-160
BPM. Bradycardia or late or variable decelerations indicate fetal distress and the need to
discontinue Pitocin. The goal of labor augmentation is to achieve three good-quality contractions in
a 10-minute period.

12. A nurse is monitoring a client in active labor and notes that the client is having contractions
every 3 minutes that last 45 seconds. The nurse notes that the fetal heart rate between
contractions is 100 BPM. Which of the following nursing actions is most appropriate?

A. Encourage the client’s coach to continue to encourage breathing exercises.


B. Encourage the client to continue pushing with each contraction.
C. Continue monitoring the fetal heart rate.
D. Notify the physician or nurse-midwife.

Correct Answer: D. Notify the physician or nurse-midwife. A normal fetal heart rate is 120-160
beats per minute. Fetal bradycardia between contractions may indicate the need for immediate
medical management, and the physician or nurse-midwife needs to be notified.

13. A nurse is admitting a pregnant client to the labor room and attaches an external electronic
fetal monitor to the client’s abdomen. After the attachment of the monitor, the initial nursing
assessment is which of the following?

A. Identifying the types of accelerations


B. Assessing the baseline fetal heart rate
C. Determining the frequency of the contractions
D. Determining the intensity of the contractions
Correct Answer: B. Assessing the baseline fetal heart rate is important so that abnormal variations
of the baseline rate will be identified if they occur. Options 1 and 3 are important to assess, but not
as the first priority.

14. A pregnant client is admitted to the labor room. An assessment is performed, and the nurse
notes that the client’s hemoglobin and hematocrit levels are low, indicating anemia. The nurse
determines that the client is at risk for which of the following?

A. A loud mouth
B. Low self-esteem
C. Hemorrhage
D. Postpartum infections

- Anemic women have a greater likelihood of cardiac decompensation during labor, postpartum
infection, and poor wound healing. Anemia does not specifically present a risk for hemorrhage.

15. Four hours after a difficult labor and birth, a primiparous woman refuses to feed her baby,
stating that she is too tired and just wants to sleep. The nurse should:

A. Tell the woman she can rest after she feeds her baby.
B. Recognize this as a behavior of the taking-hold stage.
C. Record the behavior as ineffective maternal-newborn attachment.
D. Take the baby back to the nursery, reassuring the woman that her rest is a priority at this
time.

Correct Answer: D. Take the baby back to the nursery, reassuring the woman that her rest is a
priority at this time. The behavior described is typical of this stage and not a reflection of ineffective
attachment unless the behavior persists. Mothers need to reestablish their own well-being in order
to effectively care for their baby.

16. A client arrives at a birthing center in active labor. Her membranes are still intact, and the
nurse-midwife prepares to perform an amniotomy. A nurse who is assisting the nurse-midwife
explains to the client that after this procedure, she will most likely have:

A. Less pressure on her cervix.


B. Increased efficiency of contractions.
C. Decreased number of contractions.
D. The need for increased maternal blood pressure monitoring.

- Amniotomy can be used to induce labor when the condition of the cervix is favorable (ripe) or to
augment labor if the process begins to slow. Rupturing of membranes allows the fetal head to contact
the cervix more directly and may increase the efficiency of contractions.

17. A nurse in the labor room is preparing to care for a client with hypertonic uterine dysfunction.
The nurse is told that the client is experiencing uncoordinated contractions that are erratic in their
frequency, duration, and intensity. The priority nursing intervention would be to:
A. Monitor the Pitocin infusion closely
B. Provide pain relief measures
C. Prepare the client for an amniotomy
D. Promote ambulation every 30 minutes

- Management of hypertonic labor depends on the cause. Relief of pain is the primary intervention to
promote a normal labor pattern.

18. A maternity nurse is preparing to care for a pregnant client in labor who will be delivering twins.
The nurse monitors the fetal heart rates by placing the external fetal monitor:

A. Over the fetus that is most anterior to the mother’s abdomen.


B. Over the fetus that is most posterior to the mother’s abdomen.
C. So that each fetal heart rate is monitored separately.
D. So that one fetus is monitored for a 15-minute period followed by a 15 minute fetal monitoring
period for the second fetus.

Correct Answer: C. So that each fetal heart rate is monitored separately. In a client with a multifetal
pregnancy, each fetal heart rate is monitored separately. Simultaneous monitoring of twins is
preferable to non simultaneous monitoring to discriminate between their separate FHRs (ACOG,
1989). Synchronizing the internal clocks of both monitors will help produce accurate
documentation. Otherwise, time increments should be documented on both monitor tracings for
later comparison, to ensure that each twin has been monitored. If the monitor strips are
synchronous, portable real-time ultrasound can be used to verify that both twins are being
monitored independently

19. A maternity nurse is caring for a client with abruptio placenta and is monitoring the client for
disseminated intravascular coagulopathy. Which assessment finding is least likely to be associated
with disseminated intravascular coagulation?

A. Swelling of the calf in one leg


B. Prolonged clotting times
C. Decreased platelet count
D. Petechiae, oozing from injection sites, and hematuria

20. A maternity nurse is preparing for the admission of a client in the 3rd trimester of pregnancy
that is experiencing vaginal bleeding and has a suspected diagnosis of placenta previa. The nurse
reviews the physician’s orders and would question which order?

A. Prepare the client for an ultrasound.


B. Obtain equipment for external electronic fetal heart monitoring.
C. Obtain equipment for a manual pelvic examination.|
D. Prepare to draw a Hgb and Hct blood sample.

Correct Answer: C. Obtain equipment for a manual pelvic examination. Manual pelvic
examinations are contraindicated when vaginal bleeding is apparent in the 3rd trimester until a
diagnosis is made and placenta previa is ruled out. Digital examination of the cervix can lead to
maternal and fetal hemorrhage.
QUIZ: CONGENITAL CARDIAC DISORDERS
1. The nursing student is preparing to teach a prenatal class about fetal circulation. Which
statement should be included in the teaching plan?

a. “One artery carries oxygenated blood from the placenta to the fetus”

b. “Two arteries carry oxygenated blood from the placenta to the fetus”
c. “Two arteries carry deoxygenated blood and waste products away from fetus”

d. “Two veins carry blood that is high in carbon dioxide and other waste products away from the
fetus to the placenta.”

Rationale: Blood pumped by the embryo’s heart leaves the embryo through two umbilical arteries.
When oxygenated, the blood is returned by one umbilical vein. Arteries carry deoxygenated blood
and waste products from the fetus, and then the umbilical vein carries oxygenated blood and
provides oxygen and nutrients.

2. A nursing student is assigned to care for a client in labor. The nursing instructor asks the student
to describe fetal circulation, especially the ductus venosus. Which statement is correct regarding
the ductus venosus?

a. Connects the pulmonary artery to the aorta

b. Is an opening between the right and left atria

c. Connects the umbilical vein to the inferior vena cava

d. Connects the umbilical artery to the inferior vena cava

Rationale: The ductus venosus connects the umbilical vein to the inferior vena cava. The foramen
ovale is a temporary opening between the right and left atria. The ductus arteriosus joins the right
and left atria. The ductus arteriosus joins the aorta and pulmonary artery.

3. You’re providing an in-service to a group of new nurses who will be caring for patients who have
Tetralogy of Fallot. Which statement below is INCORRECT concerning how the blood normally
flows through the heart?

a. Unoxygenated blood enters through the superior and inferior vena cava and travels to
the left atrium.
b. The pulmonic valve receives blood from the right ventricle and allows blood to flow to the lungs
via the pulmonary artery.
c. The left atrium allows blood to flow down through the bicuspid valve (mitral) into the left
ventricle.

d. Oxygenated blood leaves the left ventricle and flows up through the aortic valve and aorta to be
pumped to the rest of the body.
Rationale: This statement is INCORRECT. It should say: “Unoxygenated blood enters through the
superior and inferior vena cava to the RIGHT (not left) atrium.

4. While feeding a 3-month-old infant, who has Tetralogy of Fallot, you notice the infant’s skin
begins to have a bluish tint and the breathing rate has increased. Your immediate nursing action is
to?

a. Continue feeding the infant and place the infant on oxygen.

b. Stop feeding the infant and provide suction.


c. Stop feeding the infant and place the infant in the knee-to-chest position and administer
oxygen.

d. Assess the infant’s heart rate and rhythm.

Rationale: The patient is experiencing a “tet spell”. This is where during any type of activity like
feeding, crying, playing etc. the child’s heart (due to Tetralogy of Fallot) is unable to maintain
proper oxygen levels in the blood (these activities place extra work on the heart and it can’t keep
up). Therefore, there are low amounts of oxygen in the blood, and the skin will become cyanotic
(bluish tint) and the respiratory rate will increase (this is the body’s way of trying to increase the
oxygen levels in the body but it doesn’t work because it’s not a gas exchange problem in the lungs
but a heart problem). The nurse would want to place the infant in the knee-to-chest position. WHY?
This increases systemic vascular resistance (which will help decrease the right to left shunt that is
occurring in the heart…hence helps replenish the body with oxygenated blood). In addition, the
nurse would want to place the patient on oxygen.

5. True or False: Atrial septal defects are characterized by a hole in the interatrial septum that
allows blood to mix in the right and left atria, which are the lower chambers of the heart.

True

False

Rationale: Atrial septal defects are characterized by a hole in the interatrial septum that allows
blood to mix in the right and left atria, which are the UPPER (not lower) chambers of the heart.

6. You’re caring for a 2-year-old patient who has a large atrial septal defect that needs repair. This
defect is causing complications. These complications are arising from an abnormal shunting of
blood throughout the heart. As the nurse, you know that a __________________ shunt is
occurring in the heart due to the defect.

a. Right-to-left

b. Right

c. Left

d. Left-to-right

Rationale: A left-to-right shunt is occurring because the pressure in the left side of the heart is
higher than the right, which allows blood to easily flow into the right side through the hole in the
interatrial septum. This will cause pulmonary hypertension and heart failure overtime.
7. You’re assessing the heart sounds of a child with an atrial septal defect. You note a heart
murmur at the 2nd intercostal space at the left upper sternal border. Heart murmurs noted in
patients with an atrial septal defect are called?

a. Holosystolic murmurs

b. Diastolic murmurs

c. Early systolic murmurs

d. Midsystolic murmurs

Rationale: In patients with ASDs, the nurse may notice a midsystolic (also called systolic ejection
murmur) at the 2nd ICS at the left sternal border. This is due to increased blood flow through the
pulmonic valve. The murmur is quiet at the beginning of systole, increases mid-systole and then
decreases at the end of systole…it ends before S2 . S2 is wide, fixed splitting due to the slowness
of the pulmonic valve closing.

8. Select the structure below that allows blood to flow from the right to left atrium in utero and that
should close after birth:

a. Ductus Arteriosus

b. Formen Ovale

c. Ductus Venosus

d. Ligamentum teres

Rationale: The formen ovale allows this to occur.

9. After admitting a child with an atrial septal defect, you start developing a nursing care plan. What
nursing diagnoses can you include in the patient’s plan of care based on the complications that
arise from this condition?

a. Activity Intolerance
b. Risk for Infection
c. Decrease Cardiac Output

d. All of the above

Rationale: Patients who are hospitalized with an ASD will be having complications that arise from
the left to right shunt of blood flow in the heart, which can lead to heart failure and pulmonary
hypertension. In heart failure, there is decreased cardiac output, which will lead to activity
intolerance and excess fluid volume. Furthermore, pulmonary hypertension is presenting and this
will cause the lungs to become congested with fluid. Many patients with a large ASD will have
frequent lung infections from the congestion in the lungs. So, the patient is at risk for infection.

10. True or False: Atrial septal defects can lead to a decrease in lung blood flow

True False
Rationale: ASDs can lead to an INCREASE in lung blood flow (not decrease).

11. The nurse is observing children playing in the hospital playroom. She would expect to see 4
year-old children playing

A. Competitive board games with older children


B. With their own toys along side with other children
C. Alone with hand held computer games
D. Cooperatively with other preschoolers

12. After reviewing the client’s maternal history of magnesium sulfate during labor, which condition
would the nurse anticipate as a potential problem in the neonate?

A. Hypoglycemia

B. Jitteriness

C. Respiratory depression

D. Tachycardia

13. The most common neonatal sepsis and meningitis infections seen within 24 hours after birth
are caused by which organism?

A. Candida albicans

B. Chlamydia trachomatis

C. Escherichia coli

D. Group B beta-hemolytic streptococci

14. The parents of a 2-year-old arrive at a hospital to visit their child. The child is in the playroom
when the parents arrive. When the parents enter the playroom, the child does not readily
approach the parents. The nurse interprets this behavior as indicating that:

A. The child is withdrawn

B. The child is self-centered

C. The child has adjusted to the hospitalized setting

D. This is a normal pattern

15. A 16-year-old is admitted to the hospital for acute appendicitis, and an appendectomy is
performed. Which of the following nursing interventions is most appropriate to facilitate normal
growth and development?
A. Allow the family to bring in the child’s favorite computer games

B. Encourage the parents to room-in with the child


C. Encourage the child to rest and read

D. Allow the child to participate in activities with other individuals in the same age group
when the condition permits

16. A 16 year old child is hospitalized, according to Erik Erikson, what is an appropriate
intervention?

A. tell the friends to visit the child

B. encourage patient to help child learn lessons missed

C. call the priest to intervene

D. tell the child’s girlfriend to visit the child.

17. Children as young as age 3 years can use facial scales for discrimination. What are some
suggested anchor words for the preschool age group?

A. “No hurt.”

B. “Red pain.”

C. “Zero hurt.”

D. “Least pain.”

18. The nurse prepares for a Denver Screening test with a 3 year-old child in the clinic. The mother
asks the nurse to explain the purpose of the test. The BEST response is to tell her that the test
A. Measures potential intelligence
B. Assesses a child’s development
C. Evaluates psychological responses
D. Diagnoses specific problems

19. The nurse is assessing a four month-old infant. The nurse would anticipate finding that the
infant would be able to
A. Hold a rattle
B. Bang two blocks
C. Drink from a cup
D. Wave “bye-bye”

20. Which age group would have a tendency towards eating disorders?
A. Adolescence
B. Toddler hood
C. Childhood
D. Infancy

21. A mother of a 3-year-old tells a clinic nurse that the child is rebelling constantly and having
temper tantrums. The nurse most appropriately tells the mother to:

A. Punish the child every time the child says “no”, to change the behavior

B. Allow the behavior because this is normal at this age period

C. Set limits on the child’s behavior

D. Ignore the child when this behavior occurs

22. A nurse is evaluating the developmental level of a 2-year-old. Which of the following does the
nurse expect to observe in this child?
A. Uses a fork to eat
B. Uses a cup to drink
C. Uses a knife for cutting food
D. Pours own milk into a cup

23. A newborn’s mother is alarmed to find small amounts of blood on her infant girl’s diaper. When
the nurse checks the infant’s urine it is straw colored and has no offensive odor. Which explanation
to the newborn’s mother is most appropriate?

A. “It appears your baby has a kidney infection”

B. “Breast-fed babies often experience this type of bleeding problem due to lack of vitamin C in
the breast milk”

C. “The baby probably passed a small kidney stone”

D. “Some infants experience menstruation like bleeding when hormones from the mother
are not available”

24. Soon after delivery, a neonate is admitted to the central nursery. The nursery nurse begins the
initial assessment by:

A. auscultate bowel sounds.

B. determining chest circumference.

C. inspecting the posture, color, and respiratory effort.

D. checking for identifying birthmarks.

25. The nurse hears the mother of a 5-pound neonate telling a friend on the telephone, “As soon
as I get home, I’ll give him some cereal to get him to gain weight.” The nurse recognizes the need
for further instruction about infant feeding and tells her:

A. “If you give the baby cereal, be sure to use Rice to prevent allergies.”
B. “The baby is not able to swallow cereal, because he is too small.”

C. “The infant’s digestive tract cannot handle complex carbohydrates like cereal.”

D. “If you want him to gain weight, just double his daily intake of formula.”

Correct Answer: C. “The infant’s digestive tract cannot handle complex carbohydrates like cereal.”

Option C: An infant’s digestive system is still developing and is not yet ready to carry out the
complex tasks of masticating (liquefying) and digesting (breaking down) foods. The breakdown of
more complex starches occurs in the small intestine and involves an enzyme called pancreatic
amylase. There are widely respected experts in pediatric gastroenterology, who assert that this
essential enzyme does not appear until close to eighteen months of age and certainly not before
twelve months. Feeding infants foods that they cannot digest properly merely leads to the
decomposition of these foods in their intestines and the associated challenges which result.

26. The nurse is preparing to discharge a multipara 24 hours after a vaginal delivery. The client is
breastfeeding her newborn. The nurse instructs the client that if engorgement occurs the client
should:
A. wear a tight fitting bra or breast binder.
B. apply warm, moist heat to the breasts.
C. contact the nurse-midwife for a lactation suppressant.
D. restrict fluid intake to 1000 ml daily.

Correct Answer: B. apply warm, moist heat to the breasts.

Option B: Moist heat has this amazing ability to increase circulation, open milk ducts and stimulate
let down – all of which encourage the milk to start flowing. Option A: If a bra is worn, it should be
big enough or stretchy enough to allow for expansion if breasts fill during the night hours; a bra that
is too tight can cause soreness and potential problems such as blocked ducts. Option C: The
simplest and safest way to suppress lactation is to let milk production stop on its own. Option D:
Research has found that nursing mothers do not need to drink more fluids than what's necessary
to satisfy their thirst.
MATERNAL AND CHILD (well)

1. When assessing the adequacy of sperm for conception to occur, which of the following is the
most useful criterion?

A. Sperm count

B. Sperm motility

C. Sperm maturity

D. Semen volume

Rationale. B. Although all of the factors listed are important, sperm motility is the most
significant criterion when assessing male infertility. Sperm count, sperm maturity, and
semen volume are all significant, but they are not as significant sperm motility

2. Which of the following urinary symptoms does the pregnant woman most frequently experience
during the first trimester?

a. Dysuria

b. Frequency

c. Incontinence

d. Burning

Rationale. B. Pressure and irritation of the bladder by the growing uterus during the first
trimester is responsible for causing urinary frequency. Dysuria, incontinence, and burning
are symptoms associated with urinary tract infections.

3. In which of the following areas would the nurse expect to observe chloasma?

a. Breast, areola, and nipples

b. Chest, neck, arms, and legs

c. Abdomen, breast, and thighs

d. Cheeks, forehead, and nose

Rationale. D. Chloasma, also called the mask of pregnancy, is an irregular hyperpigmented


area found on the face. It is not seen on the breasts, areola, nipples, chest, neck, arms,
legs, abdomen, or thighs.

4. Cervical softening and uterine souffle are classified as which of the following?

a. Diagnostic signs

b. Presumptive signs
c. Probable signs

d. Positive signs

Rationale. C. Cervical softening (Goodell sign) and uterine soufflé are two probable signs of
pregnancy. Probable signs are objective findings that strongly suggest pregnancy. Other
probable signs include Hegar sign, which is softening of the lower uterine segment;
Piskacek sign, which is enlargement and softening of the uterus; serum laboratory tests;
changes in skin pigmentation; and ultrasonic evidence of a gestational sac. Presumptive
signs are subjective signs and include amenorrhea; nausea and vomiting; urinary
frequency; breast tenderness and changes; excessive fatigue; uterine enlargement; and
quickening.

5. Which of the following would the nurse identify as a presumptive sign of pregnancy?

a. Hegar sign

b. Nausea and vomiting

c. Skin pigmentation changes

d. Positive serum pregnancy test

Rationale. B. Presumptive signs of pregnancy are subjective signs. Of the signs listed, only
nausea and vomiting are presumptive signs. Hegar sign,skin pigmentation changes, and a
positive serum pregnancy test are considered probably signs, which are strongly
suggestive of pregnancy.

6. A pregnant client states that she “waddles” when she walks. The nurse’s explanation is based
on which of the following as the cause?

a. The large size of the newborn

b. Pressure on the pelvic muscles

c. Relaxation of the pelvic joints

d. Excessive weight gain

Rationale. C. During pregnancy, hormonal changes cause relaxation of the pelvic joints,
resulting in the typical “waddling” gait. Changes in posture are related to the growing fetus.
Pressure on the surrounding muscles causing discomfort is due to the growing uterus.
Weight gain has no effect on gait.

7. FHR can be auscultated with a fetoscope as early as which of the following?

a. 5 weeks gestation

b. 10 weeks gestation

c. 15 weeks gestation

d. 20 weeks gestation
8. Which of the following fundal heights indicates less than 12 weeks’ gestation when the date of
the LMP is unknown?

a. Uterus in the pelvis

b. Uterus at the xiphoid

c. Uterus in the abdomen

d. Uterus at the umbilicus

9. Which of the following would cause a false-positive result on a pregnancy test?

a. The test was performed less than 10 days after an abortion

b. The test was performed too early or too late in the pregnancy

c. The urine sample was stored too long at room temperature

d. A spontaneous abortion or a missed abortion is impending

10. Which of the following nursing interventions would the nurse perform during the third stage of
labor?

a. Obtain a urine specimen and other laboratory tests.

b. Assess uterine contractions every 30 minutes.

c. Coach for effective client pushing

d. Promote parent-newborn interaction.

Rationale. D. During the third stage of labor, which begins with the delivery of the newborn,
the nurse would promote parent-newborn interaction by placing the newborn on the
mother’s abdomen and encouraging the parents to touch the newborn. Collecting a urine
specimen and other laboratory tests is done on admission during the first stage of labor.
Assessing uterine contractions every 30 minutes is performed during the latent phase of
the first stage of labor. Coaching the client to push effectively is appropriate during the
second stage of labor

11. When talking with a pregnant client who is experiencing aching swollen leg veins, the nurse
would explain that this is most probably the result of which of the following?

a. Thrombophlebitis

b. Pregnancy-induced hypertension

c. Pressure on blood vessels from the enlarging uterus

d. The force of gravity pulling down on the uterus

Rationale. C. Pressure of the growing uterus on blood vessels results in an increased risk
for venous stasis in the lower extremities. Subsequently, edema and varicose vein
formation may occur. Thrombophlebitis is an inflammation of the veins due to thrombus
formation. Pregnancy-induced hypertension is not associated with these symptoms.
Gravity plays only a minor role with these symptoms.

12. Immediately before expulsion, which of the following cardinal movements occur?

a. Descent

b. Flexion

c. Extension

d. External rotation

Rationale. D. Immediately before expulsion or birth of the rest of the body, the cardinal
movement of external rotation occurs. Descent flexion, internal rotation, extension, and
restitution (in this order) occur before external rotation.

13. When taking an obstetrical history on a pregnant client who states, “I had a son born at 38
weeks gestation, a daughter born at 30 weeks gestation and I lost a baby at about 8 weeks,”
the nurse should record her obstetrical history as which of the following?

a. G2 T2 P0 A0 L2

b. G3 T1 P1 A0 L2

c. G3 T2 P0 A0 L2

d. G4 T1 P1 A1 L2

Rationale. D. The client has been pregnant four times, including current pregnancy (G).
Birth at 38 weeks’ gestation is considered full term (T), while birth form 20 weeks to 38
weeks is considered preterm (P). A spontaneous abortion occurred at 8 weeks (A). She has
two living children (L).

14. Which of the following refers to the single cell that reproduces itself after conception?

a. Chromosome

b. Blastocyst

c. Zygote

d. Trophoblast

Rationale. C . The zygote is the single cell that reproduces itself after conception. The
chromosome is the material that makes up the cell and is gained from each parent.
Blastocyst and trophoblast are later terms for the embryo after zygote.

15. A postpartum client has a temperature of 101.4ºF, with a uterus that is tender when palpated,
remains unusually large, and not descending as normally expected. Which of the following
should the nurse assess next?
a. Lochia

b. Breasts

c. Incision

d. Urine

Rationale. A. The data suggests an infection of the endometrial lining of the uterus. The
lochia may be decreased or copious, dark brown in appearance, and foul smelling,
providing further evidence of a possible infection. All the client’s data indicate a uterine
problem, not a breast problem. Typically, transient fever, usually 101ºF, may be present with
breast engorgement. Symptoms of mastitis include influenza-like manifestations. Localized
infection of an episiotomy or C-section incision rarely causes systemic symptoms, and
uterine involution would not be affected. The client data do not include dysuria, frequency,
or urgency, symptoms of urinary tract infections, which would necessitate assessing the
client’s urine.

16. When teaching a client about contraception. Which of the following would the nurse include as
the most effective method for preventing sexually transmitted infections?

a. Spermicides

b. Diaphragm

c. Condoms

d. Vasectomy

Rationale. C. Condoms, when used correctly and consistently, are the most effective
contraceptive method or barrier against bacterial and viral sexually transmitted infections.
Although spermicides kill sperm, they do not provide reliable protection against the spread
of sexually transmitted infections, especially intracellular organisms such as HIV. Insertion
and removal of the diaphragm along with the use of the spermicides may cause vaginal
irritations, which could place the client at risk for infection transmission. Male sterilization
eliminates spermatozoa from the ejaculate, but it does not eliminate bacterial and/or viral
microorganisms that can cause sexually transmitted infections.

17. When performing a pelvic examination, the nurse observes a red swollen area on the right side
of the vaginal orifice. The nurse would document this as enlargement of which of the following?

a. Clitoris

b. Parotid gland

c. Skene’s gland

d. Bartholin’s gland

Rationale. D. Bartholin’s glands are the glands on either side of the vaginal orifice. The
clitoris is female erectile tissue found in the perineal area above the urethra. The parotid
glands are open into the mouth. Skene’s glands open into the posterior wall of the female
urinary meatus.
18. A nurse is assessing a client in the 4th stage of labor and notes that the fundus is firm but that
bleeding is excessive. The initial nursing action would be which of the following?

a. Massage the fundus

b. Place the mother in the Trendelenburg’s position

c. Notify the physician

d. Record the findings

Rationale. C. Notify the physician. If the bleeding is excessive, the cause may be laceration
of the cervix or birth canal. Massaging the fundus if it is firm will not assist in controlling
the bleeding. Trendelenburg’s position is to be avoided because it may interfere with
cardiac function.

19. Methergine or pitocin is prescribed for a woman to treat PP hemorrhage. Before administration
of these medications, the priority nursing assessment is to check the:

a. Amount of lochia

b. Blood pressure

c. Deep tendon reflexes

d. Uterine tone

Rationale. B. Blood pressure. Methergine and pitocin are agents that are used to prevent or
control postpartum hemorrhage by contracting the uterus. They cause continuous uterine
contractions and may elevate blood pressure. A priority nursing intervention is to check
blood pressure. The physician should be notified if hypertension is present.

20. Which of the following interventions would be helpful to a breastfeeding mother who is
experiencing engorged breasts?

a. Applying ice

b. Applying a breast binder

c. Teaching how to express her breasts in a warm shower

d. Administering bromocriptine (Parlodel)

Rationale. C. Teaching how to express her breasts in a warm shower. Teaching the client
how to express her breasts in a warm shower aids with let-down and will give temporary
relief. Ice can promote comfort by vasoconstriction, numbing, and discouraging further
letdown of milk.

21. On the first PP night, a client requests that her baby be sent back to the nursery so she can get
some sleep. The client is most likely in which of the following phases?

a. Depression phase
b. Letting-go phase

c. Taking-hold phase

d. Taking-in phase

22. A nurse in the labor room is caring for a client in the active phases of labor. The nurse is
assessing the fetal patterns and notes a late deceleration on the monitor strip. The most
appropriate nursing action is to:

a. Place the mother in the supine position

b. Document the findings and continue to monitor the fetal patterns

c. Administer oxygen via face mask

d. Increase the rate of pitocin IV infusion

23. A laboring client complains of low back pain. The nurse replies that this pain occurs most when
the position of the fetus is:

a. Breech

b. Transverse

c. Occiput anterior

d. Occiput posterior

24. A nurse is describing the process of fetal circulation to a client during a prenatal visit. The
nurse accurately tells the client that fetal circulation consists of:

a. Two umbilical veins and one umbilical artery

b. Two umbilical arteries and one umbilical vein

c. Arteries carrying oxygenated blood to the fetus

d. Veins carrying deoxygenated blood to the fetus

Rationale. B. Two umbilical arteries and one umbilical vein. Blood pumped by the embryo’s
heart leaves the embryo through two umbilical arteries. Once oxygenated, the blood then is
returned by one umbilical vein. Arteries carry deoxygenated blood and waste products from
the fetus, and veins carry oxygenated blood and provide oxygen and nutrients to the fetus.

25. A nurse is performing an assessment of a primipara who is being evaluated in a clinic during
her second trimester of pregnancy. Which of the following indicates an abnormal physical
finding necessitating further testing?

a. Consistent increase in fundal height

b. Fetal heart rate of 180 BPM


c. Braxton hicks contractions

d. Quickening

Rationale. B. Fetal heart rate of 180 BPM. The normal range of the fetal heart rate depends
on gestational age. The heart rate is usually 160-170 BPM in the first trimester and slows
with fetal growth, near and at term, the fetal heart rate ranges from 120-160 BPM. The other
options are expected.

26. Before birth, which of the following structures connects the right and left auricles of the heart?

a. Umbilical vein

b. Foramen ovale

c. Ductus arteriosus

d. Ductus venosus

27. Which of the following is true regarding the fontanels of the newborn?

a. The anterior is triangular shaped; the posterior is diamond shaped.

b. The posterior closes at 18 months; the anterior closes at 8 to 12 weeks.

c. The anterior is large in size when compared to the posterior fontanel.

d. The anterior is bulging; the posterior appears sunken.

Rationale. C. The anterior fontanel is larger in size than the posterior fontanel. Additionally,
the anterior fontanel, which is diamond shaped, closes at 18 months, whereas the posterior
fontanel, which is triangular shaped, closes at 8 to 12 weeks. Neither fontanel should
appear bulging, which may indicate increased intracranial pressure, or sunken, which may
indicate dehydration.

28. A nurse prepares to administer a vitamin K injection to a newborn infant. The mother asks the
nurse why her newborn infant needs the injection. The best response by the nurse would be:

a. “Your infant needs vitamin K to develop immunity.”

b. “The vitamin K will protect your infant from being jaundiced.”

c. “Newborn infants are deficient in vitamin K, and this injection prevents your infant
from abnormal bleeding.”

d. “Newborn infants have sterile bowels, and vitamin K promotes the growth of bacteria in
the bowel.”

Rationale. C. “Newborn infants are deficient in vitamin K, and this injection prevents your
infant from abnormal bleeding.” Vitamin K is necessary for the body to synthesize
coagulation factors. Vitamin K is administered to the newborn infant to prevent abnormal
bleeding. Newborn infants are vitamin K deficient because the bowel does not have the
bacteria necessary for synthesizing fat-soluble vitamin K. The infant’s bowel does not have
support the production of vitamin K until bacteria adequately colonizes it by food ingestion.

29. The primary critical observation for Apgar scoring is the:

a. Heart rate

b. Respiratory rate

c. Presence of meconium

d. Evaluation of the Moro reflex

30. A nurse in a newborn nursery is performing an assessment of a newborn infant. The nurse is
preparing to measure the head circumference of the infant. The nurse would most
appropriately:

a. Wrap the tape measure around the infant’s head and measure just above the eyebrows.

b. Place the tape measure under the infants head at the base of the skull and wrap around
to the front just above the eyes

c. Place the tape measure under the infants head, wrap around the occiput, and
measure just above the eyes

d. Place the tape measure at the back of the infant’s head, wrap around across the ears,
and measure across the infant’s mouth.

Rationale. C. Place the tape measure under the infants head, wrap around the occiput, and
measure just above the eyes. To measure the head circumference, the nurse should place
the tape measure under the infant’s head, wrap the tape around the occiput, and measure
just above the eyebrows so that the largest area of the occiput is included.

31. Vitamin K is prescribed for a neonate. A nurse prepares to administer the medication in which
muscle site?

a. Deltoid

b. Triceps

c. Vastus lateralis

d. Biceps

32. A nursing instructor asks a nursing student to describe the procedure for administering
erythromycin ointment into the eyes of a neonate. The instructor determines that the student
needs to research this procedure further if the student states:

a. “I will cleanse the neonate’s eyes before instilling ointment.”

b. “I will flush the eyes after instilling the ointment.”


c. “I will instill the eye ointment into each of the neonate’s conjunctival sacs within one hour
after birth.”

d. “Administration of the eye ointment may be delayed until an hour or so after birth so that
eye contact and parent-infant attachment and bonding can occur.”

Rationale. B. “I will flush the eyes after instilling the ointment.” Eye prophylaxis protects
the neonate against Neisseria gonorrhoeae and Chlamydia trachomatis. The eyes are not
flushed after instillation of the medication because the flush will wash away the
administered medication.

33. A baby is born precipitously in the ER. The nurses initial action should be to:

A. Establish an airway for the baby

B. Ascertain the condition of the fundus

C. Quickly tie and cut the umbilical cord

D. Move mother and baby to the birthing unit

Rationale. A. Establish an airway for the baby. The nurse should position the baby with
head lower than chest and rub the infant’s back to stimulate crying to promote oxygenation.
There is no haste in cutting the cord.

34. The primary critical observation for Apgar scoring is the:

A. Heart rate

B. Respiratory rate

C. Presence of meconium

D. Evaluation of the Moro reflex

35. When performing a newborn assessment, the nurse should measure the vital signs in the
following sequence:

A. Pulse, respirations, temperature

B. Temperature, pulse, respirations

C. Respirations, temperature, pulse

D. Respirations, pulse, temperature

36. Within 3 minutes after birth the normal heart rate of the infant may range between:

A. 100 and 180

B. 130 and 170

C. 120 and 160


D. 100 and 130

Rationale. C. 120 and 160. The heart rate varies with activity; crying will increase the rate,
whereas deep sleep will lower it; a rate between 120 and 160 is expected.

37. The nurse is aware that a healthy newborn’s respirations are:

A. Regular, abdominal, 40-50 per minute, deep

B. Irregular, abdominal, 30-60 per minute, shallow

C. Irregular, initiated by chest wall, 30-60 per minute, deep

D. Regular, initiated by the chest wall, 40-60 per minute, shallow

Rationale. B. Irregular, abdominal, 30-60 per minute, shallow. Normally the newborn’s
breathing is abdominal and irregular in depth and rhythm; the rate ranges from 30-60
breaths per minute.

38. A newborn has small, whitish, pinpoint spots over the nose, which the nurse knows are caused
by retained sebaceous secretions. When charting this observation, the nurse identifies it as:

A. Milia

B. Lanugo

C. Whiteheads

D. Mongolian spots

Rationale. A. Milia. Milia occur commonly, are not indicative of any illness, and eventually
disappear.

39. When teaching umbilical cord care to a new mother, the nurse would include which
information?

A. Apply peroxide to the cord with each diaper change

B. Cover the cord with petroleum jelly after bathing

C. Keep the cord dry and open to air

D. Wash the cord with soap and water each day during a tub bath

Rationale. C. Keep the cord dry and open to air. Keeping the cord dry and open to air helps
reduce infection and hastens drying.

40. A mother of a term neonate asks what the thick, white, cheesy coating is on his skin. Which
correctly describes this finding?

A. Lanugo
B. Milia

C. Nevus flammeus

D. Vernix

41. Which condition or treatment best ensures lung maturity in an infant?

A. Meconium in the amniotic fluid

B. Glucocorticoid treatment just before delivery

C. Lecithin to sphingomyelin ratio more than 2:1

D. Absence of phosphatidylglycerol in amniotic fluid

Rationale. C. Lecithin to sphingomyelin ratio more than 2:1. Lecithin and sphingomyelin are
phospholipids that help compose surfactant in the lungs; lecithin peaks at 36 weeks and
sphingomyelin concentrations remain stable.

42. When performing nursing care for a neonate after a birth, which intervention has the highest
nursing priority?

A. Obtain a dextrostix

B. Give the initial bath

C. Give the vitamin K injection

D. Cover the neonates head with a cap

Rationale. D. Cover the neonates head with a cap. Covering the neonates head with a cap
helps prevent cold stress due to excessive evaporative heat loss from the neonate’s wet
head. Vitamin K can be given up to 4 hours after birth.

43. A woman delivers a 3.250 g neonate at 42 weeks’ gestation. Which physical finding is
expected during an examination if this neonate?

A. Abundant lanugo

B. Absence of sole creases

C. Breast bud of 1-2 mm in diameter

D. Leathery, cracked, and wrinkled skin

Rationale. D. Leathery, cracked, and wrinkled skin. Neonatal skin thickens with maturity and
is often peeling by post term.

44. Which age group has the greatest potential to demonstrate regression when they are sick?
a. Infant

b. Toddler

c. Adolescent

d. Young adult

Rationale. B. Regression is most seen among toddlers and it can be caused by stressful
situations such as hospitalization, the arrival of a new sibling, or starting a new school.
When a child regresses, he or she appears to be going backward in an earlier stage of
development where he or she feels comfortable (e.g. toilet trained toddlers suddenly start
wetting their pants when they become sick, thumbsucking).

45. Which stage of development is most unstable and challenging regarding the development of
personal identity?

a. Adolescence

b. Toddlerhood

c. Middle childhood

d. Young adulthood

Rationale. A. Although it occurs throughout one’s lifetime, identity development is


considered to be the primary psychosocial task of adolescence or as described by Erickson
on identity versus identity confusion. Individuals in this stage start to integrate their values,
abilities, inner drives, and past experiences into who they are as persons.

46. A nurse is evaluating the developmental level of a two (2)-year-old. Which of the following does
the nurse expect to observe in this child?

a. Uses a fork to eat

b. Uses a cup to drink

c. Uses a knife in cutting foods

d. Pours own milk into the cup

Rationale. B. By age 2 years, the child can use a cup and can use a spoon correctly but with
some spilling. Children can start learning how to use a cup without a lid when they are 9
months old. Most experts recommend introducing utensils between 10 and 12 months, as
an almost-toddler starts to show signs that she’s interested. A spoon should be first on the
child’s tray since it’s easier to use.

47. A maternity nurse is providing instruction to a new mother regarding the psychosocial
development of the newborn infant. Using Erikson’s psychosocial development theory, the
nurse would instruct the mother to

a. Allow the newborn infant to signal a need


b. Anticipate all of the needs of the newborn infant

c. Avoid the newborn infant during the first 10 minutes of crying

d. Allow the infant to cry, once lessen, then attend to the infant

Rationale. A. If a newborn is not allowed to signal a need, the newborn will not learn how to
control the environment. The primary way the caregiver can build trust with the baby is to
respond when they try to communicate. Because babies can’t use words to express
themselves, they use nonverbal strategies to communicate what they’re thinking and
feeling at all times.

48. The parents of a two (2)-year-old boy arrive at a hospital for a visit. The child is in the playroom
when the parents arrive. When the parents enter the playroom, the child does not readily
approach the parents. The nurse interprets this behavior as indicating that:

a. The child is withdrawn

b. The child is self-centered

c. The child has adjusted to the hospitalized setting

d. This is a normal pattern

49. A mother of a three (3)-year-old tells a clinic nurse that the child is constantly rebelling and
having temper tantrums. The nurse most appropriately tells the mother to:

a. Punish the child every time the child says “no”, to change the behavior

b. Allow the behavior because this is normal at this age period

c. Set limits on the child’s behavior

d. Ignore the child when this behavior occurs

Rationale. C. Being consistent and setting limits on the child’s behavior are the necessary
elements in order for the toddler to learn what is acceptable so they can understand the
impact of their behaviors.

50. A clinic nurse assesses the communication patterns of a five (5)-month-old infant. The nurse
determines that the infant is demonstrating the highest level of developmental achievement
expected if the infant:

a. Uses simple words such as “mama”

b. Uses monosyllabic babbling

c. Links syllables together

d. Coos when comforted

Rationale. B. Monosyllabic babbling occurs between 3 and 6 months of age. The infant
starts to produce vowels and combines them with consonants, producing syllables (e.g.,
ba, da, la, ga). An infant should be babbling away by now, and those babbles might even be
starting to sound like real words. Five-month-olds can begin to put consonant and vowel
sounds together.

Post Quiz-Antenatal Introduction &


Complications -MQ1
1. An 18 years old presents at the emergency department with complaints of lower abdominal
cramping and spotting at 12 weeks' gestation. The physician performs a pelvic examination and
finds that the cervix is closed. What does the physician suspect is the cause of the cramps and
spotting?*

a) Cervical insufficiency

b) Threatened abortion

c) Habitual abortion

d) Ectopic pregnancy

Rationale:

Spontaneous abortion occurs along a continuum: threatened, inevitable, incomplete, complete,


missed. The definition of each category is related to whether or not the uterus is emptied, or for
how long the products of conception are retained.

2. A 21 years old woman is admitted with a diagnosis of ectopic pregnancy. For which of the
following would you anticipate beginning preparation?

a) Bed rest for the next 4 weeks.

b) Intravenous administration of a tocolytic.

c) Immediate surgery.

d) Internal uterine monitoring.

Ectopic pregnancy means an embryo has implanted outside the uterus, usually in the fallopian
tube. Surgery is usually necessary to remove the growing structure before the tube ruptures or
repair the tube if rupture has already occurred. Bed rest will not correct the problem of an ectopic
pregnancy. Administering a tocolytic is not indicated, nor is internal uterine monitoring. This makes
options A, B, and D incorrect
3. A client at 9weeks' gestation experiences pregnancy loss. The client asks the nurse if the
bleeding and cramping that occurred during the miscarriage were caused by working long hours in
a stressful environment. What is the most appropriate response from the nurse?

a) "I can understand your need to find an answer to what caused this. Let's talk about this further."
b) "It is hard to know why a woman bleeds during early pregnancy."
c) "Your spontaneous bleeding is not work-related."
d) "Something was wrong with the fetus."

Ectopic pregnancy means an embryo has implanted outside the uterus, usually in the fallopian
tube. Surgery is usually necessary to remove the growing structure before the tube ruptures or
repair the tube if rupture has already occurred. Bed rest will not correct the problem of an ectopic
pregnancy. Administering a tocolytic is not indicated, nor is internal uterine monitoring. This makes
options A, B, and D incorrect

4. RhoGAM is given to Rh-negative women to prevent maternal sensitization. In addition to


pregnancy, Rh-negative women would also receive this medication after which of the following?*
a. Therapeutic or spontaneous abortion
b. Head injury from a car accident
c. Blood transfusion after a hemorrhage
d. Unsuccessful artificial insemination procedure

Any time there is a pregnancy with the chance of maternal and fetal blood mixing, RhoGAM is
needed to prevent sensitization or antibody production. Head injury resulting from a car crash is
not a situation in which there would be mixing of fetal or maternal blood. The trauma would cause
hemorrhage, but not a sensitization reaction. A blood transfusion after hemorrhage would require
typing and cross-matching of the client's blood; thus, she would receive blood with her own Rh
factor, not one with Rh-positive blood. Because the artificial insemination procedure was
unsuccessful, no pregnancy occurred and RhoGAM would not be necessary.

5. A pregnant woman, approximately 10 weeks' gestation, comes to the emergency department


after calling her health care provider's office and reporting moderate vaginal bleeding. Assessment
reveals cervical dilation and moderately strong abdominal cramps. She reports that she has
passed some tissue with the bleeding. The nurse interprets these findings to suggest which of the
following?*
a. Threatened abortion
b. Inevitable abortion
c. Incomplete abortion
d. Missed abortion

6. A nurse in the maternity triage unit is caring for a client with a suspected ectopic pregnancy.
Which nursing intervention should the nurse perform first?*
A. Assess the client's referral and history
B. Assess the client's emotional response
C. Assess the client's vital signs.
D. Assess the client's family status

7. A woman with an incomplete abortion is to receive misoprostol (Cytotec). The woman asks the
nurse, "Why am I getting this drug?" The nurse responds to the client, integrating understanding
that this drug achieves which effect?*
A. ensure rest and enough fluid intake
B. ensure family support
C. ensures passage of all the products of conception
D. ensure normal vital signs

8. Upon entering the room of a client who has had a spontaneous abortion, the nurse observes the
client crying. Which response by the nurse would be most appropriate?*
A. "I'm sorry you lost your baby."
B. "Don't worry, you will have another baby soon"
C. "You can't imagine the numbers of mothers having abortion"
D. "I'm sure the physician will talk to you and to your husband

9. A pregnant patient with a history of premature cervical dilatation undergoes cervical cerclage.
Which outcome indicates that this procedure has been successful?*
A. The client delivers a full-term fetus at 37 weeks' gestation.
B. The client delivers a full-term fetus at 36 weeks' gestation.
C. The client delivers a full-term fetus at 39 weeks' gestation.
D. The client delivers a full-term fetus at 35 weeks' gestation.

10. Which medication will the nurse anticipate the health care provider will prescribe as treatment
for an unruptured ectopic pregnancy?*
A. oxytocin
B. methotrexate
C. promethazine
D. methergin
11. A nurse is conducting an in-service program for a group of nurses working at the women's
health facility about the causes of spontaneous abortion. The nurse determines that the teaching
was successful when the group identifies which condition as the most common cause of first
trimester abortions?*
A. uterine fibroids
B. cervical insufficiency
C. fetal genetic abnormalities
D. maternal disease

12. Which of the following is not a first Trimester Complications of Pregnancy?*

A. Abortion (Miscarriage)

B. Incompetent cervical OS

C. Ectopic pregnancy

D. Placenta previa

13. Which of the following is inappropriate for High Risk Pregnancy Nursing diagnosis?*
A. Knowledge deficit related to normal changes of pregnancy versus illness complication
B. Fear regarding financial capacity related to chronic illness
C. Health-seeking behaviors related to effects of illness of pregnancy
D. Situational self-esteem related to illness during pregnacy

14. Which of the following is not a danger sign of pregnancy?*


A. Headache 3/10 early in the morning
B. Abdominal & chest pain
C. Visual changes
D. Vomiting persistent

15. Which of the following is categorize as an infection caused by organisms that can cross the
placenta by eating raw foods or poorly cook , contact w/ feces of infected animals & soil?*

A. Cytomegalovirus

B. Toxoplasmosis

C. Herpes simplex

D. Syphilis

16. Which of the following is the causative agent of syphilis?*


A. HSV
toxoplasma gondii
Treponema pallidum
All of the above

Threatened Abortion - Bleeding w/ closed cervix

Ectopic Pregnancy - knife like abdominal pain with cullen's sign

Incompetent cervix - pink stained vaginal discharged

Missed Abortion - Intermittent bleeding with absence of uterine growth

Pre-Test Week 4 Maternal Prenatal High


Risk Complications
1. A 39-year-old at 37 weeks gestation is admitted to the hospital with complaints of vaginal
bleeding following the use of cocaine 1 hour earlier. Which complication is most likely
causing the client’s complaint of vaginal bleeding?*

A. Placenta previa

B. Abruptio placentae

C. Ectopic pregnancy

D. Spontaneous abortion

Answer: B. Abruptio placentae

The major maternal adverse reactions from cocaine use in pregnancy include spontaneous
abortion first, not third, trimester abortion and abruptio placentae

2. Which of the following is the most common kind of placental adherence seen in pregnant
women?*

A. Accreta
B. Placenta previa
C. Percreta
D. Increta

Answer: A. Accreta

Placenta accreta is the most common kind of placental adherence seen in pregnant women and is
characterized by slight penetration of myometrium. In placenta previa, the placenta does not
embed correctly and results in what is known as a low-lying placenta. It can be marginal, partial, or
complete in how it covers the cervical os, and it increases the patient’s risk for painless vaginal
bleeding during the pregnancy and/or delivery process. Placenta percreta leads to perforation of
the uterus and is the most serious and invasive of all types of accrete. Placenta increta leads to
deep penetration of the myometrium.
3. A pregnant client is diagnosed with partial placenta previa. In explaining the diagnosis, the
nurse tells the client that the usual treatment for partial placenta previa is which of the
following?*
A. Activity limited to bed rest
B. Platelet infusion
C. Immediate cesarean delivery
D. Labor induction with oxytocin

4. A nurse in the postpartum unit is caring for a client who has just delivered a newborn infant
following a pregnancy with placenta previa. The nurse reviews the plan of care and
prepares to monitor the client for which of the following risks associated with placenta
previa?*
A. Disseminated intravascular coagulation
B. Chronic hypertension
C. Infection
D. Hemorrhage

Because the placenta is implanted in the lower uterine segment, which does not contain the same
intertwining musculature as the fundus of the uterus, this site is more prone to bleeding.

5. Maureen in her third trimester arrives at the emergency room with painless vaginal
bleeding. Which of the following conditions is suspected?*
a. Placenta previa
b. Abruptio placentae
c. Premature labor
d. Sexually transmitted disease

Placenta previa with painless vaginal bleeding.


6. Tina experienced painless vaginal bleeding has just been diagnosed as having a placenta
previa. Which of the following procedures is usually performed to diagnose placenta
previa?*

a. Amniocentesis

b. Digital or speculum examination

c. External fetal monitoring

d. Ultrasound

Answer: D. Ultrasound

Rationale: Once the mother and the fetus are stabilized, ultrasound evaluation of the placenta
should be done to determine the cause of the bleeding. Amniocentesis is contraindicated in
placenta previa. A digital or speculum examination shouldn’t be done as this may lead to severe
bleeding or hemorrhage. External fetal monitoring won’t detect a placenta previa, although it will
detect fetal distress, which may result from blood loss or placenta separation.

7. The common normal site of nidation/implantation in the uterus is:*


A. Upper uterine portion
B. Mid-uterine area
C. Lower uterine segment
D. Lower cervical segment

Answer: A. Upper uterine portion

The embryo’s normal nidation site is the upper portion of the uterus. If the implantation is in the
lower segment, this is an abnormal condition called placenta previa.

8. A 39-year-old at 37 weeks gestation is admitted to the hospital with complaints of vaginal


bleeding following the use of cocaine 1 hour earlier. Which complication is most likely
causing the client’s complaint of vaginal bleeding?*
A. Placenta previa
B. Abruptio placentae
C. Ectopic pregnancy
D. Spontaneous abortion

Answer: B. Abruptio placentae

The major maternal adverse reactions from cocaine use in pregnancy include spontaneous
abortion first, not third, trimester abortion and abruptio placentae.
9. A pregnant woman arrives at the emergency department with abruptio placentae at 34
weeks’ gestation. She’s at risk for which of the following blood dyscrasias?*
A. Thrombocytopenia.
B. Idiopathic thrombocytopenic purpura (ITP).
C. Disseminated intravascular coagulation (DIC).
D. Heparin-associated thrombosis and thrombocytopenia (HATT).

Answer: C. Disseminated intravascular coagulation (DIC).

Abruptio placentae is a cause of DIC because it activates the clotting cascade after hemorrhage.

Option A: Thrombocytopenia results from decreased production of platelets.

Option B: ITP doesn’t have a definitive cause.

Option D: A patient with abruptio placentae wouldn’t get heparin and, as a result, wouldn’t be at
risk for HATT.

10. A nurse is assessing a pregnant client in the 2nd trimester of pregnancy who was admitted
to the maternity unit with a suspected diagnosis of abruptio placentae. Which of the
following assessment findings would the nurse expect to note if this condition is present?*

A. Absence of abdominal pain


B. A soft abdomen
C. Uterine tenderness/pain
D. Painless, bright red vaginal bleeding

Answer: C. Uterine tenderness/pain.

In abruptio placentae, acute abdominal pain is present. Uterine tenderness and pain accompany
placental abruption, especially with a central abruption and trapped blood behind the placenta. The
abdomen will feel hard and board-like on palpation as the blood penetrates the myometrium and
causes uterine irritability. Observation of the fetal monitoring often reveals increased uterine resting
tone, caused by the failure of the uterus to relax in an attempt to constrict blood vessels and
control bleeding.

11. An ultrasound is performed on a client at term gestation that is experiencing moderate


vaginal bleeding. The results of the ultrasound indicate that an abruptio placentae is
present. Based on these findings, the nurse would prepare the client for:*
A. Complete bed rest for the remainder of the pregnancy
B. Delivery of the fetus
C. Strict monitoring of intake and output
D. The need for weekly monitoring of coagulation studies until the time of delivery

Answer: B. Delivery of the fetus.

The goal of management in abruptio placentae is to control the hemorrhage and deliver the fetus
as soon as possible. Delivery is the treatment of choice if the fetus is at term gestation or if the
bleeding is moderate to severe and the mother or fetus is in jeopardy.

The team of health care providers could very well be the key towards the survival and safety of
both the mother and the fetus. The role of the support system is also essential for this situation.
Finally, the willpower of the mother to survive with both of their lives intact could be the turning
point for those who care deeply for the patient and the child.

12. A client makes a routine visit to the prenatal clinic. Although she’s 14 weeks pregnant, the
size of her uterus approximates that in an 18- to 20-week pregnancy. Dr. Diaz diagnoses
gestational trophoblastic disease and orders ultrasonography. The nurse expects
ultrasonography to reveal:*

A. an empty gestational sac.


B. grapelike clusters.
C. a severely malformed fetus.
D. an extrauterine pregnancy.

In a client with gestational trophoblastic disease, an ultrasound performed after the 3rd month
shows grapelike clusters of transparent vesicles rather than a fetus. The vesicles contain a clear
fluid and may involve all or part of the decidual lining of the uterus. Usually no embryo (and
therefore no fetus) is present because it has been absorbed. Because there is no fetus, there can
be no extrauterine pregnancy. An extrauterine pregnancy is seen with an ectopic pregnancy.
13. Which of the following signs will require a mother to seek immediate medical attention?*

A. When the first fetal movement is felt


B. No fetal movement is felt on the 6th month
C. Mild uterine contraction
D. Slight dyspnea on the last month of gestation

14. Which of the following cause a “complete” molar pregnancy: Select all that applies:*

A. placenta grows and produces hCG


B. sperm fertilizes empty egg

C. no fetus is formed

15. A client is being admitted to the antepartum unit for hypovolemia secondary to hyperemesis
gravidarum. Which of the following factors predisposes a client to the development of this?*

A. trophoblastic disease
B. maternal age > 35 y.o.
C. malnourished or underweight clients
D. low levels of HCG

Trophoblastic disease is associated w/ hyperemesis gravidarum obesity and maternal age younger
than 20 y.o. are risk factors too. High levels of estrogen HCG have also been associated with the
development

16. A 21-year old client, 6 weeks’ pregnant is diagnosed with hyperemesis gravidarum. This
excessive vomiting during pregnancy will often result in which of the following conditions?*

A. Bowel perforation
B. Electrolyte imbalance
C. Miscarriage
D. Pregnancy induced hypertension (PIH)

17. Which of the following statements best describes hyperemesis gravidarum?*

A. Severe anemia leading to electrolyte, metabolic, and nutritional imbalances in the absence of
other medical problems.
B. Severe nausea and vomiting leading to electrolyte, metabolic, and nutritional
imbalances in the absence of other medical problems.
C. Loss of appetite and continuous vomiting that commonly results in dehydration and ultimately
decreasing maternal nutrients
D. Severe nausea and diarrhea that can cause gastrointestinal irritation and possibly internal
bleeding

The description of hyperemesis gravidarum includes severe nausea and vomiting, leading to
electrolyte, metabolic, and nutritional imbalances in the absence of other medical problems.
Hyperemesis is not a form of anemia. Loss of appetite may occur secondary to the nausea and
vomiting of hyperemesis, which, if it continues, can deplete the nutrients transported to the fetus.
Diarrhea does not occur with hyperemesis
18. Which of the following is the correct health teaching for pregnant mothers taking ferrous
sulfate*

A. "I should drink milk when ferrous sulfate every daily"


B. "I should drink calcium when ferrous sulfate every daily"
C. "I should drink orange juice when ferrous sulfate every daily"
D. "I should not take anything when ferrous sulfate every daily"

Feedback

Ferrous sulfate works best when you take it on an empty stomach. However, if it upsets your
stomach, you can take it with or after food. A doctor (or a pharmacist) may recommend taking
ferrous sulfate with orange juice or a vitamin C supplement. Vitamin C is believed to increase the
amount of iron absorbed by the body.

Milk, calcium and antacids should NOT be taken at the same time as iron supplements. You
should wait at least 2 hours after having these foods before taking your iron supplements because
of the absorption.

19. Which of the following is the common risk of a mother unable to take folic acid at the 1st
trimester?*

A. Risk giving birth with neural tube defects


B. Risk giving birth with cleft palate
C. Risk premature birth
D. Risk giving birth H-mole

20. Which of the following is eating disorder that involves persistent eating of non-nutritive
substances such as hair, dirt, and paint chips for a period of at least one month?*

A. Pseudocyesis
B. Pica
C. Both A&B
D. None of the above

21. Which of the following is no. 1 complication of H-mole? (CAPITALIZED ALL LETTERS)*

CHORIOCARCINOMA
22. Which of the following is the manifestation of Placenta Previa?*

A. Painless, bright red vaginal bleeding


B. Painful, bright red vaginal bleeding
C. Painless, dark red vaginal bleeding
D. Painful, dark red vaginal bleeding

Bright red vaginal bleeding without pain during the second half of pregnancy is the main sign of
placenta previa. Some women also have contractions.

In many women diagnosed with placenta previa early in their pregnancies, the placenta previa
resolves. As the uterus grows, it might increase the distance between the cervix and the placenta.
The more the placenta covers the cervix and the later in the pregnancy that it remains over the
cervix, the less likely it is to resolve.

23. Which of the following is the manifestation of Abruptio of Placenta?

A. Painful, dark red vaginal bleeding

B. Painless, dark red vaginal bleeding

C. Painful, bright red vaginal bleeding

D. Painless, bright red vaginal bleeding

UNIT EXAMINATION PART I:


MIDTERM-MATERNAL
1. A client with eclampsia begins to experience a seizure. Which of the following would the nurse
in charge do first?*
A. Pad the side rails.
B. Place a pillow under the left buttock.
C. Insert a padded tongue blade into the mouth.
D. Maintain a patent airway.
RATIONALE. The priority for the pregnant client having a seizure is to maintain a patent airway to
ensure adequate oxygenation to the mother and the fetus. Additionally, oxygen may be
administered by face mask to prevent fetal hypoxia.

• Option A: Padding the side rails should be done as a precaution before a seizure, not during the
seizure.

• Option B: The client should be placed on a flat, firm surface to avoid any injuries.

• Option C: There should be nothing inserted inside the client’s mouth to maintain airway patency
and prevent obstruction and aspiration.

2. Which of the following is the most common kind of placental adherence seen in pregnant
women?*

A. Accreta
B. Placenta previa
C. Percreta
D. Increta

Correct Answer: A. Accreta

Placenta accreta is the most common kind of placental adherence seen in pregnant women and
is characterized by slight penetration of myometrium.

• Option B: In placenta previa, the placenta does not embed correctly and results in what is
known as a low-lying placenta. It can be marginal, partial, or complete in how it covers the
cervical os, and it increases the patient’s risk for painless vaginal bleeding during the pregnancy
and/or delivery process.

• Option C: Placenta percreta leads to perforation of the uterus and is the most serious and
invasive of all types of accrete.

• Option D: Placenta increta leads to deep penetration of the myometrium.

3. When evaluating a client’s knowledge of symptoms to report during her pregnancy, which
statement would indicate to the nurse in charge that the client understands the information given
to her?*

A. “I’ll report increased frequency of urination.”


B. “If I have blurred or double vision, I should call the clinic immediately.”
C. “If I feel tired after resting, I should report it immediately.”
D. “Nausea should be reported immediately

Correct Answer: B. “If I have blurred or double vision, I should call the clinic immediately.”

Blurred or double vision may indicate hypertension or preeclampsia and should be reported
immediately. It can affect the visual pathways, from the anterior segment to the visual cortex.

• Option A: Urinary frequency is a common problem during pregnancy caused by increased


weight pressure on the bladder from the uterus. The anatomical and physiological changes
affecting the lower urinary tract in pregnancy, as well as the hormonal milieu of pregnancy, have
been postulated to underlie the pathogenesis of lower urinary symptoms in pregnancy

• Option C: Clients generally experience fatigue during pregnancy. Pregnancy is accompanied by


several psychological, emotional, and physical changes that may predispose the woman to
fatigue, which can range from mild tiredness to severe exhaustion.

• Option D: The pathophysiology of nausea and vomiting during early pregnancy is unknown,
although metabolic, endocrine, GI, and psychologic factors probably all play a role. Estrogen may
contribute because estrogen levels are elevated in patients with hyperemesis gravidarum.

4. A client makes a routine visit to the prenatal clinic. Although she is 14 weeks pregnant, the
size of her uterus approximates that in an 18- to 20-week pregnancy. Dr. Charles diagnoses
gestational trophoblastic disease and orders ultrasonography. The nurse expects
ultrasonography to reveal:*

A. An empty gestational sac.


B. Grapelike clusters.
C. A severely malformed fetus.
D. An extrauterine pregnancy

Correct Answer: B. Grapelike clusters.

In a client with gestational trophoblastic disease, an ultrasound performed after the 3rd month
shows grapelike clusters of transparent vesicles rather than a fetus. The vesicles contain a clear
fluid and may involve all or part of the decidual lining of the uterus. Usually, no embryo (and
therefore no fetus) is present because it has been absorbed.
• Option A: An anembryonic pregnancy is characterized by a gestational sac that forms and
grows while an embryo fails to develop. Etiologies include morphological abnormalities of an
embryo that prevents implantation or prevents long term survival of the embryo after
implantation; chromosomal abnormalities that collectively include autosomal trisomy, polyploidy,
sex chromosomal polysomy, and monosomy X likely represent the most common etiologies for
early pregnancy loss; and other genetic and chromosomal abnormalities include translocations,
inversions, single-gene perturbations, and placental mosaicism.

• Option C: Congenital anomalies are also known as birth defects, congenital disorders or
congenital malformations. Congenital anomalies can be defined as structural or functional
anomalies (for example, a severely malformed fetus) that occur during intrauterine life and can
be identified prenatally, at birth, or sometimes may only be detected later in infancy, such as
hearing defects.

• Option D: Because there is no fetus, there can be no extrauterine pregnancy. An extrauterine


pregnancy is seen with an ectopic pregnancy.

5. When administering magnesium sulfate to a client with preeclampsia, the nurse understands
that this drug is given to:*

A. Prevent seizures.
B. Reduce blood pressure.
C. Slow the process of labor.
D. Increase diuresis.

Correct Answer: A. Prevent seizures

The chemical makeup of magnesium is similar to that of calcium and, therefore, magnesium will
act like calcium in the body. As a result, magnesium will block seizure activity in a
hyper-stimulated neurologic system by interfering with signal transmission at the neuromuscular
junction.

• Option B: Magnesium sulfate may attenuate blood pressure by decreasing the vascular
response to pressor substances.

• Option C: Since the primary therapeutic goal of tocolysis is to delay preterm delivery within 48
hours from the initiation of steroid prophylaxis, little evidence suggests that extended MgSO4
therapy is beneficial.

• Option D: There are rare cases of pregnant women who develop polyuria after receiving
intravenous therapy of magnesium sulfate. It can be considered as another cause of solute
diuresis
6. A patient is in her last trimester of pregnancy. Nurse Vickie should instruct her to notify her
primary health care provider immediately if she notices:*

A. Blurred vision
B. Hemorrhoids
C. Increased vaginal mucus
D. Shortness of breath on exertion

Correct Answer: A. Blurred vision

Blurred vision or other visual disturbance, excessive weight gain, edema, and increased blood
pressure may signal severe preeclampsia. This condition may lead to eclampsia, which has
potentially serious consequences for both the patient and fetus.

• Option B: Although hemorrhoids may be a problem during pregnancy, they do not require
immediate attention. Hemorrhoids occur when the external hemorrhoidal veins become varicose
(enlarged and swollen), which causes itching, burning, painful swellings at the anus, dyschezia
(painful bowel movements), and bleeding.

• Option C: Almost all women have more vaginal discharge in pregnancy. This is normal, and
helps prevent any infections travelling up from the vagina to the womb. Towards the end of
pregnancy, the amount of discharge increases further. In the last week or so of pregnancy, it may
contain streaks of sticky, jelly-like pink mucus.

• Option D: Dyspnea can begin before any upward displacement of the diaphragm, suggesting
that factors other than mechanical pressure may be involved. It probably results from the
subjective awareness of hyperventilation that is universally present in pregnancy.
Hyperventilation in pregnancy is predominantly due to an increase in the depth of the tidal
volume, with little change in the respiratory rate

7. A patient with pregnancy-induced hypertension probably exhibits which of the following


symptoms?*

A. Proteinuria, headaches, vaginal bleeding


B. Headaches, double vision, vaginal bleeding
C. Proteinuria, headaches, double vision
D. Proteinuria, double vision, uterine contractions

Correct Answer: C. Proteinuria, headaches, double vision


A patient with pregnancy-induced hypertension complains of a headache, double vision, and
sudden weight gain. A urine specimen reveals proteinuria.

• Option A: Pre-eclampsia increases the risk for placental abruption, a condition in which the
placenta separates from the inner wall of the uterus before delivery. Severe abruption can cause
heavy bleeding, which can be life-threatening for both the baby and the mother.

• Option B: Any hypertensive disorder of pregnancy can result in preeclampsia. It occurs in up to


35% of women with gestational hypertension and up to 25% of those with chronic hypertension.
The underlying pathophysiology that upholds this transition to, or superposition of, preeclampsia
is not well understood; however, it is thought to be related to a mechanism of reduced placental
perfusion inducing the systemic vascular endothelial dysfunction.

• Option D: Symptoms of preeclampsia may include visual disturbances, typically scintillations


and scotomata, presumed to be due to cerebral vasospasm. The woman may describe
new-onset headache that is frontal, throbbing, or similar to a migraine headache, and
gastrointestinal complaints of sudden, new-onset, constant epigastric pain that may be moderate
to severe in intensity and due to hepatic swelling and inflammation, with stretch of the liver
capsule.

8. A patient in her 14th week of pregnancy has presented with abdominal cramping and vaginal
bleeding for the past 8 hours. She has passed several clots. What is the primary nursing
diagnosis for this patient?*

A. Knowledge deficit
B. Fluid volume deficit
C. Anticipatory grieving
D. Pain

Correct Answer: B. Fluid volume deficit

If bleeding and clots are excessive, this patient may become hypovolemic. Pad count should be
instituted. Blood volume expands during pregnancy, and a considerable portion of the weight of a
pregnant woman is retained water.

• Option A: Knowledge deficit is an appropriate nursing diagnosis because the woman might not
have any knowledge on how to manage her symptoms. However, this is not a priority diagnosis.

• Option C: Anticipatory grieving is the name given to the tumultuous set of feelings and reactions
that occur when someone is expecting the death of a loved one.
• Option D: Pain may be felt due to abdominal cramping accompanied by bleeding. This is not a
cause of alarm since true labor pain includes strong and regular contractions and lower back
pain.

9. Which of the following would the nurse most likely expect to find when assessing a pregnant
client with abruption placenta?*

A. Excessive vaginal bleeding


B. Rigid, board-like abdomen
C. Tetanic uterine contractions
D. Premature rupture of membranes

Correct Answer: B. Rigid, board-like abdomen

The most common assessment finding in a client with abruption placenta is a rigid or boardlike
abdomen. Pain, usually reported as a sharp stabbing sensation high in the uterine fundus with
the initial separation, also is common.

• Option A: It’s possible for the blood to become trapped inside the uterus, so even with a severe
placental abruption, there might be no visible bleeding.

• Option C: Uterine contractions are a common finding with placental abruption. Contractions
progress as the abruption expands, and uterine hypertonus may be noted. Contractions are
painful and palpable.

• Option D: Increased frequency of placental abruption was found in patients with early rupture of
membranes. The incidence was 50% and 44% when rupture of the membranes occurred before
20 weeks or between 20-24 weeks of pregnancy, respectively.

10. A pregnant client is diagnosed with partial placenta previa. In explaining the diagnosis, the
nurse tells the client that the usual treatment for partial placenta previa is which of the following?*

A. Activity limited to bed rest.


B. Platelet infusion.
C. Immediate cesarean delivery.
D. Labor induction with oxytocin.

Correct Answer: A. Activity limited to bed rest


Treatment of partial placenta previa includes bed rest, hydration, and careful monitoring of the
client’s bleeding.

• Option B: The greatest risk of placenta previa is hemorrhage. Bleeding often occurs as the
lower part of the uterus thins during the third trimester of pregnancy in preparation for labor. This
may require blood transfusion during Cesarean section.

• Option C: In general, there is a higher Cesarean rate associated with placental edge-to-cervical
os distances of less than 2 cm.

• Option D: Labor induction is the stimulation of uterine contractions during pregnancy before
labor begins on its own to achieve a vaginal birth. It is not an option for placenta previa.

11. When developing a plan of care for a client newly diagnosed with gestational diabetes, which
of the following instructions would be the priority?*

A. Dietary intake
B. Medication
C. Exercise
D. Glucose monitoring

Correct Answer: A. Dietary intake

Although all of the choices are important in the management of diabetes, diet therapy is the
mainstay of the treatment plan and should always be the priority. The goal of dietary therapy is to
avoid single large meals and foods with a large percentage of simple carbohydrates.

• Option B: Women diagnosed with gestational diabetes generally need only diet therapy without
medication to control their blood sugar levels. A total of 6 feedings per day is preferred, with 3
major meals and 3 snacks to limit the amount of energy intake presented to the bloodstream at
any interval.

• Option C: Exercise, is important for all pregnant women and especially for diabetic women,
because it burns up glucose, thus decreasing blood sugar. However, dietary intake, not exercise,
is the priority. The diet should include foods with complex carbohydrates and cellulose, such as
whole-grain bread and legumes.

• Option D: All pregnant women with diabetes should have periodic monitoring of serum glucose.
However, those with gestational diabetes generally do not need daily glucose monitoring. The
standard of care recommends a fasting and 2-hour postprandial blood sugar level every 2 weeks.
12. A client with severe preeclampsia is admitted with BP 160/110, proteinuria, and severe pitting
edema. Which of the following would be most important to include in the client’s plan of care?*

A. Daily weights
B. Seizure precautions
C. Right lateral positioning
D. Stress reduction

Correct Answer: B. Seizure precautions

Women hospitalized with severe preeclampsia need decreased CNS stimulation to prevent a
seizure. Seizure precautions provide environmental safety should a seizure occur.

• Option A: Because of edema, daily weight is important but not the priority. High pregnancy
weight gain was more strongly associated with term preeclampsia than early preterm
preeclampsia (eg, 64% versus 43% increased odds per 1 z score difference in weight gain in
normal-weight women, and 30% versus 0% in obese women, respectively).

• Option C: Preeclampsia causes vasospasm and therefore can reduce uteroplacental perfusion.
The client should be placed on her left side to maximize blood flow, reduce blood pressure, and
promote diuresis.

• Option D: Interventions to reduce stress and anxiety are very important to facilitate coping and
a sense of control, but seizure precautions are the priority.

13. The preferred manner of delivering the baby in a gravido-cardiac is vaginal delivery assisted
by forceps under epidural anesthesia. The main rationale for this is:*

A. To allow atraumatic delivery of the baby.


B. To allow a gradual shifting of the blood into the maternal circulation.
C. To make the delivery effort-free and the mother does not need to push with
contractions.
D. To prevent perineal laceration with the expulsion of the fetal head.

Correct Answer: C. To make the delivery effort-free and the mother does not need to push with
contractions.

Forceps delivery under epidural anesthesia will make the delivery process less painful and
require less effort to push for the mother. Pushing requires more effort which a compromised
heart may not be able to endure.
• Option A: The benefit of avoiding hemodynamic fluctuations resulting from pushing must be
weighed against an increased risk of perineal trauma, hemorrhage, and fetal head injury with
forceps or vacuum delivery. For most women, pushing may be preferred above primary assisted
delivery.

• Option B: Several studies show that cesarean section is performed more often in women with
heart disease than in a healthy population. For most cardiac patients, however, vaginal delivery is
preferred and cesarean section is reserved for obstetric indications since cesarean section is
associated with more blood loss and higher thromboembolic and infection risk.

• Option D: Using a technique called super crowning, avoiding an episiotomy, and reaching for a
vacuum device rather than forceps during operative vaginal deliveries are among the strategies
that can help reduce the number of third- and fourth-degree lacerations.

14. Which of the following are the most commonly assessed findings in cystitis?*

A. Frequency, urgency, dehydration, nausea, chills, and flank pain


B. Nocturia, frequency, urgency dysuria, hematuria, fever, and suprapubic pain
C. Dehydration, hypertension, dysuria, suprapubic pain, chills, and fever
D. High fever, chills, flank pain nausea, vomiting, dysuria, and frequency

Correct Answer: B. Manifestations of cystitis include, frequency, urgency, dysuria, hematuria


nocturia, fever, and suprapubic pain.

Dehydration, hypertension, and chills are not typically associated with cystitis. High fever chills,
flank pain, nausea, vomiting, dysuria, and frequency are associated with pyelonephritis.

• Option A: Cystitis usually develops due to the colonization of the periurethral mucosa by
bacteria from the fecal or vaginal flora and ascension of such pathogens to the urinary bladder.
Uropathogens may have microbial virulence factors that allow them to escape host defenses and
invade host tissues in the urinary tract.

• Option C: Acute cystitis often presents with urinary symptoms which include dysuria, urinary
frequency urgency, suprapubic pain or tenderness, and occasionally hematuria. Based on a
systematic review examining history and examination findings of women with uncomplicated UTI,
the combination of dysuria and urinary frequency in the absence of vaginal discharge or irritation
is highly predictive of uncomplicated cystitis.

• Option D: Cystitis may be differentiated from pyelonephritis by the absence of systemic findings
such as fever, chills, or sepsis. Findings such as flank pain, costovertebral angle tenderness,
nausea, and vomiting are also more indicative of upper UTI or pyelonephritis.
15. Which of the following is TRUE in Rh incompatibility?*

A. The condition can occur if the mother is Rh(+) and the fetus is Rh(-).
B. Every pregnancy of an Rh(-) mother will result in erythroblastosis fetalis.
C. On the first pregnancy of the Rh(-) mother, the fetus will not be affected.
D. RhoGam is given only during the first pregnancy to prevent incompatibility.

Correct Answer: C. On the first pregnancy of the Rh(-) mother, the fetus will not be affected

On the first pregnancy, the mother still has no contact with Rh(+) blood thus it has not antibodies
against Rh(+). After the first pregnancy, even if terminated into an abortion, there is already the
possibility of mixing of maternal and fetal blood so this can trigger the maternal blood to produce
antibodies against Rh(+) blood. The fetus takes its blood type usually from the father.

• Option A: The most common cause of Rh incompatibility is exposure from an Rh-negative


mother by Rh-positive fetal blood during pregnancy or delivery. As a consequence, blood from
the fetal circulation may leak into the maternal circulation, and, after a significant exposure,
sensitization occurs leading to maternal antibody production against the foreign Rh antigen.

• Option B: In women who are prone to Rh incompatibility, the second pregnancy with an
Rh-positive fetus often produces a mildly anemic infant, whereas succeeding pregnancies
produce more seriously affected infants who ultimately may die in utero from massive
antibody-induced hemolytic anemia.

• Option D: The exact mechanism by which passive administration of Rh IgG prevents Rh


immunization is unknown. The most likely hypothesis is that the Rh immune globulin coats the
surface of fetal RBCs containing Rh antigens. These exogenous antibody-antigen complexes
cross the placenta before they can stimulate the maternal endogenous immune system B cells to
produce IgG antibodies.

16. Before giving a repeat dose of magnesium sulfate to a pre-eclamptic patient, the nurse
should assess the patient’s condition. Which of the following conditions will require the nurse to
temporarily suspend a repeat dose of magnesium sulfate?*

A. 100 cc. urine output in 4 hours


B. Knee jerk reflex is (+)2
C. Serum magnesium level is 10mEg/L.
D. Respiratory rate of 16/min

Correct Answer: A. 100 cc. urine output in 4 hours


The minimum urine output expected for a repeat dose of MgSO4 is 30 cc/hr. If in 4 hours the
urine output is only 100 cc this is low and can lead to poor excretion of Magnesium with a
possible cumulative effect, which can be dangerous to the mother.

• Option B: As the plasma levels increase the muscle weakness becomes more pronounced and
there is a marked reduction and then loss of deep tendon reflexes eventually leading to flaccid
paralysis and respiratory arrest.

• Option C: Magnesium sulfate is the ideal drug for the prevention and treatment of eclampsia,
and, indeed, its universal use is recommended by the World Health Organization. Nevertheless,
the best regimen remains to be established and there is still no evidence that serum magnesium
levels between 4 and 7?mEq/L, established in a retrospective study and still considered
therapeutic, represent a guarantee that pregnant women with hypertensive disorders are
protected against eclampsia.

• Option D: Magnesium sulfate has CNS and respiratory depressant effects. It acts peripherally,
causing vasodilation; moderate doses cause flushing and sweating, whereas high doses cause
hypotension. It prevents or controls seizures by blocking neuromuscular transmission.

17. In which of the following conditions can the causative agent pass through the placenta and
affect the fetus in utero?*

A. Gonorrhea
B. Rubella
C. Candidiasis
D. Moniliasis

Correct Answer: B. Rubella

Rubella is caused by a virus and viruses have low molecular weight thus can pass through the
placental barrier. Relatively few pathogens are capable of placental and fetal infections in
humans and even for these, maternal infection does not guarantee placental or fetal infection.

• Option A: Other STIs, like gonorrhea, chlamydia, hepatitis B, and genital herpes, can pass from
the mother to the baby as the baby passes through the birth canal. This infection in an infant can
cause eye infections, pneumonia, or infections of the joints or blood. Treating gonorrhea as soon
as it is detected in pregnant women will reduce the risk of transmission.

• Option C: Candida infection of the fetus results in prematurity and death, infection can occur
across intact membranes, and systemic candidiasis in the fetus is likely to be associated with an
intrauterine device. The pathology of the placenta includes microscopic granulomata and
presence of filaments or spores on the cord and histological change of the membrane or
chorionic plate revealing intense chorioamnionitis with occasional focal granuloma.
• Option D: Bacterial or viral infection of the mother during the course of pregnancy can cross the
placenta and actively infect the fetus. However, especially for bacteria, it is more common for
mothers to experience an infection that can be treated without overt fetal infection.

18. In placenta previa marginalis, the placenta is found at the:*

A. Internal cervical os partly covering the opening.


B. External cervical os slightly covering the opening.
C. Lower segment of the uterus with the edges near the internal cervical os.
D. Lower portion of the uterus completely covering the cervix.

Correct Answer: C. Lower segment of the uterus with the edges near the internal cervical os

Placenta marginalis is a type of placenta previa wherein the placenta is implanted at the lower
segment of the uterus thus the edges of the placenta are touching the internal cervical
opening/os. The normal site of placental implantation is the upper portion of the uterus.

• Option A: Marginal placenta previa is where the placental edge is within 2cm of the internal os.
Nearly 90% of placentas identified as “low lying” will ultimately resolve by the third trimester due
to placental migration. The placenta itself does not move but grows toward the increased blood
supply at the fundus, leaving the distal portion of the placenta at the lower uterine segment with
relatively poor blood supply to regress and atrophy.

• Option B: The trophoblast adheres to the decidua basalis of the endometrium, forming a normal
pregnancy. Prior uterine scars provide an environment that is rich in oxygen and collagen. The
trophoblast can adhere to the uterine scar leading to the placenta covering the cervical os or the
placenta invading the walls of the myometrium.

• Option D: Migration can also take place by the growing lower uterine segment thus increasing
the distance from the lower margin of the placenta to the cervix.

19. The nursing measure to relieve fetal distress due to maternal supine hypotension is:*

A. Place the mother in semi-Fowler's position.


B. Put the mother on the left side-lying position.
C. Place mother on a knee-chest position.
D. Any of the above.

Correct Answer: B. Put the mother on left side-lying position.


When a pregnant woman lies in a supine position, the weight of the gravid uterus would be
compressing on the vena cava against the vertebrae obstructing blood flow from the lower
extremities. This causes a decrease in blood return to the heart and consequently immediate
decreased cardiac output and hypotension. Hence, putting the mother on side-lying will relieve
the pressure exerted by the gravid uterus on the vena cava.

• Option A: Placing the pregnant woman in a Semi-Fowler’s position would still place the weight
of the gravid uterus on the vena cava and obstruct the blood flow from the lower extremities.

• Option C: A pregnant woman would be incapable to perform the knee-chest position due to her
gravid belly. The primary treatment used for non-reassuring fetal status is intrauterine
resuscitation. This will help prevent any unnecessary procedures.

20. Which of the following signs will distinguish threatened abortion from imminent abortion?*

A. Severity of bleeding.
B. Dilation of the cervix.
C. Nature and location of pain.
D. Presence of uterine contraction.

Correct Answer: B. Dilation of the cervix

In imminent abortion, the pregnancy will definitely be terminated because the cervix is already
open unlike in threatened abortion where the cervix is still closed.

• Option A: Nearly 25% of pregnant women have some degree of vaginal bleeding during the first
two trimesters and about 50% of these progress to loss of the pregnancy. The bleeding during a
threatened abortion is typically mild to moderate.

• Option C: A threatened abortion occurs when a pregnant patient at less than 20 weeks
gestation presents with vaginal bleeding. The cervical os is closed on a physical exam. The
patient may also experience abdominal cramping, pelvic pain, pelvic pressure, and/or back pain.

• Option D: A pelvic exam is mandatory to determine the type of abortion. Determining factors
include the amount and site of bleeding, whether the cervix is dilated, and whether fetal tissue
has passed. In a threatened abortion, the vaginal exam may reveal a closed cervical os with no
tissue. There is usually no cervical motion tenderness.

21. A gravidocardiac mother is advised to observe bed rest primarily to:*

A. Allow the fetus to achieve normal intrauterine growth.


B. Minimize oxygen consumption which can aggravate the condition of the
compromised heart of the mother.
C. Prevent perinatal infection.
D. Reduce incidence of premature labor.

Feedback

Correct Answer: B. Minimize oxygen consumption which can aggravate the condition of the
compromised heart of the mother.

The activity of the mother will require more oxygen consumption. Since the heart of a
gravido-cardiac is compromised, there is a need to put a mother on bedrest to reduce the need
for oxygen.

• Option A: In cases of maternal decompensation, fetal monitoring should also be done to ensure
fetal well-being. Women with moderate-risk or high-risk lesions, especially cyanotic lesions, have
an increased risk of fetal growth restriction and should be followed with monthly ultrasound
examinations for fetal growth.

• Option C: The 2011 update to the American Heart Association guideline for the prevention of
cardiovascular disease (CVD) in women recommends that risk assessment at any stage of life
include a detailed history of pregnancy complications. Gestational diabetes, preeclampsia,
preterm birth, and birth of an infant small for gestational age are ranked as major risk factors for
CVD.

• Option D: During the third trimester, cardiac output is further influenced by body position, where
the supine position causes caval compression by the gravid uterus. This leads to a decrease in
venous return, which can cause supine hypotension of pregnancy. Stroke volume normally
increases in the first and second trimester and decreases in the third trimester. This decrease is
due to partial vena cava obstruction.

22. When a pregnant woman goes into a convulsive seizure, the MOST immediate action of the
nurse to ensure the safety of the patient is:*

A. Apply restraint so that the patient will not fall out of bed.
B. Put a mouth gag so that the patient will not bite her tongue and the tongue will not fall back.
C. Position the mother on her side to allow the secretions to drain from her mouth and
prevent aspiration.
D. Check if the woman is also having precipitate labor.
Correct Answer: C. Position the mother on her side to allow the secretions to drain from her
mouth and prevent aspiration.

Positioning the mother on her side will allow the secretions that may accumulate in her mouth to
drain by gravity thus preventing aspiration pneumonia.

• Option A: Placing a patient who is in seizure in restraints would further injure him or her. Place
the patient on a flat, firm surface during seizure.

• Option B: Putting a mouth gag is not safe since during the convulsive seizure the jaw will
immediately lock.

• Option D: The mother may go into labor also during the seizure, but the immediate concern of
the nurse is the safety of the baby. After the seizure, check the perineum for signs of precipitate
labor.

23. When a pregnant woman goes into a convulsive seizure, the MOST immediate action of the
nurse to ensure the safety of the patient is:*

A. Apply restraint so that the patient will not fall out of bed.
B. Put a mouth gag so that the patient will not bite her tongue and the tongue will not fall back.
C. Position the mother on her side to allow the secretions to drain from her mouth and
prevent aspiration.
D. Check if the woman is also having precipitate labor.

Correct Answer: C. Position the mother on her side to allow the secretions to drain from her
mouth and prevent aspiration.

Positioning the mother on her side will allow the secretions that may accumulate in her mouth to
drain by gravity thus preventing aspiration pneumonia.

• Option A: Placing a patient who is in seizure in restraints would further injure him or her. Place
the patient on a flat, firm surface during seizure.

• Option B: Putting a mouth gag is not safe since during the convulsive seizure the jaw will
immediately lock.

• Option D: The mother may go into labor also during the seizure, but the immediate concern of
the nurse is the safety of the baby. After the seizure, check the perineum for signs of precipitate
labor.
24. Which of the following signs and symptoms will most likely make the nurse suspect that the
patient has hydatidiform mole?*

A. Slight bleeding
B. Passage of clear vesicular mass per vagina
C. Absence of fetal heartbeat
D. Enlargement of the uterus

Correct Answer: B. Passage of clear vesicular mass per vagina

Hydatidiform mole (H-mole) is characterized by the degeneration of the chorionic villi wherein the
villi becomes vesicle-like. These vesicle-like substances when expelled per vagina and is a
definite sign that the woman has H-mole.

• Option A: Implantation bleeding is a common cause of spotting early on in pregnancy.


Implantation bleeding happens when the fertilized egg attaches to the uterine lining. This can
trigger a few days of light bleeding or spotting.

• Option C: If the crown-rump length (CRL) is > 7 mm and there is no embryonic cardiac activity,
this is defined as a missed miscarriage, or. If the mean gestational sac diameter is > 25 mm and
there is no yolk sac or embryonic pole, this is defined as an empty sac miscarriage.

• Option D: Two of the most common causes of an enlarged uterus are uterine fibroids and
adenomyosis. Uterine fibroids are commonly noncancerous tumors of the muscular wall of the
uterus, affecting as many as eight in 10 women by the age of 50. Fibroids more commonly affect
women over age 30.

25. Which of the following is the most likely effect on the fetus if the woman is severely anemic
during pregnancy?*

A. Large for gestational age (LGA) fetus


B. Hemorrhage
C. Small for gestational age (SGA) baby
D. Erythroblastosis fetalis

Correct Answer: C. Small for gestational age (SGA) baby

Anemia is a condition where there is a reduced amount of hemoglobin. Hemoglobin is needed to


supply the fetus with adequate oxygen. Oxygen is needed for normal growth and development of
the fetus.
• Option A: Women who gain a lot of weight during pregnancy often give birth to babies who are
large for gestational age. Diabetes in the mother is the most common cause of babies who are
large for gestational age. When a pregnant woman has high blood sugar, she can pass that
along to her baby.

• Option B: Subchorionic bleeding affects the chorionic membranes. These lift apart and form
another sac between the placenta and the uterus. The movement and resulting clots are what
cause this type of bleeding. These hematomas can range in size, with the smallest being most
common.

• Option D: Erythroblastosis fetalis is hemolytic anemia in the fetus (or neonate, as


erythroblastosis neonatorum) caused by transplacental transmission of maternal antibodies to
fetal red blood cells. The disorder usually results from an incompatibility between maternal and
fetal blood groups, often Rho(D) antigens.

26. Smoking is contraindicated in pregnancy because:*

A. Nicotine causes vasodilation of the mother’s blood vessels.


B. Carbon monoxide binds with the hemoglobin of the mother reducing available
hemoglobin for the fetus.
C. The smoke will make the fetus, and the mother feels dizzy.
D. Nicotine will cause vasoconstriction of the fetal blood vessels.

Correct Answer: B. Carbon monoxide binds with the hemoglobin of the mother reducing available
hemoglobin for the fetus.

Carbon monoxide is one of the substances found in cigarette smoke. This substance diminishes
the ability of the hemoglobin to bind with oxygen thus reducing the amount of oxygenated blood
reaching the fetus.

• Option A: There is blood flow restriction to the placenta due to the vasoconstrictive effects of
catecholamines released from the adrenals and nerve cells after nicotine activation.

• Option C: Nicotine is rapidly absorbed when the tobacco smoke reaches the small airways and
alveoli of the lung. This causes a quick rise in blood nicotine concentrations, but due to the
eventual burnout of the cigarette, these levels also peak early and thereafter drop to lower levels.

• Option D: Direct effects on nicotinic acetylcholine receptors (nAChRs), which are present and
functional very early in the fetal brain [5] are also likely to contribute.

27. The main reason for an expected increased need for iron in pregnancy is:*
A. The mother may have physiologic anemia due to the increased need for red blood
cell mass as well as the fetal requires about 350-400 mg of iron to grow.
B. The mother may suffer anemia because of poor appetite.
C. The fetus has an increased need for RBC which the mother must supply.
D. The mother may have a problem with digestion because of pica.

Correct Answer: A. The mother may have physiologic anemia due to the increased need for red
blood cell mass, as well as the fetal, requires about 350-400 mg of iron to grow.

About 400 mg of iron is needed by the mother in order to produce more RBC mass to be able to
provide the needed increase in blood supply for the fetus. Also, about 350-400 mg of iron is
needed for the normal growth of the fetus. Thus, about 750-800 mg iron supplementation is
needed by the mother to meet this additional requirement.

• Option B: If the woman loses her appetite, she may experience a general disinterest in all foods
or a lack of desire to eat. Keep in mind that appetite loss differs from an aversion to a few specific
foods, which is also fairly common during pregnancy. Pregnant women with chronically poor
appetites run a risk of anemia, fetal growth abnormalities, and preterm birth.

• Option C: Anemia during pregnancy is especially a concern because it is associated with low
birth weight, premature birth, and maternal mortality. Women who are pregnant are at a higher
risk for developing anemia due to the excess amount of blood the body produces to help provide
nutrients for the baby.

• Option D: Pica is the practice of craving substances with little or no nutritional value. Most
pregnancy and pica-related cravings involve non-food substances such as dirt or chalk. Eating
non-food substances is potentially harmful to both the mother and the baby. Eating non-food
substances may interfere with the nutrient absorption of healthy food substances and actually
cause a deficiency.

28. Which of the following signs will require a mother to seek immediate medical attention?*

A. When the first fetal movement is felt.


B. No fetal movement is felt on the 6th month.
C. Mild uterine contraction.
D. Slight dyspnea on the last month of gestation.

Correct Answer: B. No fetal movement is felt on the 6th month.


Fetal movement is usually felt by the mother during 4.5 – 5 months. If the pregnancy is already in
its 6th month and no fetal movement is felt, the pregnancy is not normal either the fetus is
already dead intra-uterine or it is an H-mole.

• Option A: The first fetal movements which are felt by the mother are called quickening. One
function of these movements is to alert the pregnant woman that she has a fetus growing in her
uterus. Quickening often occurs between the 16th to the 22nd week of pregnancy. This is called
a presumptive sign of pregnancy as the other movements of the woman’s body can mimic early
fetal movements such as flatus, peristalsis, and abdominal muscle contractions.

• Option C: Sometime during the second or third trimester, the woman might start to feel mild,
sporadic contractions, especially when she is tired or dehydrated, or after sex. These normal
contractions, called Braxton Hicks contractions or false labor, are the body’s way of rehearsing
for birth.

• Option D: In the first few weeks of pregnancy, a normal increase in the hormone progesterone
causes the woman to breathe more often. This can look and feel like shortness of breath. This
hormone expands the lung capacity, allowing blood to carry large quantities of oxygen to the
baby.

29. The neonate of a mother with diabetes mellitus is prone to developing hypoglycemia
because:*

A. The pancreas is immature and unable to secrete the needed insulin.


B. There is rapid diminution of glucose level in the baby’s circulating blood and his
pancreas is normally secreting insulin.
C. The baby is reacting to the insulin given to the mother.
D. His kidneys are immature leading to a high tolerance for glucose.

Correct Answer: B. There is rapid diminution of glucose level in the baby’s circulating blood and
his pancreas is normally secreting insulin.

If the mother is diabetic, the fetus while in utero has a high supply of glucose. When the baby is
born and is now separate from the mother, it no longer receives a high dose of glucose from the
mother. In the first few hours after delivery, the neonate usually does not feed yet thus this can
lead to hypoglycemia.

• Option A: The primary function of ?-cells is to store and secrete insulin in response to glucose
load. When ?-cells lose the ability to adequately sense blood glucose concentration, or to release
sufficient insulin in response, this is classified as ?-cell dysfunction. ?-cell dysfunction is thought
to be the result of prolonged, excessive insulin production in response to chronic fuel excess
• Option C: ?-cell dysfunction is exacerbated by insulin resistance. Reduced insulin-stimulated
glucose uptake further contributes to hyperglycemia, overburdening the ?-cells, which have to
produce additional insulin in response. The direct contribution of glucose to ?-cell failure is
described as glucotoxicity. Thus, once ?-cell dysfunction begins, a vicious cycle of
hyperglycemia, insulin resistance, and further ?-cell dysfunction is set in motion.

• Option D: Insulin resistance occurs when cells no longer adequately respond to insulin. At the
molecular level, insulin resistance is usually a failure of insulin signaling, resulting in inadequate
plasma membrane translocation of glucose transporter 4 (GLUT4)—the primary transporter that
is responsible for bringing glucose into the cell to use as energy.

30. A prenatal nurse is providing instructions to a group of pregnant clients regarding measures
to prevent toxoplasmosis. Which statement, if made by one of the clients indicates a need for
further instructions?*

A. “I need to cook meat thoroughly.”


B. “I need to avoid touching mucous membranes of the mouth or eyes while handling raw
meat.”
C. “I need to drink unpasteurized milk only.”
D. “I need to avoid contact with materials that are possibly contaminated with cat feces.”

Correct Answer: C. “I need to drink unpasteurized milk only.”

All pregnant women should be advised to do the following to prevent the development of
toxoplasmosis. Everyone, including immunocompetent patients, should be educated about
toxoplasmosis risk factors and ways to minimize the risks. Preventing toxoplasmosis is
particularly important in seronegative immunocompromised patients and in pregnant women.

• Option A: Avoid eating raw meat, unpasteurized milk, and uncooked eggs, oysters, clams, and
mussels. Rarely, infection by tachyzoites occurs from ingestion of unpasteurized milk or by direct
entry into the bloodstream through a blood transfusion or laboratory accident. Transmission can
also occur via ingestion of tissue cysts (bradyzoites) in undercooked or uncooked meat or
through transplantation of an organ that contains tissue cysts. (Slaughterhouse workers and
butchers may be at increased risk of infection.) In Europe and the United States, pork is the
major source of T gondii infection in humans.

• Option B: Women should be instructed to cook meats thoroughly, avoid touching mucous
membranes and eyes while handling raw meat; thoroughly wash all kitchen surfaces that come
into contact with uncooked meat, wash the hands thoroughly after handling raw meat; avoid
uncooked eggs and unpasteurized milk; wash fruits and vegetables before consumption.
• Option D: Avoid contact with materials that possibly are contaminated with cat feces, such as
cat litter boxes, sandboxes, and garden soil. T gondii oocysts are ingested in material
contaminated by feces from infected cats. Oocysts may also be transported to food by flies and
cockroaches. When T gondii is ingested, bradyzoites are released from cysts or sporozoites are
released from oocysts, and the organisms enter gastrointestinal cells. Host cell receptors
consisting of laminin, lectin, and SAG1 are involved in T gondii tachyzoite attachment and
penetration. Tachyzoites multiply, rupture cells, and infect contiguous cells. They are transported
via the lymphatics and are disseminated hematogenously throughout the tissues.

31. A client in the first trimester of pregnancy arrives at a health care clinic and reports that she
has been experiencing vaginal bleeding. A threatened abortion is suspected, and the nurse
instructs the client regarding management of care. Which statement, if made by the client,
indicates a need for further education?*

A. “I will maintain strict bedrest throughout the remainder of the pregnancy.”


B. “I will avoid sexual intercourse until the bleeding has stopped, and for 2 weeks following the
last evidence of bleeding.”
C. “I will count the number of perineal pads used on a daily basis and note the amount and
color of blood on the pad.”
D. “I will watch for the evidence of the passage of tissue.”

Correct Answer: A. “I will maintain strict bedrest throughout the remainder of the pregnancy.”

Strict bed rest throughout the remainder of pregnancy is not required. Bedrest and other activity
restrictions have not been found to be efficacious in the prevention of a threatened abortion
progressing to spontaneous abortion and have been shown to increase the risk of other
complications including deep vein thrombosis and/or pulmonary embolism and therefore should
not be recommended

• Option B: The woman is advised to curtail sexual activities until the bleeding has ceased, and
for 2 weeks following the last evidence of bleeding or as recommended by the physician.

• Option C: The woman is instructed to count the number of perineal pads used daily and to note
the quantity and color of blood on the pad. Patients with a threatened abortion should be
managed expectantly without any medical or surgical interventions. However, patients should be
given strict return precautions concerning excessive vaginal bleeding, abdominal pain, or fever
and patients should be educated on the importance of follow-up.

• Option D: The woman also should watch for the evidence of the passage of tissue. A
threatened abortion is defined as vaginal bleeding before 20 weeks gestational age in the setting
of positive urine and/or blood pregnancy test with a closed cervical os, without passage of
products of conception, and without evidence of fetal or embryonic demise.
32. A homecare nurse visits a pregnant client who has a diagnosis of mild Preeclampsia and who
is being monitored for pregnancy induced hypertension (PIH). Which assessment finding
indicates a worsening of the preeclampsia and the need to notify the physician?*

A. Blood pressure reading is at the prenatal baseline.


B. Urinary output has increased.
C. The client complains of a headache and blurred vision.
D. Dependent edema has resolved.

Correct Answer: C. The client complains of a headache and blurred vision.

If the client complains of a headache and blurred vision, the physician should be notified because
these are signs of worsening preeclampsia.

• Option A: In normal pregnancy, women’s mean arterial pressure drops 10-15 mm Hg over the
first half of pregnancy. Most women with mild chronic hypertension (ie, SBP 140-160 mm Hg,
DBP 90-100 mm Hg) have a similar decrease in blood pressures and may not require any
medication during this period.

• Option B: In addition to rising hormones, the body’s fluid levels start to increase during
pregnancy. This means the kidneys have to work extra hard to flush the extra fluid. The amount
of urine released will increase as well. In the third trimester, the baby’s growing size means
they’re pressing even more on the bladder.

• Option D: During normal pregnancy total body water increases by 6 to 8 liters, 4 to 6 liters of
which are extracellular, of which at least 2 to 3 liters are interstitial. At some stage in pregnancy 8
out of 10 women have demonstrable clinical edema.

33. A primigravida is receiving magnesium sulfate for the treatment of pregnancy induced
hypertension (PIH). The nurse who is caring for the client is performing assessments every 30
minutes. Which assessment finding would be of most concern to the nurse?*

A. Urinary output of 20 ml since the previous assessment


B. Deep tendon reflexes of 2+
C. Respiratory rate of 10 BPM
D. Fetal heart rate of 120 BPM

Correct Answer: C. Respiratory rate of 10 BPM.


Magnesium sulfate depresses the respiratory rate. If the respiratory rate is less than 12 breaths
per minute, the physician or other health care provider needs to be notified, and continuation of
the medication needs to be reassessed.

• Option A: A urinary output of 20 ml in a 30 minute period is adequate; less than 30 ml in one


hour needs to be reported. The kidneys face remarkable demands during pregnancy, and it is
critical that the practicing nephrologist understands the normal kidney adaptations to pregnancy.
GFR rises early to a peak of 40% to 50% that of prepregnancy levels, resulting in lower levels of
serum creatinine, urea, and uric acid. There is a net gain of sodium and potassium, but a greater
retention of water, with gains of up to 1.6 L.

• Option B: Deep tendon reflexes of 2+ are normal. With preeclampsia, a woman’s reflexes
become unusually active. Increasing blood pressure will lead to increasing hyperreflexia until
uncontrollable seizures eventually result. Testing for this change is difficult in the field setting; in a
clinic setting an overactive patellar response is a good indicator.

• Option D: The fetal heart rate is WNL for a resting fetus. Current international guidelines
recommend for the normal fetal heart rate (FHR) baseline different ranges of 110 to 150 beats
per minute (bpm) or 110 to 160 bpm.

34. A nurse is caring for a pregnant client with preeclampsia. The nurse prepares a plan of care
for the client and documents in the plan that if the client progresses from preeclampsia to
eclampsia, the nurse’s first action is to:*

A. Administer magnesium sulfate intravenously


B. Assess the blood pressure and fetal heart rate.
C. Clean and maintain an open airway.
D. Administer oxygen by face mask.

Correct Answer: C. Clean and maintain an open airway.

The immediate care during a seizure (eclampsia) is to ensure a patent airway. The other options
are actions that follow or will be implemented after the seizure has ceased.

• Option A: In this case, the doctor may prescribe magnesium sulfate as well as medications to
help reduce blood pressure. Magnesium sulfate therapy is used to prevent seizures in women
with preeclampsia. It can also help prolong a pregnancy for up to two days.

• Option B: Preeclampsia is when the blood pressure, or the force of blood against the walls of
the arteries, becomes high enough to damage the arteries and other blood vessels. Damage to
the arteries may restrict blood flow. It can produce swelling in the blood vessels in the brain and
to the growing baby. If this abnormal blood flow through vessels interferes with the brain’s ability
to function, seizures may occur.

• Option D: The initial treatment for eclampsia includes maintaining oxygen delivery to both
mother and fetus, minimizing the risk of aspiration, treating the seizure, and controlling
hypertension.

35. A nurse is monitoring a pregnant client with pregnancy induced hypertension who is at risk for
preeclampsia. The nurse checks the client for which specific signs of preeclampsia? Select all
that apply.*

A. Elevated blood pressure


B. Negative urinary protein
C. Facial edema
D. Increased respirations
E. Polydipsia

Correct Answer: A & C. Elevated blood pressure and facial edema.

The three classic signs of preeclampsia are hypertension, generalized edema, and proteinuria.
Increased respirations are not a sign of preeclampsia.

• Option A: Preeclampsia is defined as the presence of (1) a systolic blood pressure (SBP)
greater than or equal to 140 mm Hg or a diastolic blood pressure (DBP) greater than or equal to
90 mm Hg or higher, on two occasions at least 4 hours apart in a previously normotensive
patient, OR (2) an SBP greater than or equal to 160 mm Hg or a DBP greater than or equal to
110 mm Hg or higher.

• Option B: In addition to the blood pressure criteria, proteinuria of greater than or equal to 0.3
grams in a 24-hour urine specimen, a protein (mg/dL)/creatinine (mg/dL) ratio of 0.3 or higher, or
a urine dipstick protein of 1+ (if a quantitative measurement is unavailable) is required to
diagnose preeclampsia.

• Option C: Edema exists in many pregnant women, but a sudden increase in edema or facial
edema is suggestive of preeclampsia. The edema of preeclampsia occurs by a distinct
mechanism that is similar to that of angioneurotic edema.

• Option D: Shortness of breath, a racing pulse, mental confusion, a heightened sense of anxiety,
and a sense of impending doom can be symptoms of preeclampsia. If these symptoms are new
to you, they could indicate an elevated blood pressure, or more rarely, fluid collecting in your
lungs (pulmonary edema).
• Option E: Primary polydipsia (PP) is a condition where there is excess consumption of fluids
leading to polyuria with diluted urine and, ultimately, hyponatremia.

36. Rho (D) immune globulin (RhoGAM) is prescribed for a woman following delivery of a
newborn infant and the nurse provides information to the woman about the purpose of the
medication. The nurse determines that the woman understands the purpose of the medication if
the woman states that it will protect her next baby from which of the following?*

A. Being affected by Rh incompatibility.


B. Having Rh-positive blood.
C. Developing a rubella infection.
D. Developing physiological jaundice.

Correct Answer: A. Being affected by Rh incompatibility.

Rh incompatibility can occur when an Rh-negative mom becomes sensitized to the Rh antigen.
Sensitization may develop when an Rh-negative woman becomes pregnant with a fetus who is
Rh-positive. Administration of Rho(D) immune globulin prevents the woman from developing
antibodies against Rh-positive blood by providing passive antibody protection against the Rh
antigen.

• Option B: During pregnancy and at delivery, some of the baby’s Rh-positive blood can enter the
maternal circulation, causing the woman’s immune system to form antibodies against Rh-positive
blood.

• Option C: Rubella can be prevented with MMR vaccine. This protects against three diseases:
measles, mumps, and rubella. CDC recommends children get two doses of MMR vaccine,
starting with the first dose at 12 through 15 months of age, and the second dose at 4 through 6
years of age. Teens and adults also should also be up to date on their MMR vaccination.

• Option D: The best preventive of infant jaundice is adequate feeding. Breast-fed infants should
have eight to 12 feedings a day for the first several days of life. Formula-fed infants usually
should have 1 to 2 ounces (about 30 to 60 milliliters) of formula every two to three hours for the
first week.

37. A pregnant client is receiving magnesium sulfate for the management of preeclampsia. A
nurse determines the client is experiencing toxicity from the medication if which of the following is
noted on assessment?*

A. Presence of deep tendon reflexes.


B. Serum magnesium level of 6 mEq/L.
C. Proteinuria of +3.
D. Respirations of 10 per minute.

Correct Answer: D. Respirations of 10 per minute.

Magnesium toxicity can occur from magnesium sulfate therapy. Signs of toxicity relate to the
central nervous system depressant effects of the medication and include respiratory depression,
loss of deep tendon reflexes, and a sudden drop in the fetal heart rate and maternal heart rate
and blood pressure.

• Option A: Although deep tendon reflexes are more useful in assessing magnesium toxicity, the
presence of clonus may indicate an increased risk of convulsions.

• Option B: Therapeutic levels of magnesium are 4-7 mEq/L. Magnesium sulfate is the first-line
treatment for the prevention of primary and recurrent eclamptic seizures. For eclamptic seizures
that are refractory to magnesium sulfate, lorazepam and phenytoin may be used as second-line
agents.

• Option C: Proteinuria of +3 would be noted in a client with preeclampsia. Proteinuria is defined


as the presence of at least 300 mg of protein in a 24-hour urine collection, a protein
(mg/dL)/creatinine (mg/dL) ratio greater than or equal to 0.3, or a urine dipstick protein of 1+ (if a
quantitative measurement is unavailable). Serial confirmations 6 hours apart increase the
predictive value. Although more convenient, a urine dipstick value of 1+ or more (30 mg/dL) is
not reliable in the diagnosis of proteinuria.

38. A woman with preeclampsia is receiving magnesium sulfate. The nurse assigned to care for
the client determines that the magnesium therapy is effective if:*

A. Ankle clonus is noted.


B. The blood pressure decreases.
C. Seizures do not occur.
D. Scotomas are present.

Correct Answer: C. Seizures do not occur.

For a client with preeclampsia, the goal of care is directed at preventing eclampsia (seizures).
Seizures were a half or a third less likely to recur after treatment with magnesium. Maternal
mortality was also lower in women allocated magnesium rather than phenytoin or diazepam,
although this did not achieve statistical significance. Recent Cochrane reviews, however,
indicated a significant reduction in maternal mortality with magnesium.
• Option A: Ankle clonus indicated hyperreflexia and may precede the onset of eclampsia.
Although brisk or hyperactive reflexes are common during pregnancy, clonus is a sign of
neuromuscular irritability that usually reflects severe preeclampsia.

• Option B: Magnesium sulfate is an anticonvulsant, not an antihypertensive agent. Although a


decrease in blood pressure may be noted initially, this effect is usually transient.

• Option D: Scotomas are areas of complete or partial blindness. Visual disturbances, such as
scotomas, often precede an eclamptic seizure.

39. In the 12th week of gestation, a client completely expels the products of conception. Because
the client is Rh-negative, the nurse must:*

A. Administer RhoGAM within 72 hours.


B. Make certain she receives RhoGAM on her first clinic visit.
C. Not give RhoGAM, since it is not used with the birth of a stillborn.
D. Make certain the client does not receive RhoGAM since the gestation only lasted 12 weeks.

Correct Answer: A. Administer RhoGAM within 72 hours.

RhoGAM is given within 72 hours postpartum if the client has not been sensitized already. When
the blood of an Rh-positive fetus gets into the bloodstream of an Rh-negative woman, her body
will recognize that the Rh-positive blood is not hers. Her body will try to destroy it by making
anti-Rh antibodies. These antibodies can cross the placenta and attack the fetus’s blood cells.
This can lead to serious health problems, even death, for a fetus or a newborn.

• Option B: RhoGAM is a prescription medicine that is used to prevent Rh immunization, a


condition in which an individual with Rh-negative blood develops antibodies after exposure to
Rh-positive blood. RhoGAM is administered by intramuscular (IM) injection. RhoGAM is purified
from human plasma containing anti-Rh (anti-D).

• Option C: The doctor will administer at least one dose of RhoGAM between 26 and 28 weeks of
pregnancy. If the baby is found to be Rh-positive at birth, the mother will receive an additional
dose within 72 hours after delivery.

• Option D: 1 Rh-negative pregnant woman in 5 will become sensitive to the Rh-positive factor if
she doesn’t receive RhoGAM. That means that her baby can be born with one or more of the
following things: anemia, a lack of healthy red blood cells. heart failure.
40. During a prenatal examination, the nurse draws blood from a young Rh-negative client and
explain that an indirect Coombs test will be performed to predict whether the fetus is at risk for:*

A. Acute hemolytic disease


B. Respiratory distress syndrome
C. Protein metabolic deficiency
D. Physiologic hyperbilirubinemia

Correct Answer: A. Acute hemolytic disease.

When an Rh-negative mother carries an Rh-positive fetus there is a risk for maternal antibodies
against Rh-positive blood; antibodies cross the placenta and destroy the fetal RBCs.

• Option B: Respiratory distress syndrome, also known as hyaline membrane disease, occurs
almost exclusively in premature infants. In premature infants, respiratory distress syndrome
develops because of impaired surfactant synthesis and secretion leading to atelectasis,
ventilation-perfusion (V/Q) inequality, and hypoventilation with resultant hypoxemia and
hypercarbia.

• Option C: Infants with protein metabolism disorders are unable to metabolize certain amino
acids and require specialized formulas without the offending amino acid, allowing the baby to
receive essential nutrients for growth.

• Option D: Physiologic jaundice is also referred to as non-pathologic jaundice, and it is mild and
transient. This occurs because of differences in the metabolism of bilirubin in the neonatal period
leading to an increased bilirubin load.

41. A 26-year old multigravida is 14 weeks pregnant and is scheduled for an alpha-fetoprotein
test. She asks the nurse, “What does the alpha-fetoprotein test indicate?” The nurse bases a
response on the knowledge that this test can detect:*

A. Kidney defects
B. Cardiac defects
C. Neural tube defects
D. Urinary tract defects

Correct Answer: C. Neural tube defects.

The alpha-fetoprotein test detects neural tube defects and Down syndrome. Alpha-fetoprotein
(AFP) is a plasma protein produced by the embryonic yolk sac and the fetal liver. AFP levels in
serum, amniotic fluid, and urine functions as a screening test for congenital disabilities,
chromosomal abnormalities, as well as some other adult occurring tumors and pathologies.

• Option A: In some cases, one or both kidneys may fail to develop. In other instances, an
abnormality may be present that blocks the outflow of urine. This blockage may cause urine to
back up into the kidney, a condition called hydronephrosis, which causes the kidney to appear
enlarged on the ultrasound test. Another common abnormality is called reflux. This occurs when
a valve-like mechanism at the point where the ureter joins the bladder does not work, allowing
urine to wash back up into the kidney.

• Option B: The baby’s heart begins to form immediately after conception and is complete by
eight week’s gestation. The heart begins as a tube-shaped structure that twists and divides to
form the heart and heart valves. A congenital heart defect usually occurs because the heart does
not twist or divide normally. Some mothers wonder if drugs, alcohol, or medications contributed to
their child’s heart defect. In most cases, we don’t know why these defects occur. Although, some
heart defects can run in families or be related to a disease the mother has, diabetes mellitus, for
example.

• Option D: Common birth defects of the urinary system include hypospadias, obstructive defects
of the renal pelvis, and renal agenesis. Hypospadias is characterized by the location of the
urethral opening on the underside of the penis. Obstructive defects of the renal pelvis prevent
urine from entering the bladder.

42. A 21-year old client, 6 weeks pregnant is diagnosed with hyperemesis gravidarum. This
excessive vomiting during pregnancy will often result in which of the following conditions*

A. Bowel perforation
B. Electrolyte imbalance
C. Miscarriage
D. Pregnancy induced hypertension (PIH)

Correct Answer: B. Electrolyte imbalance.

Excessive vomiting in clients with hyperemesis gravidarum often causes weight loss and fluid,
electrolyte, and acid-base imbalances. Severe hyperemesis requiring hospital admission occurs
in 0.3-2% of pregnancies.

• Option A: Gastrointestinal perforation (GP) occurs when a hole forms all the way through the
stomach, large bowel, or small intestine. It can be due to a number of different diseases,
including appendicitis and diverticulitis. It can also be the result of trauma, such as a knife wound
or gunshot wound.
• Option C: Most miscarriages occur because the fetus isn’t developing normally. About 50
percent of miscarriages are associated with extra or missing chromosomes. Most often,
chromosome problems result from errors that occur by chance as the embryo divides and grows
— not problems inherited from the parents.

• Option D: Gestational hypertension refers to hypertension with onset in the latter part of
pregnancy (>20 weeks’ gestation) without any other features of preeclampsia, and followed by
normalization of the blood pressure postpartum. Of women who initially present with apparent
gestational hypertension, about one third develops the syndrome of preeclampsia.

43. Clients with gestational diabetes are usually managed by which of the following therapies?*

A. Diet
B. NPH insulin (long-acting)
C. Oral hypoglycemic drugs
D. Oral hypoglycemic drugs and insulin

Correct Answer: A. Diet.

Clients with gestational diabetes are usually managed by diet alone to control their glucose
intolerance. The initial treatment in these patients with Type 2 diabetes and gestational diabetes
is by diet modification and in the event of failure of control with diet, the treatment is shifted to
insulin to achieve tight glucose control with no risk of placental transfer.

• Option B: Insulin therapy remains the standard of care for type 1 diabetes, type 2 diabetes, and
uncontrolled GDM during pregnancy. Regular insulin, insulin aspart, insulin lispro, and NPH have
the most human pregnancy data.

• Option C: There are significant concerns about recommending oral hypoglycemic agents in
pregnancy because of their possible risks of transplacental passage and consequent fetal
teratogenesis, hypoglycemia, hyperbilirubinemia, and polycythemia.

• Option D: The pathogenesis of both gestational diabetes and Type 2 diabetes are insulin
resistance and inadequate insulin secretion, and hence the beneficial role of sulfonylurea is
evident.

44. The antagonist for magnesium sulfate should be readily available to any client receiving IV
magnesium. Which of the following drugs is the antidote for magnesium toxicity?*

A. Calcium gluconate
B. Hydralazine (Apresoline)
C. Narcan
D. RhoGAM

Correct Answer: A. Calcium gluconate.

Calcium gluconate is the antidote for magnesium toxicity. Ten ml of 10% calcium gluconate is
given IV push over 3-5 minutes. Hydralazine is given for sustained elevated blood pressures in
preeclamptic clients.

• Option B: Hydralazine is used to treat high blood pressure. Hydralazine is in a class of


medications called vasodilators. It works by relaxing the blood vessels so that blood can flow
more easily through the body.

• Option C: Narcan (naloxone) is an opioid antagonist used for the complete or partial reversal of
opioid overdose, including respiratory depression. Narcan is also used for diagnosis of suspected
or known acute opioid overdose and also for blood pressure support in septic shock. Narcan is
available in generic form.

• Option D: RhoGAM is a prescription medicine that is used to prevent Rh immunization, a


condition in which an individual with Rh-negative blood develops antibodies after exposure to
Rh-positive blood.

45. A maternity nurse is caring for a client with abruptio placenta and is monitoring the client for
disseminated intravascular coagulopathy. Which assessment finding is least likely to be
associated with disseminated intravascular coagulation?*

A. Swelling of the calf in one leg


B. Prolonged clotting times
C. Decreased platelet count
D. Petechiae, oozing from injection sites, and hematuria

Correct Answer: A. Swelling of the calf in one leg

DIC is a state of diffuse clotting in which clotting factors are consumed, leading to widespread
bleeding. Swelling and pain in the calf of one leg are more likely to be associated with
thrombophlebitis.

• Option B: Fibrin plugs may clog the microvasculature diffusely, rather than in an isolated area.
Derangement of the fibrinolytic system further contributes to intravascular clot formation, but in
some cases, accelerated fibrinolysis may cause severe bleeding. Hence, a patient with DIC can
present with a simultaneously occurring thrombotic and bleeding problem, which obviously
complicates the proper treatment.

• Option C: Platelets are decreased because they are consumed by the process; coagulation
studies show no clot formation (and are thus normal to prolong). Exposure to tissue factor (TF) in
the circulation occurs via endothelial disruption, tissue damage, or inflammatory or tumor cell
expression of procoagulant molecules (including TF). TF activates coagulation via the extrinsic
pathway involving factor VIIa. The TF-VIIa complex activates thrombin, which cleaves fibrinogen
to fibrin while simultaneously causing platelet aggregation..

• Option D: The presence of petechiae, oozing from injection sites, and hematuria are signs
associated with DIC. With acute DIC, the physical findings are usually those of the underlying or
inciting condition; however, patients with the acute disease (ie, the hemorrhagic variety
associated with excess plasmin formation) have petechiae on the soft palate, trunk, and
extremities from thrombocytopenia and ecchymosis at venipuncture sites. These patients also
manifest ecchymosis in traumatized areas.

46. A nurse is assessing a pregnant client in the 2nd trimester of pregnancy who was admitted to
the maternity unit with a suspected diagnosis of abruptio placentae. Which of the following
assessment findings would the nurse expect to note if this condition is present?*

A. Absence of abdominal pain


B. A soft abdomen
C. Uterine tenderness/pain
D. Painless, bright red vaginal bleeding

Correct Answer: C. Uterine tenderness/pain

In abruptio placentae, acute abdominal pain is present. Uterine tenderness and pain accompany
placental abruption, especially with a central abruption and trapped blood behind the placenta.
Observation of the fetal monitoring often reveals increased uterine resting tone, caused by failure
of the uterus to relax in an attempt to constrict blood vessels and control bleeding.

• Option A: Placental abruption occurs when there is a compromise of the vascular structures
supporting the placenta. In other words, the vascular networks connecting the uterine lining and
the maternal side of the placenta are torn away. These vascular structures deliver oxygen and
nutrients to the fetus.

• Option B: The abdomen will feel hard and board like on palpation as the blood penetrates the
myometrium and causes uterine irritability. Disruption of the vascular network may occur when
the vascular structures are compromised because of hypertension or substance use or by
conditions that cause stretching the uterus. The uterus is a muscle and is elastic whereas the
placenta is less elastic than the uterus. Therefore, when the uterine tissue stretches suddenly,
the placenta remains stable and the vascular structure connecting the uterine wall to the placenta
tears away.

• Option D: If bleeding is present, the quantity and characteristic of the blood, as well as the
presence of clots, is evaluated. Remember, the absence of vaginal bleeding does not eliminate
the diagnosis of placental abruption.

47. A maternity nurse is preparing for the admission of a client in the 3rd trimester of pregnancy
that is experiencing vaginal bleeding and has a suspected diagnosis of placenta previa. The
nurse reviews the physician’s orders and would question which order?*

A. Prepare the client for an ultrasound.


B. Obtain equipment for external electronic fetal heart monitoring.
C. Obtain equipment for a manual pelvic examination.
D. Prepare to draw a Hgb and Hct blood sample.

Correct Answer: C. Obtain equipment for a manual pelvic examination.

Manual pelvic examinations are contraindicated when vaginal bleeding is apparent in the 3rd
trimester until a diagnosis is made and placenta previa is ruled out. Digital examination of the
cervix can lead to maternal and fetal hemorrhage.

• Option A: A diagnosis of placenta previa is made by ultrasound. A patient presenting with


vaginal bleeding in the second or third trimester should receive a transabdominal sonogram
before a digital examination. If there is a concern for placenta previa, then a transvaginal
sonogram should be performed to confirm the location of the placenta. Transvaginal sonogram
has been shown to be superior to a transabdominal sonogram and is safe.

• Option B: External fetal monitoring is crucial in evaluating the fetus that is at risk for severe
hypoxia. Placental abruption presents with severe abdominal pain, vaginal bleeding, and
electronic fetal monitoring may show tachysystole and a nonreassuring fetal heart tracing; this
too can lead to high morbidity in mortality to the fetus and mother secondary to hemorrhage.

• Option D: The H/H levels are monitored, and external electronic fetal heart rate monitoring is
initiated. A leading cause of third-trimester hemorrhage, placenta previa presents classically as
painless bleeding. Bleeding is thought to occur in association with the development of the lower
uterine segment in the third trimester. Placental attachment is disrupted as this area gradually
thins in preparation for the onset of labor; this leads to bleeding at the implantation site, because
the uterus is unable to contract adequately and stop the flow of blood from the open vessels.
48. An ultrasound is performed on a client at term gestation that is experiencing moderate
vaginal bleeding. The results of the ultrasound indicate that an abruptio placenta is present.
Based on these findings, the nurse would prepare the client for:*

A. Complete bed rest for the remainder of the pregnancy.


B. Delivery of the fetus.
C. Strict monitoring of intake and output.
D. The need for weekly monitoring of coagulation studies until the time of delivery.

Correct Answer: B. Delivery of the fetus.

The goal of management in abruptio placentae is to control the hemorrhage and deliver the fetus
as soon as possible. Delivery is the treatment of choice if the fetus is at term gestation or if the
bleeding is moderate to severe and the mother or fetus is in jeopardy.

• Option A: Placental abruption occurs when there is a compromise of the vascular structures
supporting the placenta. In other words, the vascular networks connecting the uterine lining and
the maternal side of the placenta are torn away. These vascular structures deliver oxygen and
nutrients to the fetus. Disruption of the vascular network may occur when the vascular structures
are compromised because of hypertension or substance use or by conditions that cause
stretching the uterus. The uterus is a muscle and is elastic whereas the placenta is less elastic
than the uterus.

• Option C: Evaluation of vital signs to detect tachycardia or hypotension, which may be


indicators of a concealed hemorrhage are taken. Blood specimens such as a complete blood
count (CBC), fibrinogen, clotting profile, and type and RH may be collected. These laboratory
values will not aid in the diagnosis of placental abruption but will provide baseline data against
which to evaluate the patient’s condition over time.

• Option D: Women classified with a class 1 or mild placental abruption and no signs of maternal
or fetal distress and a pregnancy less than 37 weeks gestation may be managed conservatively.
These patients are usually admitted to the obstetrical unit for close monitoring of maternal and
fetus status. Intravenous access and blood work for type and crossmatch is part of the plan of
care. The maternal-fetal dyad will continue to be monitored until there is a change in condition or
until fetal maturity is reached.

49. Client is being treated for DVT. The nurse understands that the client’s response to treatment
will be evaluated by regularly assessing the client for:*

A. Dysuria, ecchymosis, and vertigo


B. Epistaxis, hematuria, and dysuria
C. Hematuria, ecchymosis, and epistaxis
D. Hematuria, ecchymosis, and vertigo

Correct Answer: C. Hematuria, ecchymosis, and epistaxis.

The treatment for DVT is anticoagulant therapy. The nurse assesses for bleeding, which is an
adverse effect of anticoagulants. This includes hematuria, ecchymosis, and epistaxis. Dysuria
and vertigo are not associated specifically with bleeding. The cornerstone of treatment is
anticoagulation. NICE guidelines only recommend treating proximal DVT (not distal) and those
with pulmonary emboli. In each patient, the risks of anticoagulation need to be weighed against
the benefits.

• Option A: Low-molecular-weight heparin or fondaparinux for five days or until INR is greater
than 2 for 24 hours (unfractionated heparin for patients with renal failure and increased risk of
bleeding). If platelet count drops to less than 75,000, switch from heparin to fondaparinux, which
is not associated with heparin-induced thrombocytopenia.

• Option B: The use of thrombolytic therapy can result in an intracranial bleed, and hence, careful
patient selection is vital. Recently endovascular interventions like catheter-directed extraction,
stenting, or mechanical thrombectomy have been tried with moderate success. The duration of
treatment for DVT is for 3-6 months, but recurrent episodes may require at least 12 months of
treatment. Patients with cancer need long term treatment.

Option D: Inferior vena cava filters are not recommended in acute DVT. There are both
permanent and temporary inferior vena cava filters available. These devices may decrease the
rate of recurrent DVT but do not affect survival. Today, only patients with contraindications to
anticoagulation with an increased risk of bleeding should have these filters inserted

50. What is the first sign of hydramnios in a pregnant woman?*

A. Shortness of breath
B. Varicosities and hemorrhoids
C. Difficulty in auscultating the fetal heart rate
D. Rapid growth of the uterus

Answer: D. Rapid growth of the uterus

• Rapid growth of the uterus is the first noticeable sign of hydramnios.

• Option A: This symptom occurs later in the development of the disease.

• Option B: Varicosities and hemorrhoids are symptoms yet they do not appear first.
• Option C: Difficulty in auscultating the heart rate is also a symptom yet develops later in the
progress of the disease.

51. What is the common reason for oligohydramnios?*

A. A bladder or renal disorder in the fetus that interferes with voiding


B. Decrease production of amniotic fluid
C. A small uterine capacity to hold the amniotic fluid
D. Perforation of the amniotic sac

Answer: A. A bladder or renal disorder in the fetus that interferes with voiding
• The fetus may have difficulty in voiding as it swallows the amniotic fluid leading to a decrease in
the amount of amniotic fluid.
• Option B: The production of the amniotic fluid is normal.
• Option C: The uterine capacity does not affect the amount of amniotic fluid produced.
• Option D: There is no perforation of the amniotic sac in oligohydramnios.

52. A patient is currently 34 weeks pregnant with her first baby. Which findings below could
indicate the development of preeclampsia in this patient that would need to be reported to the
physician? Select all that apply:*

A. blood pressure 144/100, 1700: blood pressure 120/80


B. 3+ dipstick urine protein
C. 1 hour glucose tolerance test 90 mg/dL
D. 0800: blood pressure 142/92, 1230: blood pressure: 144/98
E. <300 mg/dL 24-hour urine protein

The answers are B and D. Signs and symptoms of preeclampsia include: proteinuria (>1+
dipstick urine protein or >300 mg/dL 24 hour urine protein, hypertension >140/90.Two reading at
least 4-6 hours apart), swelling in face, eyes, extremities, headaches, vision changes, etc.

53. Which of the following physical findings would lead the nurse to suspect that a client with
severe pre-eclampsia has developed HELLP syndrome? Select all that apply.*

A. +3 pitting edema.
B. Petechiae.
C. Jaundice.
D. +4 deep tendon reflexes.
E. Elevated specific gravity.

54. The Human Immunodeficiency Virus (HIV) mainly attacks what type of cells in the human
body?*

A. Red Blood Cells


B. CD4 positive cells
C. Stem Cells
D. Platelets

The answer is B. The HIV virus attacks the human body's immune system, specifically the CD4
positive cells. Mainly the helper t cells. These cells are white blood cells that help the immune
system fight infection.

55. Nausea and vomiting is the most common complaint of pregnancy. If it becomes severe it is
diagnosed as:*

A. Hyperemesis gravidarum
B. Pica
C. Ptyalism
D. Pseudocyesis

Answer: A

□ Excessive nausea and vomiting during pregnancy is called hyperemesis gravidarum. Pica
refers to cravings of pregnant women for unusual food stuffs. Ptyalism is excessive salivation.
Pseudocyesis refers to false belief that one is pregnant.

56. During a pregnant client’s initial visit to the facility, the physician diagnose gonorrhea. Which
fact about gonorrhea should the nurse explain to the client?*

A. Her partner won’t need to be treated.


B. The disease will cause no problems for her neonate.
C. Unless she’s treated, her neonate can become infected during delivery.
D. Unless she’s treated, her neonate can have severe congenital defect.
Feedback

Majority of woman infected with gonorrhea does not experience any signs and symptoms and
may unknowingly transmit the disease to their sexual partners. Most common means of
transmission of the disease in adult women is by sexual intercourse. Gonorrhea is transmitted by
direct contract of the infected mucous membranes of the mouth, anus, and genitals of a person
to another person . Newborns acquire the infection when microorganisms in the birth canal enter
their eyes resulting in opthalmia neonatorum.

57. Human immunodeficiency virus is not transmitted through:*

A. Sexual contact
B. Kissing
C. Blood transfusion
D. Breastfeeding

QUIZ ON G&D
1. The nurse is assessing a six-month-old child. Which developmental skills are normal and
should be expected?
A. Speaks in short sentences.

B. Sits alone.

C. Can feed self with a spoon.

D. Pulling up to a standing position.

2. While teaching a 10 year-old child about their impending heart surgery, the nurse should
A. Provide a verbal explanation just prior to the surgery
B. Provide the child with a booklet to read about the surgery
C. Introduce the child to another child who had heart surgery three days ago
D. Explain the surgery using a model of the heart

3. The nurse prepares for a Denver Screening test with a 3 year-old child in the clinic. The
mother asks the nurse to explain the purpose of the test. The BEST response is to tell her
that the test

A. Measures potential intelligence


B. Assesses a child’s development
C. Evaluates psychological responses
D. Diagnoses specific problems

4. The nurse is planning care for an 18 month-old child. Which of the following should be
included the in the child’s care?
A. Hold and cuddle the child often
B. Encourage the child to feed himself finger food
C. Allow the child to walk independently on the nursing unit
D. Engage the child in games with other children

5. The nurse is assessing a four month-old infant. The nurse would anticipate finding that the
infant would be able to
A. Hold a rattle
B. Bang two blocks
C. Drink from a cup
D. Wave “bye-bye”

6. Which stage of development is most unstable and challenging regarding development of


personal identity?
A. Adolescence
B. Toddler hood
C. Childhood
D. Infancy

7. Which age group would have a tendency towards eating disorders?


A. Adolescence
B. Toddler hood
C. Childhood
D. Infancy

8. A maternity nurse is providing instruction to a new mother regarding the psychosocial


development of the newborn infant. Using Erikson’s psychosocial development theory, the
nurse would instruct the mother to
A. Allow the newborn infant to signal a need
B. Anticipate all of the needs of the newborn infant
C. Avoid the newborn infant during the first 10 minutes of crying
D. Attend to the newborn infant immediately when crying

9. A mother of a 3-year-old tells a clinic nurse that the child is rebelling constantly and having
temper tantrums. The nurse most appropriately tells the mother to:
A. Punish the child every time the child says “no”, to change the behavior
B. Allow the behavior because this is normal at this age period
C. Set limits on the child’s behavior
D. Ignore the child when this behavior occurs

10. The mother of a 3-year-old is concerned because her child still is insisting on a bottle at
nap time and at bedtime. Which of the following is the most appropriate suggestion to the
mother?
A. Do not allow the child to have the bottle
B. Allow the bottle during naps but not at bedtime
C. Allow the bottle if it contains juice
D. Allow the bottle if it contains water

11. A nurse is evaluating the developmental level of a 2-year-old. Which of the following does
the nurse expect to observe in this child?
A. Uses a fork to eat
B. Uses a cup to drink
C. Uses a knife for cutting food
D. Pours own milk into a cup

12. A clinic nurse assesses the communication patterns of a 5-month-old infant. The nurse
determines that the infant is demonstrating the highest level of developmental achievement
expected if the infant:
A. Uses simple words such as “mama”
B. Uses monosyllabic babbling
C. Links syllables together
D. Coos when comforted
13. Which finding would the nurse consider abnormal when performing a physical assessment
on a 6-month-old?

A. Anterior fontanel is open.


B. Beginning signs of tooth eruptionB. .
C. Able to track and follow objects.
D. Posterior fontanel is open.

14. A 12-month-old boy weighed 8 lb 2 oz at birth. Understanding developmental milestones,


what should the nurse caring for the child expect the current weight to be?

A. 24 lb 6 oz
B. 32 lb 8 oz
C. 20 lb 5 oz
D. 16 lb 4 oz

QUIZ ON HOSPITALIZED CHILD AND PAIN


MANAGEMENT
1. While giving nursing care to a hospitalized adolescent, the nurse should be aware that the
MAJOR threat felt by the hospitalized adolescent is

A. Pain management

B. Restricted physical activity

C. Altered body image

D. Separation from family

2. The nurse is observing children playing in the hospital playroom. She would expect to see 4
year-old children playing

A. Competitive board games with older children

B. With their own toys along side with other children

C. Alone with hand held computer games

D. Cooperatively with other preschoolers


3. Which age group has the greatest potential to demonstrate regression when they are sick?

A. Adolescent

B. Young Adult

C. Toddler

D. Infant

4. The parents of a 2-year-old arrive at a hospital to visit their child. The child is in the playroom
when the parents arrive. When the parents enter the playroom, the child does not readily
approach the parents. The nurse interprets this behavior as indicating that:

A. The child is withdrawn

B. The child is self-centered

C. The child has adjusted to the hospitalized setting

D. This is a normal pattern

5. A nurse is preparing to care for a 5-year-old who has been placed in traction following a
fracture of the femur. The nurse plans care, knowing that which of the following is the most
appropriate activity for this child?

A. Large picture books

B. A radio

C. Crayons and coloring book

D. A sports video

6. A 16-year-old is admitted to the hospital for acute appendicitis, and an appendectomy is


performed. Which of the following nursing interventions is most appropriate to facilitate normal
growth and development?

A. Allow the family to bring in the child’s favorite computer games

B. Encourage the parents to room-in with the child

C. Encourage the child to rest and read


D. Allow the child to participate in activities with other individuals in the same age
group when the condition permits

7. A 16 year old child is hospitalized, according to Erik Erikson, what is an appropriate


intervention?

A. tell the friends to visit the child

B. encourage patient to help child learn lessons missed

C. call the priest to intervene

D. tell the child’s girlfriend to visit the child.

8. The nurse is planning pain control for a child. Which is the advantage of administering pain
medication by the intravenous (IV) bolus route?

A. Less expensive than oral medications

B. Produces a first-pass effect through the liver

C. Does not need to be administered frequently

D. Provides most rapid onset of effect, usually in about 5 minutes

9. Children as young as age 3 years can use facial scales for discrimination. What are some
suggested anchor words for the preschool age group?

A. “No hurt.”

B. “Red pain.”

C. “Zero hurt.”

D. “Least pain.”

QUIZ: Hematologic Disorders


1. The nurse writes a client problem of "activity intolerance" for a client diagnosed with anemia.
Which intervention should the nurse implement?

a. Pace activities according to tolerance.


b. Provide supplements high in iron and vitamins.

c. Administer packed red blood cells.

d. Monitor vital signs every four (4) hours.

The client is experiencing dyspnea on exertion, which is common for clients with anemia.
The client needs a wheelchair to limit the exertion.

2. Which statement is the scientific rationale for infusing a unit of blood in less than four (4) hours?

a. The blood will coagulate if left out of the refrigerator for >four (4) hours.
b. The blood has the potential for bacterial growth if allowed to infuse longer.
c. The blood components begin to break down after four (4) hours.
d. The blood will not be affected; this is a laboratory procedure
Blood will coagulate if left out for an extended period of, but blood is stored with a preservative that
prevents this and prolongs the life of the blood.
(CORRECT). Blood is a medium for bacterial growth, and any bacteria contaminating the unit will
begin to grow if left outside of a controlled refrigerated temperature for longer than four (4) hours,
placing the client at risk for septicemia.
Blood components are stable and do not break down after four (4) hours.
These are standard nursing and laboratory procedures to prevent the complication of septicemia."

3. Nurse Katrina is planning a client education program for sickle cell disease (SCD); what topics
should be included in the plan of care?

A. Aerobic exercise to improve oxygenation

B. Fluid restraint to 1 qt (1 L)/day

C. A high-iron, high-protein diet

D. Proper hand washing and infection avoidance


Proper hand washing and infection avoidance
Rationale: Prevention of infection is vital in the prevention of sickle cell crisis.

4. The client diagnosed with anemia begins to complain of dyspnea when ambulating in the hall.
Which intervention should the nurse implement first?

a. Apply oxygen via nasal cannula.

b. Get a wheelchair for the client.

c. Assess the client's lung fields.

d. Assist the client when ambulating in the hall.

The client is experiencing dyspnea on exertion, which is common for clients with anemia.
The client needs a wheelchair to limit the exertion.
TEST-TAKING HINT: The test taker should ask, "What is causing the distress and what will
alleviate the distress the fastest?" The distress is caused by exertion and it is occurring in the
hallway. The most expedient intervention is to have the client stop the activity that is causing the
distress; a chair will accomplish this, but the client should be returned to the room, so it needs to
be a wheelchair.

5. In children diagnosed with sickle cell disease (SCD), tissue damage results from which of the
following?

A. Air hunger and respiratory alkalosis due to deoxygenated red blood cells.
B. Hypersensitivity of the central nervous system (CNS) due to elevated serum bilirubin levels
C. A general inflammatory response due to an autoimmune reaction from hypoxia
D. Local tissue damage with ischemia and necrosis due to obstructed circulation

Rationale: Characteristic sickle cells tend to clump, which results in weak and inadequate blood
flow to tissue, local tissue damage, and eventual ischemia and necrosis.

6. Nurse Christine is planning a client education program for sickle cell disease (SCD) in children;
which of the following interventions would be included in the care plan?

A. Health teaching to help reduce sickling crises

B. Avoidance of the use of opioids

C. Administration of an anticoagulant to prevent sickling

D. Observation of the imposed fluid restriction

Rationale: Prevention is one of the principal goals of therapeutic management because there is no
cure for sickle cell disease. Consequently, health education to help lessen sickling crises is key

7. Mr. and Mrs. Estacio's child has hemophilia; which of the following actions would you instruct
them to avoid?

A. Immobilizing the joint

B. Lowering the injured area

C. Applying cold to the area

D. Applying pressure

Rationale: With hemophilia, the injured area must be elevated, not lowered

8. Mrs. Santos was instructed by the nurse on foods to encourage her child's diet concerning the
latter's iron deficiency anemia; which of the following if stated by the mother would indicate the
need for further instruction?

A. Fish

B. Lean meats
C. Whole-grain breads

D. Yellow vegetables

If a parent states that she should stress the intake of yellow vegetables, she needs additional
teaching because yellow vegetables are not good source of iron.

Options A, B, and C: Fish, lean meats, and whole-grain breads are good food sources of iron and
should be supported and encouraged.

9. Nurse Glenda is performing community assessment; which of the following age-groups would
be inappropriate for her to monitor iron deficiency anemia?

A. Toddlers

B. School-age children

C. Adolescents

D. Pregnant women

Women of childbearing age are the population with the most affected individuals, with an estimated
468 million being non-pregnant women, according to WHO1 and are the highest at-risk population
group for iron-deficiency anemia.

10. The long-term complications seen in thalassemia major are associated to which of the
following?

A. Anemia

B. Growth retardation

C. Hemochromatosis

D. Splenomegaly

11. It is a serious condition in which the bone marrow does not produce enough new blood cells. It
may be passed down from the parents or develop sometime during childhood:

A. Iron deficiency anemia

B. Sickle cell disease

C. Aplastic anemia

D. Thalassemia

E. Hemophilia

Aplastic anemia is a serious condition in which the bone marrow does not produce enough new
blood cells. It may be passed down from the parents or develop sometime during childhood. Some
symptoms include tiredness, paleness, frequent infections, and easy bruising and bleeding.
12. The patient with anemia has been prescribed an iron supplement. With which beverage should
the nurse encourage the patient to take the supplement?

A) Milk

B) Water only

C) Orange Juice

D) Tea

Vitamin C helps aid in the absorption of iron. Therefore, iron supplements should be taken with a
glass or orange juice or a vitamin C tablet.

13. The nurse is teaching the client prescribed with an iron supplement. Which statement, if made
by the patient, indicates the need for further teaching?

A) I should take this medication on an empty stomach


B) When I start taking this pill, I will start with only one tablet a day and then increase to two

C) My stools can become really dark in color when I take this supplement

D) I need to watch out for diarrhea that can occur with this supplement

Constipation is expected with this supplement, not diarrhea. Increasing fiber in the diet can prevent
this problem. Food in the stomach impairs iron absorption so the patient should take it on an empty
stomach. Slowly increasing the amount of pills can help prevent any GI discomfort. Stool is
expected to become dark (almost black).

14. Which of the following symptoms is expected with hemoglobin of 10 g/dl?

a. None

b. Pallor

c. Palpitations

d. Shortness of breath

Symptoms are not expected with mild anemia

QUIZ: Infertility
1. This is a condition of inability to conceive after at least 1 year of sexual intercourse at least four
times per week without contraception.

a. Subfertility

b. Infertility
c. Sterility

d. all of the above

2. It is said to exist when a pregnancy has not occurred after at least 1 year of engaging in
unprotected coitus

a. Subfertility
b. Infertility

c. Sterility

d. all of the above

3. It is the inability to ejaculate into the vagina or delayed ejaculation. Male difficulty to reach
orgasm.

a. retarded ejaculation

b. rapid ejaculation

c. impotence

d. erectile dysfunction

4. It is a persistent or recurring absence of sexual thoughts or disinterest in sexual activity.

a. impotence

b. hyperactive sexual desire

c. hypoactive sexual desire

d. sexual arousal disorder

5. A leading cause of infertility in both men and women is sexually transmitted diseases (STDs)
and scarring or damage of the fallopian tubes may cause infertility

a. urinary tract infection

b. yeast infection

c. either a or b

d. neither a nor b

6. A condition wherein partner experience pain during intercourse due to inadequate lubrication,
infection or hormonal imbalance.
a. dyspareunia

b. vaginitis

c. salphingitis

d. any of the above

7. It is an uncommon but potential cause of female infertility, and the cervical opening is narrower
than it should be.

a. cervical dilatation

b. cervical stenosis

c. cervical conization

d. dilatation and curettage

8. The most common cause of subfertility in women, may occur from a genetic abnormality in
which there are no ovaries to produce ova.

a. Turner’s syndrome (hypogonadism)

b. Myoma

c. Endometriosis

d. Polycystic ovary syndrome

9. The secretion of progesterone by the ovary is below normal or the endometrium isn't responding
to the normal stimulation of progesterone

a. inadequate estrogenic phase

b. overly producing progesterone phase

c. Insufficient luteal phase

d. overly producing secretory phase

10. A condition where the urethral opening on male's reproductive system is in the ventral surface
of the penis located near the base of the penile shaft.

a. epispadias

b. hypospadias
c. penoscrotal

d. subcoronal

11. The minimum normal sperm count per milliliter of seminal fluid.

15 million

20 million

25 million

30 million

12. A condition occurs when veins inside the scrotum become enlarged. This has also been linked
to decreased sperm motility.

a. varicocele

b. meningocele

c. either a or b

d. neither a nor b

13. A condition wherein most often the result of a bacterial infection and some viral cause like in
the mumps.

a. hernia

b. varicocele

c. orchitis

d. any of the above

14. The least costly way to determine a woman’s ovulation pattern is to ask her to record her basal
body temperature (BBT) for at least 4 months.

True

False

15. Endometrial biopsy is used to reveal an endometrial problem and done 2 or 3 days after
menstruation.

True
False

Post Quiz: Pedia Respi Disorders

1. What is the danger sign manifestation of Choanal Atresia?*

● CYANOSIS

2. Which of the following is not a nursing care of Choanal Atresia?*

A. Participate in early screening: Occlude two nostril (by pressing the both outside of
the nostril using a finger) at a time during the initial assessment of the newborn after
birth.
B. If the left nostril is occluded and the newborn is asymptomatic, the right nostril is likely
patent.
C. If the right nostril is occluded and the infant cries, the left nostril needs further investigation.
Remember that the newborn is an obligate nosebreather; if his nostrils are patent, he will
have normal, rapid (30-60 per min), shallow, quiet, synchronized and nasal breathing.
D. Maintain patency of an oral airway, if present. Monitor respiration.

3. Which of the following is not included in the unilateral manifestation of choanal atresia?*

A. Persistent nasal discharge


B. Recurrent sinus infections
C. cyanosis
D. Difficulty of breathing

4. Which of the following is not included in acute spasmodic laryngitis?*

A. Bark-like, metallic cough


B. Respiratory stridor
C. febrile
D. Retractions: substernal and suprasternal

Signs and Symptoms:


Bark-like, metallic cough
Respiratory stridor
Retractions: substernal and suprasternal
Afebrile
Restlessness
Dyspnea
Cyanosis: a late sign

5. What is the late sign manifestation of acute spasmodic laryngitis?*


● CYANOSIS

6. Identify the 2 causes of acute spasmodic laryngitis*


● PARAINFLUENZA VIRUSES
● MYCOPLASMA PNEUMONIAE
7. Identify nursing diagnosis of croup? (just nursing diagnosis without any related to*
● INEFFECTIVE AIRWAY CLEARANCE
● DEFICIENT FLUID VOLUME

8. Identify the following nursing care for croup that described as young children should be kept as
comfortable as possible, allowing him or her to remain in a parent’s arms and avoiding
unnecessary painful interventions that may cause agitation, respiratory distress, and lead to
increased oxygen requirements; persistent crying increases oxygen demands, and respiratory
muscle fatigue can worsen the airway obstruction.*

● DECREASE ANXIETY

9. Identify 1 contraindicated uncontrolled systemic disorder with tonsillectomy*

● RHEUMATIC FEVER
● CARDIAC DISORDER
● RENAL DISEASE
● DIABETES

10. Offering hot compress promote rest, comfort and relief pain in a post-tonsillectomy?*

● True
● False

11. How many hours is considered as nosocomial hospital acquired pneumonia?*


● MORE THAN 48 HOURS AFTER ADMISSION

12. What is the bacteria that cause the leading cause of community-acquired pneumonia*
● STREPTOCOCCUS PNEUMONIAE

13. What is the identified prevention for pneumonia in healthy patients with an efficiency of 65%
to 85%?*
● PNEUMOCOCCAL VACCINE

14. Which is the priority nursing diagnosis to pneumonia patient?(no related to)*

● INEFFECTIVE AIRWAY CLEARANCE

15. What is the first sign/manifestation of cystic fibrosis*


● MECONIUM ILEUS

16. What is the other term ro PPD or purified protein derivative?*

● MANTOUX TEST

17. Which of the following medications has a side effect of peripheral neuritis?*
● Isoniazid (INH).

18. What is the side effect of Rifampin to your client's secretions?*


● ORANGE
● RED

19. Which tuberculosis medication results to optic neuritiis?*


A. Isoniazid (INH)
B. Ethambutol (Myambutol)
C. Pyrazinamide
D. Rifampin (Rifadin)

20. How many hours will the PPD will be recheck?*


● 48 TO 72 HOURS

21. After an incubation period of 14 to 28 weeks, TB is usually asymptomatic in primary infection*

A. True
B. False
Pedia Respiratory Disorders
1. When developing a plan of care for a hospitalized child, nurse Mary knows that children in
which age group is most likely to view illness as a punishment for misdeeds?

A. Infancy
B. Preschool age
C. School age
D. Adolescence

Correct Answer: B. Preschool age Preschool-age children are most likely to view illness as a
punishment for misdeeds. When children in this age group become seriously ill, they may think it's
punishment for something they did or thought about. They don't understand how their parents
could not have protected them from this illness.

• Option A: Separation anxiety, although seen in all age groups, is most common in older infants.
Keeping a consistent routine is important for a baby and their caregivers. Because babies can't talk
about their needs, fear is often expressed by crying.

• Option C: Fear of the unknown, loss of control, and separation from family and friends can be the
school-aged child's main sources of anxiety and fear related to death. They may fear their own
death because of the uncertainty of what happens to them after they die.

• Option D: Fear of death is typical of adolescents. Adolescents also fear mutilation. Most teens are
starting to establish their identity, independence, and relation to peers. The main theme in teens is
feeling immortal or being exempt from death. Their realization of their own death threatens all of
these objectives.

2. Winnie is a 7-year-old girl diagnosed with cystic fibrosis. Which of the following must Nurse
Tricia keep in mind when developing a care plan for the child?*

A. Pulmonary secretions are abnormally thick.


B. Elevated levels of potassium are found in sweat.
C. CF is an autosomal dominant hereditary disorder.
D. Obstruction of the endocrine glands occurs.

Correct Answer: A. Pulmonary secretions are abnormally thick. CF is identified by abnormally thick
pulmonary secretions. Researchers now know that cystic fibrosis is an autosomal recessive
disorder of exocrine gland function most commonly affecting persons of Northern European
descent at a rate of 1 in 3500. It is a chronic disease that frequently leads to chronic
sinopulmonary infections and pancreatic insufficiency. The most common cause of death is
end-stage lung disease.
• Option B: Diagnosis of CF is based on elevated chloride levels detected in sweat. High levels of
salt in the sweat of patients with cystic fibrosis suggested an abnormality in electrolyte transport
from the sweat gland. Quinton postulated that sweat ducts in these patients were impermeable to
chloride.

• Option C: It is a chronic, inherited disorder, particularly an autosomal recessive hereditary


disorder concerning the exocrine, not endocrine glands. In 1949, Lowe et al. postulated that cystic
fibrosis must be caused by a genetic defect from the autosomal recessive pattern of inheritance of
the disease.

• Option D: The thick mucus blocks the exocrine glands. Further studies led to the hypothesis that
the faulty chloride channel must be situated in the apical membranes of the lung surface or
glandular epithelium to explain the respiratory and systemic organ failure associated with cystic
fibrosis.

3. Marisol is rushed to the emergency department during an acute, severe prolonged asthma
attack and is unresponsive to usual treatment. The condition is referred to as which of the
following?*

A. Status asthmaticus

B. Reactive airway disease

C. Intrinsic asthma

D. Extrinsic asthma

Correct Answer: A. Status asthmaticus Status asthmaticus is an acute, prolonged, severe asthma
attack that is unresponsive to usual treatment. Typically, the child requires hospitalization. One of
the most common causes of emergency room visits in the United States is status asthmaticus, an
acute, emergent episode of bronchial asthma that is poorly responsive to standard therapeutic
measures.

• Option B: Reactive airway disease is another general term for asthma. In children, the diagnosis
of RAD (reactive airway disease) or recurrent WARIs (wheezing-associated respiratory infections)
often precede a formal diagnosis of asthma.

• Option C: Intrinsic is a term used to denote internal precipitating factors, such as viruses. In
intrinsic asthma, IgE is usually only involved locally, within the airway passages. The airways
become more and more narrow, resulting in an asthma attack. Unlike extrinsic asthma, which is
triggered by commonly known allergens, intrinsic asthma may be triggered by a wide range of
non-allergy-related factors.

• Option D: Extrinsic is a term used to denote external precipitating factors, such as allergens.
Extrinsic asthma is more common than intrinsic asthma. In extrinsic asthma, symptoms are
triggered by an allergen (such as dust mites, pet dander, pollen, or mold). The immune system
overreacts, producing too much of a substance (called IgE) throughout the body. It’s the IgE that
triggers an extrinsic asthma attack.
4. The Tamayo family has been taking good care of their youngest, Buchik, who was
diagnosed with asthma. Which of the following statements indicate a need for further home
care teaching?*

A. "He should increase his fluid intake regularly to thin secretions."

B. "We'll make sure that he avoids exercise to prevent attacks."

C. "He is to use his bronchodilator inhaler before the steroid inhaler."

D. "We need to identify what things trigger his attack

5. You are caring for a 8-year-old child with asthma. The mother of the patient asks you to
explain the pathophysiology of the disease. When explaining what causes the obstruction
of the airway you mention except for:*
A. Contraction of muscles surrounding the bronchi
B. Swelling of bronchial membranes
C. Destruction of the alveolar wall
D. Thick mucus

6. A mother calls the clinic requesting an consultation appointment for her 8-year-old child.
She states he has asthma and is telling her he had trouble breathing last night and does
not want to go to school. In triaging this child, which is the most important question to
initially ask the mother?*
A. "Is your child crying and irritable?"
B. "Does your child have a productive cough?"
C. "Did he have a temperature last night of greater than 100° F?"
D. "Is your child telling you at this time he is having trouble breathing?"

7. The physical assessment done on Claire reveals these signs of acute bronchial asthma,
except:*
A. Rapid breathing and pulse
B. Wheezy respiration, more on expiration
C. Chest retractions
D. Rusty sputum

Answer: D
Signs of acute asthmatic attack include rapid respiration (tachypnea) and pulse (tachycardia),
wheezy respiration more on expiration, dyspnea, chest retractions, perspiration and anxiety. Rusty
sputum is characteristic of pneumonia.

8. Susie, age 9 years old, is admitted because of acute asthmatic attack. She also has cystic
fibrosis. The physical assessment of Susie yielded signs and symptoms typical of chronic
cystic fibrosis. Which is not a true sign?*
A. undernutrition
B. Clubbing of fingers
C. decreased perspiration
D. recurrent respiratory infection

Answer: C

Cystic fibrosis is an inherited disorder of the exocrine glands. It is a multi-system disorder


commonly affecting the lungs, pancreas, intestines, and sweat glands. The presence of excessive
thick/frothy secretions in the respiratory tract favors the growth of bacterial infection, recurrent
respiratory infections, and chronic hypoxemia, thus the clubbing of fingers and toes. The
affectation of the sweat glands causes excessive sweating or perspiration and increases loss of
electrolytes sodium and chlorides through sweat; thus, a person tastes salty when kissed and
becomes weak with exercises and during hot weather. The fibrosis of the pancreas causes
inadequate enzymes needed for digestion of carbohydrates, proteins and fats, resulting in
malabsorption syndrome, characterized primarily by steatorrhea, avitaminosis A-D-E-K (fat-soluble
vitamins), and under- or malnutrition.

9. Susie, age 9 years old, is admitted because of acute asthmatic attack. She also has cystic
fibrosis. Which of the following is used to confirm cystic fibrosis?*
A. Guthrie test
B. sweat test
C. stool test
D. all of these

Answer: B

The sweat test identifies excessive sodium and chloride in the sweat. The Guthrie test is the
screening test for phenylketonuria. Although a stool test is done in cystic fibrosis, it is not used to
confirm it; the characteristic steatorrheic stools may also be found in celiac disease and in other
malabsorption syndromes.
10. Susie, age 9 years old, is admitted because of acute asthmatic attack. For schooler Susie,
who has both cystic fibrosis and asthma, which of these exercises is best?*
A. running
B. swimming
C. basketball
D. baseball

Answer: B

For both cystic fibrosis and bronchial asthma, swimming is most appropriate, as it improves
expiration, strengthens muscles of respiration, and prevents excessive loss of electrolytes through
sweat.

11. Respiratory function is altered in clients with an asthmatic attack. Which of the following is
the cause of this alteration?*
A. inadequate surfactant production
B. increased airway resistance
C. paradoxical movements of the chest wall
D. narrowing of the upper air passages

Asthma is characterized by spasm and constriction of the smaller, lower airways resulting in
increased resistance to airflow. Inadequate surfactant production is found in preterm neonates and
in infants with sudden infant death syndrome. Paradoxical breathing occurs when a negative
intrathoracic pressure is transmitted to the abdomen by a weakened, poorly functioning diaphragm.

12. A resident treatment for cystic fibrosis is:*


A. Postural drainage
B. Antitussives
C. Narcotics
D. Antibiotics

Answer: A

To promote clearing the lower airway of thick mucus,, postural drainage is done at least twice a
day: before night sleep (to prevent coughing that disturbs sleep), and upon waking up (to eliminate
thick secretions that have accumulated through the night due to shallow breathing and depressed
cough reflex during sleep). A mucolytic drug is often prescribed to be given 30 minutes before
postural drainage. Expectorants, and not antitussives, are given to promote evacuation of
pulmonary secretions; antibiotics may be administered for treatment of infections, and at times as
prophylaxis, but not always. Narcotics tend to depress the respiratory center, and thus is not a part
of the medical management of children with chronic obstructive pulmonary disease such as cystic
fibrosis and bronchial asthma.

13. A nurse reinforces instructions to the mother of a child who has been hospitalized with
croup. Which of the following statements, if made by the mother, would indicate the need
for further instruction?*
A. “I will give my child cough syrup if a cough develops.”
B. “During an attack, I will take my child to a cool location.”
C. “I will give acetaminophen (Tylenol) if my child develops a fever.”
D. “I will be sure that my child drinks at least three to four glasses of fluids every day.”

Answer: A. “I will give my child cough syrup if a cough develops.”

• Option A: Cough syrups and cold medicines are not to be given, because they may dry and
thicken secretions.

• Option B: During a croup attack, the child can be taken to a cool basement or garage.

• Option C: Acetaminophen is used if a fever develops.

• Option D: Adequate hydration of 500 to 1000 mL of fluids daily is important for thinning
secretions.

14. A child with croup is placed in a cool, high-humidity tent connected to room air. The primary
purpose of the tent is to:*
A. Prevent insensible water loss.
B. Provide a moist environment with oxygen at 30%.
C. Prevent dehydration and reduce fever.
D. Liquefy secretions and relieve laryngeal spasm.

Answer: D. Liquefy secretions and relieve laryngeal spasm.

• Option D: The primary reason for placing a child with croup under a mist tent is to liquefy
secretions and relieve laryngeal spasms.

• Option A: Answer A is incorrect because it does not prevent insensible water loss.

• Option B: Answer B is incorrect because the oxygen concentration is too high.


• Option C: Answer C is incorrect because the mist tent does not prevent dehydration or reduce
fever.

15. A child with laryngotracheobronchitis (croup) is placed in a cool mist tent. The mother
becomes concerned because the child is frightened, consistently crying and trying to climb
out of the tent. Which is the MOST APPROPRIATE nursing action?*
A. Tell the mother that the child must stay in the tent.
B. Place a toy in the tent to make the child feel more comfortable.
C. Call the health care provider and obtain a prescription for a mild sedative.
D. Let the mother hold the child and direct the cool mist over the child’s face

Answer: D. Let the mother hold the child and direct the cool mist over the child’s face.

• Option D: To decrease the child’s anxiety, the mother may hold and accompany the child in the
tent.

• Option A: The child’s anxiety may increase and worsen the respiratory distress.

• Option B: Toys may introduce pathogens that may be harmful to the child.

• Option C: A mild sedative is unnecessary.

16. You have obtained the following assessment information about a 3-year old who has just
returned to the pediatric unit after having a tonsillectomy. Which finding requires the most
immediate follow-up?*
A. Frequent swallowing.
B. Hypotonic bowel sounds.
C. Complaints of a sore throat.
D. Heart rate of 112 beats/min.

Answer: A. Frequent swallowing.

• Option A: Frequent swallowing after a tonsillectomy may indicate bleeding. You should inspect
the back of the throat for evidence of bleeding.

• Options B, C, D: The other assessment results are not unusual in a 3-year old after surgery.
17. Isaiah is diagnosed with “strep throat.” Which clinical manifestation would the nurse expect
to the client?*
A. A fiery red pharyngeal membrane and fever.
B. Pain over the sinus area and purulent nasal secretions.
C. Foul-smelling breath and noisy respirations.
D. Weak cough and high-pitched noise on respirations.

Answer: A. A fiery red pharyngeal membrane and fever.

• Option A: Strep throat, or acute pharyngitis, results in a red throat, edematous lymphoid tissues,
enlarged lymph nodes, fever, and sore throat.

• Option B: Pain over the sinus area and purulent nasal secretions would be evident with sinusitis.

• Option C: Foul-smelling breath and respirations indicate adenoiditis.

• Option D: A weak cough and high-pitched noisy respirations are associated with foreign-body
aspiration.

18. Which of the following respiratory conditions is always considered a medical emergency?*

A. Asthma

B. Cystic fibrosis (CF)

C. Epiglottitis

D. Laryngotracheobronchitis (LTB)

19. Aminophylline (theophylline) is prescribed for a client with acute bronchitis. A nurse
administers the medication, knowing that the primary action of this medication is to:*
A. Promote expectoration.
B. Suppress the cough.
C. Relax smooth muscles of the bronchial airway.
D. Prevent infection.

20. Obstruction of the airway in the patient with asthma is caused by all of the following
except:*
A. Thick mucus
B. Swelling of bronchial membranes
C. Destruction of the alveolar wall
D. Contraction of muscles surrounding the bronchi

QUIZ 1: INTEGUMENTARY, PSYCHOSOCIAL


DISORDERS & PEDIATRIC INJURIES
1. A nurse is assessing an infant who has scabies. which of the following findings should the nurse
expect? (select all that apply) A. presence of nits on the hair shaft B. pencil-like marks on hands
C. blisters on the soles of the feet D. small red bumps on the scalp E. pimples on the trunk*

A. A,B,C
B. B, C, E
C. A, C,E
D. B,D,E

2. This is an infestation of mites, specific to this type, causing papular-vesicular lesions. These
burrow into the epidermis and lay eggs as well as leave feces.- They are transmitted by contact-
The intense itch leads to vesicular eruption*

A. IMPETIGO CONTAGIOSA
B. SCABIES
C. ATOPIC ECZEMA
D. NONE OF THESE

3. Permethrin (ELMITE) is the medication used to treat scabies. It is placed all over the body, from
the CHEST DOWN. Wait 30 minutes before dressing. One application is usually sufficient enough
to treat scabies. If necessary, wait 7 - 10 days to reapply.- This will be washed or showered off 12
hours later.*

A. TRUE
B. FALSE

4. The nurse is caring for a adolescent with Eczema. The lesions on the antecubital area appears
honey-colored with crusts surrounding erythema. The nurse knows that this finding indicates.*

A. a normal flare-up
B. infection
C. minute papules
D. progressing lesions

5. Which of the following is an important nursing consideration when caring for a child with
impetigo contagiosa?*

A. a. Apply topical corticosteroids to decrease inflammation.


B. b. Carefully remove dressings so as not to dislodge undermined skin, crusts, and debris.
C. c. Carefully wash hands and maintain cleanliness when caring for an infected child.
D. d. Examine a child under a Wood lamp for possible spread of lesions.

RATIONALE. A major nursing consideration related to bacterial skin infections, such as


impetigo contagiosa, is to prevent the spread of the infection and complications. This is
done by thorough hand washing before and after contact with the affected child.

6. Impetigo ordinarily results in which of the following?*

A. a. No scarring
B. b. Pigmented spots
C. c. Slightly depressed scars
D. d. Atrophic white scars

RATIONALE. Impetigo tends to heal without scarring unless a secondary infection occurs.

7. What is the treatment/care for impetigo?*

A. Systemic antibiotics are given. The treatment for impetigo also includes softening of
the crusts with warm saline soaks(1 : 20 Burow solution) and then soap-and-water
removal
B. Board spectrum antibiotics are given. The treatment for impetigo also includes hardening of
the crusts with warm saline soaks(1 : 20 Burow solution) and then soap-and-water removal
C. Systemic antibiotics are given. The treatment for impetigo also includes softening of the
crusts with warm saline soaks(1 : 40 Burow solution) and then soap-and-water removal
D. None of these

8. Impetigo contagiosa Drug of choice is __________.*

A. Penicillin
B. Cephalosporin
C. any broad spectrum antibiotics
D. Ampicillin

9. Atopic dermatitis is common in which of these groups?*

A. children of low socio-economic status


B. poorly nourished and underweight children
C. well-nourished infants aged 2-6 months
D. children of poorly educated parents

10. To help prevent atopic dermatitis, the parents should be taught the principles of introducing the
first solid foods in infancy. Which of the following is incorrect and should not be included in parental
teaching?*

A. The first solid food is rice cereal.


B. Introduce one food at a time.
C. Have an interval of 5-7 days before introducing another new food.
D. A small amount, 1 tablespoon, of the new food is introduced.

11. A 3-year-old child is hospitalized with burns covering her trunk and upper extremities. Which of
the following should the nurse use to have an early and accurate assessment of adequacy of fluid
resuscitation in the burned child?*

A. blood pressure
B. urine output
C. pulse rate
D. serum sodium level

12. A school-age child sustained 35 percent burns over his lower extremities. Which nursing
intervention would be considered the top priority that needs immediate implementation?*

A. Start an IV.
B. Immediately perform endotracheal intubation.
C. Clean and dress the wound.
D. Administer a tetanus booster.

13. The nurse is caring for a preschool child who was trapped in the garage and suffered an
inhalation injury. If the carbon monoxide level is 32%, which of these signs will the nurse likely
anticipate?*
A. Coma
B. Dizziness and vomiting
C. Flushing and headache
D. Tachycardia and tachypnea

14. The senior head nursing student reviews with the juniors the phases of management of burn
injury. Which statement, when given by the junior student Nalda, needs further explanation?*

A. “Emergent phase begins with initiation of fluids.”


B. “Acute phase begins when the client is hemodynamically stable and dieresis has begun.”
C. “The resuscitative phase ends when capillary integrity returns to near-normal levels.”
D. “Rehabilitative phase is the final phase of burn care and ends when patient achieve
maximal function.”

15. The nurse understands that poisoning is highest in incidence in the toddler stage because this
group is:*

A. seldom supervised by parents.


B. uses all its senses, including taste, to test reality.
C. is in the oral stage of development according to Freud.
D. lacks adequate cognitive development.

16. A mother frantically calls the ER asking what to do about her 2-year-old girl who was found
unconscious and with a bottle of pills beside her. The ER nurse tells the mother to:*

A. Give the child 15 mL of syrup of ipecac.


B. Give the child 10 mL syrup of ipecac with a sip of water.
C. Give the child 1 cup of water to induce vomiting.
D. Bring the child to the ER immediately

RATIONALE. (A) Before giving any emetic, the substance ingested must be known. (B) At
least 8 oz of water should be administered along with ipecac syrup to increase volume in
the stomach and facilitate vomiting. (C) Water alone will not induce vomiting. An emetic is
necessary to facilitate vomiting. (D) Vomiting should never be induced in an unconscious
client because of the risk of aspiration.

17. The nurse in the ER has reason to suspect that Lito is a victim of child abuse. The nurse’s
responsibility is to _________*

A. confront the parents.


B. call the supervisor.
C. notify the authorities.
D. investigate the cause.

18. Characteristics of parents who abuse their child/children include ___________.*

A. low socio-economic status


B. low educational level
C. product of abusive parents
D. large family

19. Which of the following signs, if manifested by a toddler, is a danger sign of battered child
syndrome?*

A. Negativistic and insists on getting what he wants


B. Irritable and throws a tantrum when frustrated
C. Does not cry when the mother leaves him behind on hospital admission
D. Slow in responding to commands

20. When discussing the mode of transmission of phenylketonuria (PKU) with parents, the nurse
can tell them that it is _________*

A. unknown
B. an x-linked recessive gene
C. an autosomal dominant gene
D. an autosomal recessive gene

21. Early screening and management of phenylketonuria is primarily to prevent the dreaded
complication of the disease, which is ________*

A. mental retardation
B. physical growth retardation
C. blindness
D. blond hair and blue eyes

22. To prevent complications of PKU, this formula is taken for about 6-8 years:*

A. lactose-free formula
B. lofenalac
C. fat-free formula (skim milk)
D. iron-fortified formula

23. Which of the following conditions does not require surgical management?*

A. Imperforate anus
B. Celiac disease
C. aganglionic megacolon
D. Intussusception

24. The nurse is reinforcing the teaching on how to prevent celiac crisis to the parents of a child
with celiac disease. The foods that are high in gluten and should be avoided included _________*

A. rice cake
B. corn flakes
C. breaded chicken and gravy
D. rice cereals

25. The result of the child’s intelligence quotient (IQ) test is 45. The nurse understands that the
result indicates which type of mental retardation and special consideration?*

A. severe retardation: needs marked supervision and dependent on others for care
B. profound retardation: incapable of self-care and requires custodial care
C. moderate retardation: can be independent with some self-help activities with
supervision and needs sheltered care
D. mild retardation: can achieve social and vocational skills with special education

26. In the care of children with mental retardation, which principle/(s) of care should be
considered?*

A. giving him all his needs


B. priority of care given on nutrition and safety
C. avoiding disciplining the child
D. all of the above

27. When obtaining history information from the parent of a 2-year-old boy with suspected mental
retardation, about which of the following would the nurse ask?*

A. period the child attained bladder training


B. age of weaning from the bottle
C. age at which the child is able to sit, stand and walk
D. number of words the child has spoken

28. Which of this medication is likely to be prescribed for a child who has attention deficit
hyperactive disorder? *

A. nardial
B. elavil
C. lithium
D. ritalin

29. The nursing care for Claire, a 4-year-old girl with severe autistic disorder, is most likely to
include _________*

A. Play therapy
B. Group therapy
C. Psychotropic drug therapy
D. Social skills training

30. Jane is a 3-year old child diagnosed with autism. The priority nursing goal when working with
an autistic child is to:*

A. Meet nutritional needs

B. Establish trust with the child

C. Maintenance contact with the family

D. Promote involvement in nursery school activities

QUIZ 1: GI Disorders
1. A 2-day-old infant was admitted for correction of hypertrophic pyloric stenosis. In assessing
the infant, which one of the following clinical manifestation is not consistent with the
diagnosis of pyloric stenosis?*
A. severe projectile vomiting after each feeding
B. fluid deficit, demonstrated by sunken fontanels, dry mucous membrane and scanty urine
C. sausage-shaped mass in the upper left quadrant
D. visible peristaltic waves passing left to right during and after feeding
2. To help prevent vomiting in an infant with pyloric stenosis which nursing action is
appropriate?*
A. Burf infant every after feeding
B. Place infant quite on his back after feeding
C. Feed with a thickened formula as prescribed
D. Feed with a diluted formula as indicated

3. A 5-day-old baby is admitted, with the chief complaint according to the mother as frequent
liquid stools. A thorough evaluation of the infant ruled out a congenital defect requiring
surgery. The baby has sepsis and is placed on antibiotic therapy. Which of the following
findings would best indicate to the nurse an improvement in the baby’s condition?
A. Dried umbilical cord
B. Increase in the strength of sucking
C. 6-8 wet diapers in 24-hours
D. Moist mucous membrane

4. To evaluate accurately the urine output of an infant having diarrhea, it is best to:*
A. Catheterize the infant aseptically
B. Weigh the infant’s wet diaper.
C. Instruct the mother on correct urine collection
D. Insert an indwelling catheter

5. Situation: Mrs. Go tells a nurse, “My 1-1⁄2 year-old child recently passed several stools that
are large, pale, diarrheic and have a food odor” Based on this information, the nurse
should suspect that child has developed _________*
A. Biliary atresia
B. Intussusceptions
C. Celiac disease
D. Hirschsprung’s disease

6. If pete gets admitted because of celiac crisis, which of the following actions should be done
to reverse the crisis?*
A. Strickly monitor of I & O
B. Implement IV therapy promptly
C. Avoid gluten-rich foods
D. Prevent infection.

7. To which of the following nursing diagnoses should the nurse give priority in the child’s
long-term care plan?*
A. Impaired skin integrity
B. Activity intolerance
C. Self-concept disturbance
D. High risk for ineffective family coping

8. The nurse is reinforcing the teaching on how to prevent celiac crisis to the parents of a child
with celiac disease. The foods that are high in gluten and should be avoided included
_________*
A. rice cake
B. corn flakes
C. breaded chicken and gravy
D. rice cereals

9. Situation: Fred, a 10-year-old boy with a tentative diagnosis of appendicitis is admitted to


the Pediatric Surgical Unit. Which of the following manifestations would be most important
for the nurse to follow-up?*

A. tympanic temperature of 39℃

B. anorexia and nausea

C. cessation of abdominal pain

D. generalized abdominal pain

10. On assessment, Fred is found to be febrile, with colicky abdominal pain and guarding of the
abdomen. Which of the following comfort measures would be taken until a diagnosis is made?*

A. Maintain the child in a semi-Fowler’s position.

B. Distract the child with an age-appropriate video.

C. Apply warm compresses to the affected area.

D. Obtain an order for an age-appropriate analgesic.

11. The mother of Fred needs to know why her son is not given a strong medication for pain relief.
The nurse’s response should reflect an understanding of which of the following?*
A. Pain medication may mask early symptoms of appendicitis.
B. Non-pharmacologic measures are started in the first 4 hours of the condition, then narcotics
are started.

C. Children cannot tolerate strong pain relievers.

D. Potent pain relievers are toxic to children

12. What are the 3 "C" early signs of esophageal atresia in neonate?*
A. choking, coughing, cyanosis

B. choking, crying, cyanosis

C. choking, comatose, cyanosis

D. choking, cramping, cyanosis

13. Tracheo-Esophageal Atresia & Fistula are common to fetus with diagnosis of
A. Polyhydramnios

B. hydramnios

C. Both A & B

D. None of the above

14. Tracheo-Esophageal Atresia & Fistula emergency surgery to prevent ________gastrostomy*


A. pneumonia

B. dehydration

C. Both A & B

D. None of the above

15. Which of the following is the manifestation of gastroesophageal reflux?*


A. Self-limiting, projectile, bile vomiting

B. Self-limiting, non-projectile, non-bile vomiting

C. both A & B

D. None of the above

16. Which of the following is the manifestation of dehydration in infant?*


A. Sunken fontanel, Rapid, thread pulse

B. Slow, thread pulse, Absence of tears

C. Scanty, concentrated urine, good skin turgor


D. Scanty, concentrated urine and afebrile

17. What is the first sign of dehydration in infants?*


A. Dry mucus, thirst

B. Sunken eyeballs

C. Lethargy, weakness

D. Oliguria

18. What is the assessment for pyloric stenosis?*


A. Sausage-shaped mass at the right upper quadrant

B. Olive-shaped mass at the left upper quadrant

C. Olive-shaped mass at the right upper quadrant

D. Sausage-shaped mass at the left upper quadrant

19. Which of the following interventions should not be included in the care plan for a
three-month-old infant who has just undergone cleft lip repair?*
A. Apply elbow restraints to the infants

B. Avoid placing the infant in the prone position

C. Clean the suture line after feeding the infant

D. Provide a pacifier to prevent the baby from crying

20. Baby Michael is a 4 month old infant with a tentative diagnosis of intussusceptions. Which
procedure will likely be ordered for the infant?*
A. Colonoscopy

B. Rectal biopsy

C. Barium enema

D. Cholangiography

21. The 8 month old infant of a neighbor sustained a minor scald burn. As a nurse, you will instruct
the parents to:*
A. Bring the child to the hospital right away

B. Apply alcohol to the area to prevent infection

C. Immerse the area in cool water

D. Cover the area with dry gauze


22. A three year old child has just been diagnosed with gastroenteritis. Which plan of care should
the nurse give priority?*
A. Provide sleep and rest

B. Preserve skin integrity

C. Promote hydration

D. Promote family coping

23. When assessing a child with a cleft palate, the nurse is aware that the child is at risk for more
frequent episodes of otitis media due to which of the following?*
A. Lowered resistance from malnutrition.

B. Ineffective functioning of the Eustachian tubes.

C. Plugging of the Eustachian tubes with food particles.

D. Associated congenital defects of the middle ear.

24. While assessing a newborn with cleft lip, the nurse would be alert that which of the following
will most likely be compromised?*
A. A. Sucking ability.

B. B. Respiratory status.

C. C. Locomotion.

D. D. GI function.

25. When providing postoperative care for the child with a cleft palate, the nurse should position
the child in which of the following positions?*
A. Supine.
B. Prone.
C. In an infant seat.
D. On the side.

26. An 18-month-old is scheduled for a cleft palate repair. The usual type of restraints for the child
with a cleft palate repair are:*
A. Elbow restraints.
B. Full arm restraints.
C. Wrist restraints.
D. Mummy restraints.
27. A home care nurse provides instructions to the mother of an infant with cleft palate regarding
feeding. Which statement if made by the mother indicates a need for further instructions?*
A. A. “I will use a nipple with a small hole to prevent choking.”

B. “I will stimulate sucking by rubbing the nipple on the lower lip.”

C. “I will allow the infant time to swallow.”

D. ”I will allow the infant to rest frequently to provide time for swallowing what has been placed
in the mouth.”

28. Imperforate anus manifestations are:


A. No meconium passed, B. Abdominal distension, C. negative wink reflex*
A. A & B

B. A,B, C

C. A &C

D. None of the above

29. The nurse is assessing an infant with Hirschsprung’s disease. The nurse can expect the infant
to:*
A. A. Weigh less than expected for height and age.

B. B. Have a scaphoid-shaped abdomen.

C. C. Exhibit clubbing of the fingers and toes.

D. D. Have hyperactive deep tendon reflexes.

30. A newborn’s failure to pass meconium within the first 24 hours after birth may indicate which of
the following?
A. A. Hirschsprung disease

B. B. Celiac disease

C. C. Intussusception

D. D. Abdominal wall defect

31. A 2-year-old is hospitalized with suspected intussusception. Which finding is associated with
intussusception?*
A. A. “Ribbon-like” stools

B. B. “Currant jelly” stools

C. C. Palpable mass over the flank

D. D. Projectile vomiting
32. A nurse is preparing to care for a child with a diagnosis of intussusception. The nurse reviews
the child’s record and expects to note which symptom of this disorder documented?*
A. A. Watery diarrhea

B. B. Ribbon-like stools

C. C. Profuse projectile vomiting

D. D. Bright red blood and mucus in the stools

33. The biggest single reason why children need a liver transplant is _________________*
A. Biliary Atresia

B. Hiatal Hernia

C. Diaphragmatic hernia

D. Gastroschisis

34. Which of the following is the protrusion of the abdominal contents (liver, intestine & stomach)
due to a defect in the closure of the umbilical ring*
A. Gastroschisis

B. Omphalocele

C. Intestinal Obstruction

D. Meconium Plug Syndrome

QUIZ 2: INTEGUMENTARY, PSYCHOSOCIAL


DISORDERS & PEDIATRIC INJURIES

1. What is the confirmatory test for scabies?*


● (+) SARCOPTES SCABIEI

2. Scabies classic sign is ________________ (commonly between fingers, on the wrist and trunk,
in the axillary folds) occurs at night due to the increased body heat that stimulates the parasite.*
● INTENSE PRURITUS
3. In scabies : ____________ is multiple, straight or wavy, fine, threadlike, grayish, brown, or black
lesions that may be difficult to see; most frequently seen between the fingers and on the wrists*
● SKIN BURROWS

4. Lindane cream can cause neurotoxicity and should not be used in children under 5 years old*
● TRUE
● FALSE
5. IMPETIGO CONTAGIOSA is highly contagious condition of the superficial layer of the skin
characterized by small, red macules that develop into vesicles, rupture and release serous fluid
and become covered with a loosely-adherent, _____________(color);
has_____________incubation period and infectious during the course of the infection*
● HONEY-YELLOW CRUSTS
● 7-10 DAYS

6. What is a non-infectious inflammatory dermatosis characterized by erythema,


papulovesiculation, oozing, crusting, and scaling; not a true disease but a reaction state of the
skin?*
● ATOPIC ECZEMA

7. The classification of burns based on extent of injury that uses the ____________________*
● RULE OF NINES

8. Which of the following is the most common complication of burns within the first 24 hours?*
● SHOCK

9. For Burns/Fire Health teaching to children, what should be demonstrated?*


● STOP, DROP, & ROLL

10. For Burn patients, needed to observe signs of respiratory distress: rales, wheezes, flaring,
stridor, tachypnea, dyspnea, and air hunger. *
● True
● False

11. In burn patient, signs of infection is characterized by: Systemic infection: fever, redness at the
wound site; offensive/foul odor, green-gray drainage Wound infection: fever and chills,
tachycardia, hyperemia.*
● True
● False

12. Which of the following nursing care that would limit or alleviate pain to a child with burns?*
A. Provide age-related diversional activities.
B. Determine special likes and dislikes food that will be served
C. Do not force eating. Do not use eating as a weapon
D. Provide a diet high in protein, calories, vitamins, and minerals

13. Classification of burns: epidermis, dermis m nerve, subcutaneous tissue and muscles*
A. 1st degree burn
B. Partial Thickness
C. 3rd degree burn
D. 2nd degree burn

14. Significant manifestations for 3nd degree burn:*


A. escobar
B. blister
C. edematous
D. Painful
15. BURNS : Estimating Body Surface Area (BSA) burned using the Rule of Nines : Adult head,
trunk and genital affected. What is the total percentage?*
● 19%

16. BURNS : Estimating Body Surface Area (BSA) burned using the Rule of Nines : Child trunk,
both legs and back affected. What is the total percentage?*
● 36%

17. Manifestation mildpoisoning include ___________, nausea, vomiting, diarrhea, sweating,


abdominal pain, delirium, dizziness and visual disturbance.*
● TINNITUS

18. First manifestation severe positioning ___________, *


● ACUTE LEAD COLIC

19. Poisoning treatment are _______________, activated charcoal and sodium bicarbonate.
● IPECAC SYRUP

20. Other term for lead poisoning*


● PLUMBISM

21. In lead poisoning, _____________ before onset of symptoms lessens the risk of
encephalophy*
● CHELATION THERAPY

22. Signs of child abuse are injuries, physical, emotional neglect and ____________________*
● BEHAVIORAL CHANGES

23. A child with IQ score is 83 and the level is categorized as _________________*


● BELOW AVERAGE

24. Which of the following is degree type of mental retardation with an assessment of IQ test of 24
and gross retardation?*
A. mild
B. moderate
C. profound
D. Severe
25. Trisomy 21 facial characteristics are wide gap between the eyes, _____________, Oriental
eyes, flat nose, large tongue*
● EPICANTHAL FOLDS

26. Trisomy 21 extremities are ______________ abnormal single horizontal line on the palm of the
hand; broad hands; plantar furrow: vertical line on the sole; first and second toes widely-spaced*
● SIMIAN CREASE

27. What is the hallmark of Down's syndrome?*

● HYPOTONIA

28. What is the etiology of autism?*


● NEUROPHYSIOLOGIC ORIGIN

29. What is the treatment for ADHD?*


● RITALIN

QUIZ 2: GI Disorders

1. Signs of phenylalanine deficiency: diarrhea, anorexia, lethargy, anemia and ____________


ANSWER: SKIN IS FAIR
2. What is the characteristic of the urine of a newborn with phenylketonuria?*
ANSWER: MUSTY ODOR URINE
3. What is the screening test of phenylketonuria newborn 2 to 4 days after birth?*
ANSWER: GUTHRIE CAPILLARY BLOOD TEST
4. What is the Phenylketonuria out-patient follow-up screening test effective only when the infant
is over two weeks old, when brain damage may have occurred?*
ANSWER: FERRIC CHLORIDE URINE TEST
5. What is a congenital absence (or fibrosis) of bile ducts, causing obstruction of bile flow. Bile may
be within the liver (intrahepatic) or in the main biliary system or bile passage (extrahepatic), the
most common site*
ANSWER: BILIARY ATRESIA

6. Which kind of jaundice is seen in Biliary atresia?*


ANSWER: PATHOLOGIC JAUNDICE

7. Describe the stool and urine of a biliary atresia newborn?*


ANSWERS: CLAY-COLORED STOOLS
DARK-COLORED URINE
8. What is the prescribed feeding with hiatal hernia?*
ANSWER: THICKENED FEEDING WITH CEREALS
9. What is the most common hiatal hernia?*
ANSWER: SLIDING HIATAL HERNIA
10. What is the assessment in the abdomen of diaphragmatic hernia?*

ANSWER: SCAPHOID ABDOMEN


11. What is the newborn condition of being prescribed saline enema and never using tap water to
prevent intoxication.*
ANSWER: MECONIUM PLUG SYNDROME
12. What is the common GI disorder of mothers with hydramnios?*
ANSWER: INTESTINAL OBSTRUCTION
13. Intussusception is a condition characterized by the _________________of the intestine along
any point of the intestinal tract (usually in the ileocecal valve), resulting in intestinal obstruction and
interference with the passage of intestinal contents*
ANSWER: TELESCOPING
14. Congenital aganglionic megacolon is common in boys with ______________________*
ANSWER: DOWN’S SYNDROME
15. What is the hallmark manifestation of Hirschsprung disease?*
ANSWER: CHRONIC CONSTIPATION
16. What is the description of Hirschsprung disease stools of infants?*
ANSWER: RIBBON-LIKE STOOLS
17. What do you call the contraction of the skin near the rectum if touched-sensory nerve ending
deficit in imperforate anus newborn?*
ANSWER: WINK REFLEX
18. What is the repair of cleft lip for 2 to 3 months infant?*
ANSWER: CHEILOPLASTY
19. What is the repair of cleft lip for toddlers?*
ANSWER: URANOPLASTY
20. What Congenital hypertrophy/hyperplasia of the muscle of the pylorus, causing obstruction of
the pyloric sphincter ?*
ANSWER: PYLORIC STENOSIS

21. The surgery of esophageal atresia is __________*


ANSWER: ESOPHAGEAL ANASTOMOSIS
22. Kwashiorkor manifestation ______________, sparse hair, moon face, flaky appearance of skin,
swollen abdomen and then muscles.*
ANSWER: DYSPIGMENTATION/FLAG SIGN? DEPOSED CHILD?
23. Marasamus assessment : normal hair, ___________appearance, thin libs and under wieght*
ANSWER: OLD MAN’S FACE
24. What is protein-calorie malnutrition (PCM) ?*
ANSWER: MARASMIC-KWASHIORKOR
25. Marasmus: more common in infants Kwashiorkor: more common in toddlers*
True
False
26. What is prescribed to prevent high levels of phenylalanine, which can cause mental
retardation*
ANSWER: LOFENALAC
27. What is the diet for Celiac Disease?*
ANSWER: GLUTEN-FREE DIET FOR LIFE
28. celiac crisis: _____________ and acidosis*

ANSWER: SEVERE DIARRHEA


29. Prevention of celiac crisis: Avoid gluten. _________.and avoid infection.*
ANSWER: AVOID PROLONGED FASTING
Quiz: VISION, HEARING & SPEECH
DISORDERS

1. A nurse is caring for a 2-year-old child who has had three ear infections in the past 5 months.
The nurse should know that the child is at risk for developing which of the following as a long-term
complication?
A. Balance difficulties
B. Prolonged hearing loss
C. Speech delays
D. Mastoiditis

2. Which of the following factors places the infant at risk for otitis media? (Select all that apply.)A.
Breastfeeding without formula supplementation. B. Attends day care 4 days per week. C.
Immunizations are up to date. D. History of cleft palate repair. E. Parents smoke cigarettes
outside.*
A. A,B,C
B. B,D,E
C. A,B,E
D. B.C.D

3. Which of the following findings are clinical manifestations of acute otitis media? (Select all that
apply.)A. Decreased pain in the supine position B. Rolling head side to side C. Loss of appetite D.
Increased sensitivity to sound E. Crying*

B,D,E

A,B,C

A,C,E

B,C,E

4. Which of the following instructions should Nurse Mia include in her teaching plan for the parents
of Allan with otitis media?*
A. Placing the child in the supine position to bottle-feed.
B. Giving prescribed amoxicillin (Amoxil) on an empty stomach.
C. Cleaning the inside of the ear canals with cotton swabs.
D. Avoiding contact with people who have upper respiratory tract infections.
5. Nurse Luisa is teaching a group of parents about otitis media. When discussing why children are
predisposed to this disorder, the nurse should mention the significance of which anatomical
feature?*
A. Nasopharynx.
B. Eustachian tubes.
C. External ear canal.
D. Tympanic membrane.

6. When assessing a child with a cleft palate, the nurse is aware that the child is at risk for more
frequent episodes of otitis media due to which of the following?*
A. Lowered resistance from malnutrition.
B. Ineffective functioning of the Eustachian tubes.
C. Plugging of the Eustachian tubes with food particles.
D. Associated congenital defects of the middle ear.

7. Nurse Bean is doing an assessment of a child with strabismus. Which of the following is
consistent with the said condition?*
A. Presbyopia

B. Myopia

C. Diplopia

D. Milky-white lens

8. At a well-child exam, a 3-year-old boy is diagnosed with strabismus. Which of the following
clinical manifestations does the nurse interpret as indicative of this disorder? Select all that apply:
A. Closing one eye to see B. Positioning self-close to the TVC. Squinting D. Difficulty doing close
work E. Excessive eye rubbing F. Tilting head to one side to see*
A,B,C
A,C,D
A,C,F
A,B,F

9. An important nursing intervention in the care of a child with conjunctivitis is:*


A. administering oral antihistamine to minimize itching

B. applying intermittent warm, moist compresses to remove crusts in the eye area
C. applying continuous warm compresses to relieve discomfort

D. administering optic corticosteroids to reduce inflammation

10. A pediatric client has been diagnosed with otitis media. The nurse should place highest priority
on teaching the parent:*
A. How to administer ophthalmic medication
B. About eliminating environmental allergens
C. About myringotomy and tympanostomy tube insertion
D. The importance of completing the full course of antibiotic therapy

11. A parent asks the nurse why her children get fewer ear infections as they grow older. The nurse
bases on her answer on which aspects of the infant's Eustachian tube?*
a. It is shorter, wider, and more horizontal than an older child's Eustachian tube.
b. It is shorter, wider, and more diagonal than an older child's Eustachian tube.
c. It is shorter, narrower, and more diagonal than an older child's Eustachian tube.
d. It is shorter, narrower, and more horizontal than an older child's Eustachian tube.

12. As the nurse performs auditory screening on an infant, the parents ask about potential causes
of hearing loss. The nurse reviews many causes but states which as the most common cause of
hearing loss?*
a. Aminoglycoside medication use
b. Genetic recessive inheritance
c. Mechanical ventilation
d. Bacterial meningitis

13. A limp 10-year-old boy is carried into the emergency department by a parent who states that
the child has a severe nosebleed. What is the priority action by the nurse?*
a. Administering oxygen.
b. Assessing for airway patency.
c. Obtaining a history.
d. Suctioning the blood.

14. During a hearing assessment, the nurse notes that the sound lateralizes to the clients left ear
with the Weber test. The nurse analyzes this result as:*
A) A normal finding
B) A conductive hearing loss in the right ear
C) A sensorineural or conductive loss
D) The presence of nystagmus

15. A 9-year-old client is being evaluated for a hearing impairment. Which question would be most
appropriate to provide the most useful information?*
A) "Are you having difficulty hearing high-frequency sounds?"
B) "Do you notice any drainage from your ears?"
C) "Are you experiencing any pain in your ears?"
D) "Have you felt any popping sensations?"
E) "Are you having difficulty hearing high-frequency sounds?"

16. Ophthalmia Neonatorum is the most comment infection of newborn with gonococcal infection
upon birth.*
True
False

17. In speech disorder of a 4 years old, a nurse check on the criteria for screening and evaluation
for speech and language disorder except for:*
A. If the child does not say single words at 18 months
B. If the child does not use word combinations at 2 years
C. If the child does not utter simple, intelligible sentences at 3 years
D. If the child does not communicate looking at the eyes of parents at 2 years

18. To evaluate the dangers sign of problem in language and speech development , the nurse will
record a danger sign if the child at 18 months could communicate 10 -word vocabulary.*
True
False

19. Which of the following is the danger sign of 3 years old manifest with possible problem in
language and speech development?*
A. does not turn eyes to sound coming from the sides or behind
B. Does not respond to bye-bye or no-no
C. Does not begun to ask simple questions or even talk simple sentences
D. Does not have 10-word vocabulary

20. To evaluate deafness to an infant , perform early check of hearing: Elicit the Moro reflex with a
loud banging sound.*
True

Quiz 1: Postpartum Complications

1. Rubin identified a series of changes that a new mother makes during the postpartum
period. The correct sequence of these changes is*
a) Taking-in, taking-hold, letting-go
b) Taking, holding-on, letting-go
c) Taking-in, holding-on, letting-go
d) Taking-in, taking-on, letting-go

2. A nurse is monitoring the vital signs of a client 24 hours after childbirth. She notes that the
client's blood pressure is 100/60 mm Hg. Which of the following postpartum complications
should the nurse most suspect in this client, based on this finding?*
a) Postpartal gestational hypertension
b) Bleeding
c) Diabetes
d) Infection

3. The process by which the reproductive organs return to the nonpregnant size and function
is termed :*
a) Evolution
b) Involution
c) Decrement
d) Progression
4. The nurse is caring for a client in the postpartum period. The client has difficulty in voiding
and is catheterized. The nurse then would monitor the client for which of the following?*
a) Loss of pelvic muscle tone
b) Stress incontinence
c) Urinary tract infection
d) Increased urine output

5. On assessment of a client who is 30 minutes into the fourth stage of labor, the nurse finds
the client's perineal pad saturated with blood and blood soaked into the bed linen under the
client's buttocks. The nurse's initial action is which of the following.*
A. Call the physician
B. Assess the client's vital signs
C. Gently massage the uterine fundus
D. Administer a 300ml bolus of a 20 units/L Oxytocin(Pitocin) solution

6. A second-day postpartum client with diabetes mellitus has scant lochia with a foul odor and
a temperature of 101.6 degrees F. The physician suspects infection and writes orders to
treat the client. Which of the following orders written by the physician would the nurse
complete first?*
A. Obtain culture and sensitivity of lochia and urine
B. Administer Ceftriaxone (Rocephin)
C. Check the client's temperature
D. Increase the intake of oral fluids.

7. A postpartum nurse is taking the vital signs of a woman who delivered a healthy newborn
infant 4 hours ago. The nurse notes that the mother's temperature is 100.2*F. Which of the
following actions would be most appropriate?*
A) Retake the temperature in 15 minutes
B) Notify the physician
C) Document the findings
D) Increase hydration by encouraging oral fluids

8. The nurse is assessing a client who is 6 hours PP after delivering a full-term healthy infant.
The client complains to the nurse of feelings of faintness and dizziness. Which of the
following nursing actions would be most appropriate?*
A) Obtain hemoglobin and hematocrit levels
B) Instruct the mother to request help when getting out of bed
C) Elevate the mother's legs
D) Inform the nursery room nurse to avoid bringing the newborn infant to the mother until the
feelings of lightheadedness and dizziness have subsided
9. A nurse is preparing to perform a fundal assessment on a postpartum client. The initial
nursing action in performing this assessment is which of the following?*
A) Ask the client to turn on her side
B) Ask the client to lie flat on her back with the knees and legs flat and straight
C) Ask the mother to urinate and empty her bladder
D) Massage the fundus gently before determining the level of the fundus.

10. The nurse is assessing the lochia on a 1 day PP patient. The nurse notes that the lochia is
red and has a foul-smelling odor. The nurse determines that this assessment finding is:*
A) Normal
B) Indicates the presence of infection
C) Indicates the need for increasing oral fluids
D) Indicates the need for increasing ambulation

11. A nurse in a PP unit is instructing a mother regarding lochia and the amount of expected
lochia drainage. The nurse instructs the mother that the normal amount of lochia may vary
but should never exceed the need for:*
A) One peripad per day
B) Two peripads per day
C) Three peripads per day
D) Eight peripads per day

12. A PP nurse is providing instructions to a woman after delivery of a healthy newborn infant.
The nurse instructs the mother that she should expect normal bowel elimination to return:*
A) One the day of the delivery
B) 3 days PP
C) 7 days PP
D) within 2 weeks PP

13. The perinatal nurse is caring for a woman in the immediate postbirth period. Assessment
reveals that the woman is experiencing profuse bleeding. The most likely etiology for the
bleeding is:*
A. Uterine atony
B. Uterine inversion
C. Vaginal hematoma
D. Vaginal laceration

14. A primary nursing responsibility when caring for a woman experiencing an obstetric
hemorrhage associated with uterine atony is to*
A. Establish venous access
B. Perform fundal massage
C. Prepare the woman for surgical intervention
D. Catheterize the bladder

15. The perinatal nurse caring for the postpartum woman understands that late postpartum
hemorrhage is most likely caused by:*
A. Subinvolution of the placental site
B. Defective vascularity of the decidua
C. Cervical lacerations
D. Coagulation disorders

16. What woman is at greatest risk for early postpartum hemorrhage?*


A. A primiparous woman (G 2 P 1 0 0 1) being prepared for an emergency cesarean birth for fetal
distress
B. A woman with severe preeclampsia on magnesium sulfate whose labor is being
induced
C. A multiparous woman (G 3 P 2 0 0 2) with an 8-hour labor
D. A primigravida in spontaneous labor with preterm twins

17. The first and most important nursing intervention when a nurse observes profuse
postpartum bleeding is to:*
A. Call the woman's primary health care provider
B. Administer the standing order for an oxytocic
C. Palpate the uterus and massage it if it is boggy
D. Assess maternal blood pressure and pulse for signs of hypovolemic shock

18. When caring for a postpartum woman experiencing hemorrhagic shock, the nurse
recognizes that the most objective and least invasive assessment of adequate organ
perfusion and oxygenation is:*
A. Absence of cyanosis in the buccal mucosa
B. Cool, dry skin
C. Diminished restlessness
D. Urinary output of at least 30 ml/hr
19. The most effective and least expensive treatment of puerperal infection is prevention. What
is important in this strategy?*
A. Large doses of vitamin C during pregnancy
B. Prophylactic antibiotics
C. Strict aseptic technique, including handwashing, by all health care personnel
D. Limited protein and fat intake

20. The perinatal nurse assisting with establishing lactation is aware that acute mastitis can be
minimized by:*
A. Washing the nipples and breasts with mild soap and water once a day
B. Using proper breastfeeding techniques
C. Wearing a nipple shield for the first few days of breastfeeding
D. Wearing a supportive bra 24 hours a day

21. The clinic nurse is caring for a woman who is suspected of developing postpartum
psychosis. Which of the following statements characterizes this disorder:*
A. Symptoms start within several days of delivery
B. The disorder is common in postpartum women
C. Suicide and infanticide are uncommon in this disorder
D. Delusions and hallucinations accompany this disorder

22. The nurse is providing instructions about measures to prevent postpartum mastitis to a
client who is breast-feeding her newborn. Which client statement would indicate a need for
further instruction?*
A. "I should breast-feed every 2 to 3 hours."
B. "I should change the breast pads frequently."
C. "I should wash my hands well before breast- feeding."
D. "I should wash my nipples daily with soap and water."

23. The nurse notes that the client is tachycardic and the respiratory rate is elevated. The
nurse suspects a pulmonary embolism. Which should be the initial nursing action?*
A. Initiate an intravenous line
B. Prepare to administer morphine sulfate.
C. Administer oxygen, 8 to 10 L/ minute, by face mask.
D. Assess the client's blood pressure.

24. On the third day post-partum, Lorine's breast became full and the breast appeared
inflamed. She complained of a feeling of tension in the breast. The condition is termed:*
a. Let-down reflex
b. Mastitis
c. Engorgement
d. Milk fever

25. Lita complains of intermittent afterbirth pains. The nurse can best help her if the nurse does
all of the following except:*
a. Applies a hot water bag
b. Gives analgesics as ordered PRN
c. Explains to the client that these are normal
d. Applies an ice cap

26. Charm asked the nurse within how many days will the lochia alba last. The correct reply of
the nurse is:*
a. 10
b. 7
c. 8
d. 6

27. Due to prolonged labor and trauma, it is the best to provide comfort and prevent
complication immediately after delivery by:*
a. Administering hot sitz bath
b. Giving analgesics
c. Explains to the client that these are normal
d. Applying ice cap over the perineum

28. Absence of lochia during puerperium may mean:*


a. Uterine atony
b. New pregnancy
c. Hemorrhage
d. Infection
29. Aileen, a 38 year old multipara, is admitted with a tentative diagnosis of femoral
thrombophlebitis. The nurses assesses the patient with:*
a. Burning on urination
b. Leg pain
c. Abdominal pain
d. Increased lochial flow

30. Fever, foul lochial discharge and subinvolution of the uterus are the signs of:*
a. Puerperial psychosis
b. Puerperial sepsis
c. Postpartum hemorrhage
d. Hypertensive disorders

Quiz 2: VISION, HEARING & SPEECH


DISORDERS

1. What is the most frequent gonococcal infection in the newborn?


ANSWER: OPHTHALMIA NEONATORUM

2. What are the 2 diagnostic procedures of evaluating strabismus?*


ANSWER: CORNEAL LIGHT REFLEX AND COVER EYE TEST

3. Strabismus is high in infants up to five months and beyond five months, strabismus requires
evaluation and treatment.*
A. True
B. False

4. Instill the prescribed eye ointment to a child from _________*


ANSWER: INNER CANTHUS TO OUTER CANTHUS

5. What is the one of the most common illnesses in infancy and early childhood and most
prevalent childhood disease after respiratory tract infections?*
ANSWER: MENINGITIS

6. For administering otic medications: for infants and toddler is ______________*


ANSWER: OINTMENT
7. The 2 complications of acute otitis media are conductive hearing loss and
___________________

ANSWER: MENINGITIS

8. It is the caused by interruption in the passage of sound waves from the external ear through the
middle ear*
ANSWER: CONDUCTIVE HEARING LOSS

9. It is a middle ear loss with interference of transmission along neural pathways.*


ANSWER: MIXED HEARING LOSS

10. Acute Otitis Media precipitating factor for hereditary is ____________*


ANSWER: HEARING LOSS

11. infant manifestation for hearing impairment is NO ________ from loud noise but other reflexes
are normal/active*
ANSWER: MORO REFLEX

12. After surgery of strabismus pedia client: Teach child to prevent ______________,
_____________, rubbing of eyes, and sudden head movement.*
ANSWER: COUGHING, STRAINING

13. True or False: Criteria for screening and evaluation for speech and language disorder: If the
child does not say single words at 9 months If the child does not use word combinations at 2 years
If the child does not utter simple, intelligible sentences at 5 years.*
ANSWER: FALSE

14. True or False for Acute otitis media nursing care: Relieve pain and promote comfort. Apply
local external heat: lie with the unaffected ear down on a heating pad wrapped in a towel; observe
safety precautions. Avoid foods that cause difficulty swallowing; provide a soothing liquid diet.
Administer analgesic drug (e.g., acetaminophen) as ordered.*
A. True
B. False
15. True or False in Strabismus surgical treatment is done to correct iris and lens to better their
alignment*

A. True
B. False

Quiz 2: Postpartum Complications


1. ___________ and ____________ are associated with early post partum hemorrhage.

ANSWER: UTERINE ATONY & VAGINA AND CERVICAL TEAR

2. ___________palpable above the umbilicus, soft and poorly contracted

ANSWER: BOGGY UTERUS

3. Over _____________ is the estimated mayor PPH blood loss*

ANSWER: 1000 ML

4. _____________ PPH is defined as abnormal bleeding from the genital tract, from 24 hours
after delivery until six weeks postpartum.*

ANSWER: SECONDARY PPH

5. One of the causes of Postpartum Hemorrhage is _________ which is the uterine atony and
distended bladder.*

ANSWER: UTERINE ATONY

6. Trauma is a common cause of post partum hemorrhage. identify the 3 areas injured.*
ANSWER: UTERINE INJURY, CERVICAL INJURY AND VAGINAL INJURY

7. Technique of ________ for uterine atony*


ANSWER: BIMANUAL MASSAGE

8. Bubble He Postpartum assessment : Breast is to assess ________________ vs mastitis*

ANSWER: ENGORGEMENT

9. Bubble He Postpartum assessment : Uterus is to assess ________________ vs uterine


atony*

ANSWER: CONTRACTED UTERUS

10. Physiologic assessment, _____________ is normal physiologic change for 6-10 days
postpartum.*
ANSWER: BRADYCARDIA

11. What is the common rationale for 38'C temperature after delivery of an infant?*

ANSWER: DEHYDRATION
12. Non-nursing woman-tight bra for 72 hours , _____________ and minimizes breast
stimulations for postpartum nursing care,*

ANSWER: ICE PACKS

13. What health teaching to postpartum mothers to prevent thrombophlebitis and paralytic
ileus.*

ANSWER: ENCOURAGE AMBULATION

14. ______ (word not number) degree of episiotomy affects the perineal skin, muscles and
external anal sphincter*
ANSWER: THIRD DEGREE

15. Nursing care for perineal hematoma or discomfort: apply ___________to the perineum
during the first 24 hours to reduce the swelling.*

ANSWER: ICE PACKS

16. ______________is positive Homan's sign with deep vein thrombosis manifestation of with
dyspnea, pleuritic chest pain, hemoptysis , hypertension and dypnea.*
ANSWER: THROMBOPHLEBITIS

17. It is delayed return of the enlarged uterus to normal size and function.*
ANSWER: SUBINVOLUTION

18. Fundal height decreases about ___________ (1 cm) per day By 10 days postpartum,
uterus cannot be palpated abdominally*
ANSWER: 1 FINGERBREADTH

19. True or False : Assessments for subinvolution of the uterus include: Fundal height by
palpation and fingerbreadth measurement from the level of the umbilicus. Fundal position
and consistency. Lochia.*
A. True
B. False
20. Sheehan syndrome is a condition that can occur in a woman who ______________ during
childbirth. It affects women who lose a life-threatening amount of blood or who have severe
low blood pressure during or after childbirth.
ANSWER: PITUITARY GLAND IS DAMAGED

21. It is post partum disorders : 2 weeks or less with fatigue, weeping anxiety and mood
instability*
ANSWER: POSTPARTUM DEPRESSION
22. It is post partum disorders : 23-5 days post partum or deppresion plus delusions,
hallucinations and hyperactivity.*
ANSWER: POSTPARTUM PSYCHOSIS

23. Rubin's Post partum phases that mother is passive and dependent.*
ANSWER: TAKING IN PHASE

24. Rubin's Post partum phases that mother is on mood swing and maximal stage of learning
readiness*
ANSWER: TAKING HOLD PHASE

25. Encourage the client to utilize factors that can enhance uterine involution : Breastfeeding,
early and frequent ambulation and regular bowel movement.*

A. True B. False

Quiz : Female and Male Reproductive Disorders


1. Which pt statement alerts the nurse to a warning sign of cancer?
A. "I have a BM every 4 days"
B. I had a wart on my finger that lasted for 2 weeks"
C. "I have a blister on my toe that never seems to get well"
D. I experience moderately heavy bleeding with my periods every month"
Rationale: A sore that does not heal is a warning sign of cancer. A bowel movement every 4 days
does not indicate any change in bowel habits. Warts often last for several weeks, and the patient
did not describe any change in appearance. Moderately heavy bleeding in conjunction with
menstruation is normal and not unusual.

2. The nurse is caring for a pt who requires compromised host precautions. Which action is
most important for the nurse to take before delivering the pt's breakfast tray?*
A. Ask pt if she feels like eating at this time
B. Remove fresh apple and orange from tray
C. Call dietary department and ask for disposable utensils
D. Declutter room before assisting pt to to couch to eat
Rationale: The patient is at a significantly increased risk for infection. Fresh produce can harbor
bacteria that can lead to infection, so the apple and orange should be removed from the tray.
Assessing the quality of the patient's appetite does not protect against infection. Although
disposable utensils are not harmful, sterilized standard utensils do not pose an overt risk for
infection. Limiting clutter in the room is an action that would protect the patient with a low platelet
count in case of a fall but would not decrease infection risk.
3. A nurse is obtaining a health history on a pt who was admitted with lung cancer. Which
question is most important for the nurse to ask to gather data about the chief complaint?*
A. "How often, if ever, do you drink alcohol"
B. "Do you see a primary care health provider regularly?"
C. "Do you have any history of serious diseases or cancer in your immediate family?"
D. "Can you tell me more about the problem that caused you to come to the hospital?"
Rationale: The chief complaint is the patient's main complaint or concern, and the nurse should
seek a complete description of the problem with related signs and symptoms. Asking about alcohol
consumption and medical checkups are components of a functional assessment. Incidence of
familial diseases is part of the family history.

4. A 58-year-old male patient is being seen by the health care provider for an annual physical.
According to the American Cancer Society, which cancer screenings should be
recommended every year for asymptomatic men? (Select all that apply.)
A. A fecal occult blood stool test
B. A prostate-specific antigen test
C. A Papanicolaou test
D. A digital rectal examination
E. A pelvic examination
F. A colonoscopy
Rationale: Evidence supports that proactive and timely cancer screening reduces the likelihood of
developing cancer. Cancer diagnosed at a lower stage has a greater likelihood of cure. A fecal
occult blood stool test and a prostate-specific antigen test are recommended annually. The digital
rectal examination is recommended annually. The Papanicolaou test and pelvic examination are
tests for women's health. A colonoscopy is recommended every 10 years for low-risk patients.

5. Which of these does the nurse recognize as the goal of palliative surgery for the client with
cancer?*
A. Cure of the cancer
B. Relief of symptoms or improved quality of life
C. Allowing other therapies to be more effective
D. Prolonging the client's survival time

6. When caring for a client receiving chemotherapy, the nurse plans care during the nadir of
bone marrow activity to prevent which complication?*
A Drug toxicity
B Polycythemia
C Infection
D Dose-limiting side effects
Rationale: The lowest point of bone marrow function is referred to as the nadir; risk for infection is
highest during this phase. Drug toxicity can develop when drug levels exceed peak
concentrations. Polycythemia refers to an increase in the number of red blood cells; typically
chemotherapy causes reduction of red blood cells or anemia. Dose limiting side effects occur
when the dose or frequency of chemotherapy need to be altered or held, such as in the case of
severe neutropenia or neurologic dysfunction .

7. Enlargement of the prostate gland, BPH, is usually associated with:*


A. Dysuria.
B. Dilation of the ureters.
C. Hydronephrosis.
D. All of the above.

8. The following are surgical procedures used in BPH except:*


A. Prostatectomy.
B. TURP.
C. TUNA.
D. Circumcision.

9. A result of the digital rectal examination in a patient with BPH includes what findings?*
A. Enlarged, tender prostate.
B. Large, rubbery prostate.
C. Small, nontender prostate.
D. Pus-covered prostate.

10. What is the surgical removal of the inner portion of the prostate through an endoscope
inserted through the urethra?*
A. Open prostatectomy.
B. TUNA.
C. DRE.
D. TURP.

11. The nurse assesses a client with dimpling of the skin in the left upper outer quadrant of the
right breast and nipple discharge. Which question should the nurse ask to learn if the client
is experiencing other clinical manifestations of breast​cancer?*
A. "Have you been experiencing any night sweats or​low-grade fevers?"
B. ​"Does the skin in the left upper outer quadrant of the right breast feel warm to​touch?"
C. ​"Have you noticed any itching around the nipple or in the area affected by​dimpling?"
D. ​"Have you noticed a rash or skin irritation around the nipple​area?"
12. The nurse is caring for a patient diagnosed with breast cancer who just underwent an
axillary lymph node dissection. What intervention should the nurse use to decrease the
lymphedema?*
A. Keep the affected arm flat at the patient's side.
B. Apply an elastic bandage on the affected arm.
C. Assess blood pressure only on unaffected arm.
D. Restrict exercise of the affected arm for 1 week.

13. A 72-year-old patient who had a mastectomy for breast cancer 6 months ago wants to have
breast reconstructive surgery. The nurse knows that what is the most likely motivation for
this patient seeking this surgery?*
A. Improve the woman's self-image
B. Be able to experience sexual arousal
C. To get a tummy tuck as well as the breast mound
D. Restore the pre-mastectomy appearance of the breast

14. The nurse teaches a 53-year-old patient about screening for early detection of breast
cancer. Which statement by the patient requires an intervention by the nurse?*
A. "I should plan to have a mammogram every year."
B. "I will see a health care provider every year for a breast examination."
C. "A breast examination should be done right after my menstrual period."
D. "Self-breast examination is a reliable way to detect breast cancer early."

15. It is the collection of fluid between the visceral and parietal layers of the tunica vaginalis of
the testicles or along spermatic cord.*
● HYDROCELE

16. It is a mass of dilated and tortous varicose veins in the spermatic cord and classically called
as Bag of worms.*
● VARICOCELE

17. What is the meaning of TNA diagnosis of Breast Cancer,?*


● TUMOR, LYMPH NODE, METASTASIS

18. True of False: Cystocele occurs when support between the vagina and bladder is
weakened. The pelvic organs are held in place by muscles and tissues that can sometimes
weaken and stretch.*

True
False

19. True or False: Rectocele results from weakening between the vagina and rectum. Woman
may not be able to empty bladder or bowel*
True
False

20. Symptoms of Cancer : N is for __________________*

LYMPH NODES ARE ENLARGED

21. Symptoms of Cancer : T is for __________________*

TUMOR SIZE GREATER THAN 4CM

UNIT EXAMINATION (FINALS)- MATERNAL


& CHILD HIGH RISK

1. A client is admitted in labor with spontaneous ROM 24hr earlier. The fluid is clear and the
FHR is 124 w/ moderate variability. Which assessment is most important for the nurse to
make at this time?
C) Cervical Dilation & effacement
4) maternal pulse rate
B) Maternal Temperature
A) Contraction frequency and duration

2. A woman is 6cm dilated. The fetal monitor tracing shows recurring deep late decelerations.
The woman's doctor informs her that the baby must be delivered by C-sect. The woman
refuses to sign the informed consent. Which of the following actions by the RN is
appropriate?
A) strongly encourage the woman to sign consent
B) prepare the woman for c-sect
C) inform the woman that the baby will likely die w/out surgery
D) provide the woman with ongoing labor support

3. A nurse is monitoring a client in active labor and notes that the client is having contractions
every 3 minutes that last 45 seconds. The nurse notes that the fetal heart rate between
contractions is 100 BPM. Which of the following nursing actions is most appropriate?
A. Encourage the client’s coach to continue to encourage breathing exercises.
B. Encourage the client to continue pushing with each contraction.
C. Continue monitoring the fetal heart rate.
D. Notify the physician or nurse-midwife.

Correct Answer: D. Notify the physician or nurse-midwife. A normal fetal heart rate is 120-160
beats per minute. Fetal bradycardia between contractions may indicate the need for immediate
medical management, and the physician or nurse-midwife needs to be notified.

4. Four hours after a difficult labor and birth, a primiparous woman refuses to feed her baby,
stating that she is too tired and just wants to sleep. The nurse should:
A. Tell the woman she can rest after she feeds her baby.
B. Recognize this as a behavior of the taking-hold stage.
C. Record the behavior as ineffective maternal-newborn attachment.
D. Take the baby back to the nursery, reassuring the woman that her rest is a priority at
this time.

Correct Answer: D. Take the baby back to the nursery, reassuring the woman that her rest is a
priority at this time. The behavior described is typical of this stage and not a reflection of ineffective
attachment unless the behavior persists. Mothers need to reestablish their own well-being in order
to effectively care for their baby.

5. A client arrives at a birthing center in active labor. Her membranes are still intact, and the
nurse-midwife prepares to perform an amniotomy. A nurse who is assisting the
nurse-midwife explains to the client that after this procedure, she will most likely have:
A. Less pressure on her cervix.
B. Increased efficiency of contractions.
C. Decreased number of contractions.
D. The need for increased maternal blood pressure monitoring.

6. A maternity nurse is preparing to care for a pregnant client in labor who will be delivering
twins. The nurse monitors the fetal heart rates by placing the external fetal monitor:
A. Over the fetus that is most anterior to the mother’s abdomen.
B. Over the fetus that is most posterior to the mother’s abdomen.
C. So that each fetal heart rate is monitored separately.
D. So that one fetus is monitored for a 15-minute period followed by a 15 minute fetal monitoring
period for the second fetus.
Correct Answer: C. So that each fetal heart rate is monitored separately. In a client with a multifetal
pregnancy, each fetal heart rate is monitored separately. Simultaneous monitoring of twins is
preferable to non simultaneous monitoring to discriminate between their separate FHRs (ACOG,
1989). Synchronizing the internal clocks of both monitors will help produce accurate
documentation. Otherwise, time increments should be documented on both monitor tracings for
later comparison, to ensure that each twin has been monitored. If the monitor strips are
synchronous, portable real-time ultrasound can be used to verify that both twins are being
monitored independently
7. A maternity nurse is caring for a client with abruptio placenta and is monitoring the client for
disseminated intravascular coagulopathy. Which assessment finding is least likely to be
associated with disseminated intravascular coagulation?
A. Swelling of the calf in one leg
B. Prolonged clotting times
C. Decreased platelet count
D. Petechiae, oozing from injection sites, and hematuria

Correct Answer: A. Swelling of the calf in one leg DIC is a state of diffuse clotting in which clotting
factors are consumed, leading to widespread bleeding. Swelling and pain in the calf of one leg are
more likely to be associated with thrombophlebitis.

8. The nurse is observing children playing in the hospital playroom. She would expect to see 4
year-old children playing
A. Competitive board games with older children
B. With their own toys alongside with other children
C. Alone with hand held computer games

D. Cooperatively with other preschoolers

9. The parents of a 2-year-old arrive at a hospital to visit their child. The child is in the
playroom when the parents arrive. When the parents enter the playroom, the child does
not readily approach the parents. The nurse interprets this behavior as indicating that:
A. The child is withdrawn
B. The child is self-centered
C. The child has adjusted to the hospitalized setting
D. This is a normal pattern

10. A 16-year-old is admitted to the hospital for acute appendicitis, and an appendectomy is
performed. Which of the following nursing interventions is most appropriate to facilitate
normal growth and development?
A. Allow the family to bring in the child’s favorite computer games
B. Encourage the parents to room-in with the child
C. Encourage the child to rest and read
D. Allow the child to participate in activities with other individuals in the same age group
when the condition permits

11. A 16 year old child is hospitalized, according to Erik Erikson, what is an appropriate
intervention?
A. tell the friends to visit the child
B. encourage patient to help child learn lessons missed
C. call the priest to intervene
D. tell the child’s girlfriend to visit the child.

12. A 16 year old child is hospitalized, according to Erik Erikson, what is an appropriate
intervention?
A. tell the friends to visit the child
B. encourage patient to help child learn lessons missed
C. call the priest to intervene
D. tell the child’s girlfriend to visit the child.

13. Children as young as age 3 years can use facial scales for discrimination. What are some
suggested anchor words for the preschool age group?
A. “No hurt.”
B. “Red pain.”
C. “Zero hurt.”
D. “Least pain.”

14. The nursing student is preparing to teach a prenatal class about fetal circulation. Which
statement should be included in the teaching plan?
a. “One artery carries oxygenated blood from the placenta to the fetus”
b. “Two arteries carry oxygenated blood from the placenta to the fetus”
c. “Two arteries carry deoxygenated blood and waste products away from fetus”
d. “Two veins carry blood that is high in carbon dioxide and other waste products away from fetus
to the placenta.”

Answer C. Blood pumped by the embryo’s heart leaves the embryo through two umbilical arteries.
When oxygenated, the blood is returned by one umbilical vein. Arteries carry deoxygenated blood
and waste products from the fetus, and then umbilical vein carries oxygenated blood and provides
oxygen and nutrients.
15. You’re providing an in-service to a group of new nurses who will be caring for patients who
have Tetralogy of Fallot. Which statement below is INCORRECT concerning how the blood
normally flows through the heart?
a. Unoxygenated blood enters through the superior and inferior vena cava and travels to
the left atrium.
b. The pulmonic valve receives blood from the right ventricle and allows blood to flow to the lungs
via the pulmonary artery.
c. The left atrium allows blood to flow down through the bicuspid valve (mitral) into the left
ventricle.
d. Oxygenated blood leaves the left ventricle and flows up through the aortic valve and aorta to
be pumped to the rest of the body.

Answer is A. This statement is INCORRECT. It should say: “Unoxygenated blood enters through
the superior and inferior vena cava to the RIGHT (not left) atrium.

16. While feeding a 3-month-old infant, who has Tetralogy of Fallot, you notice the infant’s skin
begins to have a bluish tint and the breathing rate has increased. Your immediate nursing?
a. Continue feeding the infant and place the infant on oxygen.
b. Stop feeding the infant and provide suction.
c. Stop feeding the infant and place the infant in the knee-to-chest position and administer
oxygen.
d. Assess the infant’s heart rate and rhythm.

Answer is C. The patient is experiencing a “tet spell”. This is where during any type of activity like
feeding, crying, playing etc. the child’s heart (due to Tetralogy of Fallot) is unable to maintain
proper oxygen levels in the blood (these activities place extra work on the heart and it can’t keep
up). Therefore, there are low amounts of oxygen in the blood, and the skin will become cyanotic
(bluish tint) and the respiratory rate will increase (this is the body’s way of trying to increase the
oxygen levels in the body but it doesn’t work because it’s not a gas exchange problem in the lungs
but a heart problem). The nurse would want to place the infant in the knee-to-chest position. WHY?
This increases systemic vascular resistance (which will help decrease the right to left shunt that is
occurring in the heart…hence helps replenish the body with oxygenated blood). In addition, the
nurse would want to place the patient on oxygen.

17. You’re caring for a 2-year-old patient who has a large atrial septal defect that needs repair.
This defect is causing complications. These complications are arising from an abnormal
shunting of blood throughout the heart. As the nurse, you know that a
__________________ shunt is occurring in the heart due to the defect.
a. Right-to-left
b. Right
c. Left
d. Left-to-right

The answer is D. A left-to-right shunt is occurring because the pressure in the left side of the heart
is higher than the right, which allows blood to easily flow into the right side through the hole in the
interatrial septum. This will cause pulmonary hypertension and heart failure overtime.
Select the structure below that allows blood to flow from the right to left atrium in utero and that
should close after birth:

a. Ductus Arteriosus
b. Formen Ovale
c. Ductus Venosus
d. Ligamentum teres

The answer is B. The formen ovale allows this to occur.

18. True or False: Atrial septal defects can lead to a decrease in lung blood flow
True
False

Answer is FALSE: ASDs can lead to an INCREASE in lung blood flow (not decrease).

19. The nurse is assessing a six-month-old child. Which developmental skills are normal and
should be expected?
Speaks in short sentences.
Sits alone.
Can feed self with a spoon.
Pulling up to a standing position.

20. The nurse prepares for a Denver Screening test with a 3 year-old child in the clinic. The
mother asks the nurse to explain the purpose of the test. The BEST response is to tell her
that the test
A. Measures potential intelligence
B. Assesses a child’s development
C. Evaluates psychological responses
D. Diagnoses specific problems

21. The nurse is assessing a four month-old infant. The nurse would anticipate finding that the
infant would be able to
A. Hold a rattle
B. Bang two blocks
C. Drink from a cup
D. Wave “bye-bye”

22. Which age group would have a tendency towards eating disorders?
A. Adolescence
B. Toddler hood
C. Childhood
D. Infancy

23. A mother of a 3-year-old tells a clinic nurse that the child is rebelling constantly and having
temper tantrums. The nurse most appropriately tells the mother to:
A. Punish the child every time the child says “no”, to change the behavior
B. Allow the behavior because this is normal at this age period
C. Set limits on the child’s behavior
D. Ignore the child when this behavior occurs

24. A nurse is evaluating the developmental level of a 2-year-old. Which of the following does
the nurse expect to observe in this child?
A. Uses a fork to eat
B. Uses a cup to drink
C. Uses a knife for cutting food
D. Pours own milk into a cup

25. Which of the following actions is NOT appropriate in the care of a 2-month-old infant?
A. Place the infant on her back for naps and bedtime.
B. Allow the infant to cry for 5 minutes before responding if she wakes during the night as
she may fall back asleep.
C. Talk to the infant frequently and make eye contact to encourage language development.
D. Wait until at least 4 months to add infant cereals and strained fruits to the diet.

Answer: B. Allow the infant to cry for 5 minutes before responding if she wakes during the night as
she may fall back asleep.

Option B: Infants under 6 months may not be able to sleep for long periods because their
stomachs are too small to hold adequate nourishment to take them through the night.

Option A: Infants should always be placed on their backs to sleep. Research has shown a dramatic
decrease in sudden infant death syndrome (SIDS) with back sleeping.

Option C: Eye contact and verbal engagement with infants are important to language development.

Option D: The best diet for infants under 4 months of age is breast milk or infant formula.

26. Sudden infant death syndrome (SIDS) is one of the most common causes of death in
infants. At what age is the diagnosis of SIDS most likely?
A. At 1 to 2 years of age.
B. At 1 week to 1 year of age, peaking at 2 to 4 months.
C. At 6 months to 1 year of age, peaking at 10 months.
D. At 6 to 8 weeks of age.

Answer: B. At 1 week to 1 year of age, peaking at 2 to 4 months.

Options B: SIDS can occur anytime between 1 week and 1 year of age.

Options A, C, D: The incidence peaks at 2 to 4 months of age.

27. The nurse teaches the mother of a newborn that in order to prevent sudden infant death
syndrome (SIDS) the best position to place the baby after nursing is
A. Prone.
B. Trendelenberg
C. Supine.
D. Fowler’s.

Answer: C

Options B and C. Research demonstrate that the occurrence of SIDS is reduced with these two
positions.

Options A and D: Both are inappropriate positions for infants.

28. A 7-year-old client is brought to the E.R. He’s tachypneic and afebrile and has a respiratory
rate of 36 breaths/minute and a nonproductive cough. He recently had a cold. From his
history, the client may have which of the following?
A. Acute asthma
B. Bronchial pneumonia
C. Chronic obstructive pulmonary disease (COPD)
D. Emphysema

Correct Answer: A. Acute asthma

Based on the client’s history and symptoms, acute asthma is the most likely diagnosis. Patients will
usually give a history of a wheeze or a cough, exacerbated by allergies, exercise, and cold. There
is often diurnal variation, with symptoms being worse at night. Many asthmatics have nocturnal
coughing spells but appear normal in the daytime. He’s unlikely to have bronchial pneumonia
without a productive cough and fever and he’s too young to have developed COPD or
emphysema.

29. A client with acute asthma showing inspiratory and expiratory wheezes and a decreased
expiratory volume should be treated with which of the following classes of medication right
away?
A. Beta-adrenergic blockers
B. Bronchodilators
C. Inhaled steroids
D. Oral steroids

Correct Answer: B. Bronchodilators

Bronchodilators are the first line of treatment for asthma because bronchoconstriction is the cause
of reduced airflow. Bronchodilators are indicated for individuals that have lower than optimal airflow
through the lungs. The mainstay of treatment is beta-2 agonists that target the smooth muscles in
the bronchioles of the lung. Various respiratory conditions may require bronchodilators, including
asthma and chronic obstructive pulmonary disease.

30. Emergency treatment of a client in status asthmaticus includes which of the following
medications?
A. Inhaled beta-adrenergic agents
B. Inhaled corticosteroids
C. I.V. beta-adrenergic agents
D. Oral corticosteroids

Correct Answer: A. Inhaled beta-adrenergic agents

Inhaled beta-adrenergic agents help promote bronchodilation, which improves oxygenation.


Albuterol is preferred over metaproterenol in that class because of its higher beta 2 selectivities
and longer duration of action. The dose-response curve and duration of action of these
medications are adversely affected by a combination of patient factors, including pre-existing
bronchoconstriction, airway inflammation, mucus plugging, poor patient effort, and coordination.

31. After reviewing the client’s maternal history of magnesium sulfate during labor, which
condition would the nurse anticipate as a potential problem in the neonate?
A. Hypoglycemia
B. Jitteriness
C. Respiratory depression
D. Tachycardia

C. Respiratory depression. Magnesium sulfate crosses the placenta and adverse neonatal effects
are respiratory depression, hypotonia, and Bradycardia.

32. The most common neonatal sepsis and meningitis infections seen within 24 hours after
birth are caused by which organism?
A. Candida albicans
B. Chlamydia trachomatis
C. Escherichia coli
D. Group B beta-hemolytic streptococci
33. Which condition or treatment best ensures lung maturity in an infant?
A. Meconium in the amniotic fluid
B. Glucocorticoid treatment just before delivery
C. Lecithin to sphingomyelin ratio more than 2:1
D. Absence of phosphatidylglycerol in amniotic fluid

The lecithin to sphingomyelin ratio of 2:1 or greater is characteristic of mature fetal lungs

34. A newborn’s mother is alarmed to find small amounts of blood on her infant girl’s diaper.
When the nurse checks the infant’s urine it is straw colored and has no offensive odor.
Which explanation to the newborn’s mother is most appropriate?
A. “It appears your baby has a kidney infection”
B. “Breast-fed babies often experience this type of bleeding problem due to lack of vitamin C in
the breast milk”
C. “The baby probably passed a small kidney stone”
D. “Some infants experience menstruation like bleeding when hormones from the mother
are not available”

35. Soon after delivery, a neonate is admitted to the central nursery. The nursery nurse begins
the initial assessment by:
A. auscultate bowel sounds.
B. determining chest circumference.
C. inspecting the posture, color, and respiratory effort.
D. checking for identifying birthmarks.

36. The nurse hears the mother of a 5-pound neonate telling a friend on the telephone, “As
soon as I get home, I’ll give him some cereal to get him to gain weight.” The nurse
recognizes the need for further instruction about infant feeding and tells her:
A. “If you give the baby cereal, be sure to use Rice to prevent allergy.”
B. “The baby is not able to swallow cereal, because he is too small.”
C. “The infant’s digestive tract cannot handle complex carbohydrates like cereal.”
D. “If you want him to gain weight, just double his daily intake of formula.”

37. The nurse is preparing to discharge a multipara 24 hours after a vaginal delivery. The client
is breastfeeding her newborn. The nurse instructs the client that if engorgement occurs the
client should:
A. wear a tight fitting bra or breast binder.
B. apply warm, moist heat to the breasts.
C. contact the nurse-midwife for a lactation suppressant.
D. restrict fluid intake to 1000 ml daily.

Correct Answer: B. apply warm, moist heat to the breasts.

Option B: Moist heat has this amazing ability to increase circulation, open milk ducts and stimulate
let down – all of which encourage the milk to start flowing. Option A: If a bra is worn, it should be
big enough or stretchy enough to allow for expansion if breasts fill during the night hours; a bra that
is too tight can cause soreness and potential problems such as blocked ducts. Option C: The
simplest and safest way to suppress lactation is to let milk production stop on its own. Option D:
Research has found that nursing mothers do not need to drink more fluids than what's necessary
to satisfy their thirst.

38. This is a condition of inability to conceive after at least 1 year of sexual intercourse at least
four times per week without contraception.
a. Subfertility
b. Infertility
c. Sterility
d. all of the above

39. It is a persistent or recurring absence of sexual thoughts or disinterest in sexual activity.


a. impotence
b. hyperactive sexual desire
c. hypoactive sexual desire
d. sexual arousal disorder

40. A condition where in partner experience pain during intercourse due to inadequate
lubrication, infection or hormonal imbalance.
a. dyspareunia
b. vaginitis
c. salphingitis
d. any of the above
41. The most common cause of subfertility in women, may occur from a genetic abnormality in
which there are no ovaries to produce ova. T
a. Turner’s syndrome (hypogonadism)
b. Myoma
c. Endometriosis
d. Polycystic ovary syndrome

42. The minimum normal sperm count per milliliter of seminal fluid.
15 million
20 million
25 million
30 million

43. The least costly way to determine a woman’s ovulation pattern is to ask her to record her
basal body temperature (BBT) for at least 4 months.
True
False

44. Which statement is the scientific rationale for infusing a unit of blood in less than four (4)
hours?
a. The blood will coagulate if left out of the refrigerator for >four (4) hours.
b. The blood has the potential for bacterial growth if allowed to infuse longer.
c. The blood components begin to break down after four (4) hours.
d. The blood will not be affected; this is a laboratory procedure

answer B. The blood has the potential for bacterial growth if allowed to infuse longer.

45. The client diagnosed with anemia begins to complain of dyspnea when ambulating in the
hall. Which intervention should the nurse implement first?
a. Apply oxygen via nasal cannula.
b. Get a wheelchair for the client.
c. Assess the client's lung fields.
d. Assist the client when ambulating in the hall.

46. Nurse Christine is planning a client education program for sickle cell disease (SCD) in
children; which of the following interventions would be included in the care plan?
A. Health teaching to help reduce sickling crisis
B. Avoidance of the use of opioids
C. Administration of an anticoagulant to prevent sickling
D. Observation of the imposed fluid restriction

Rationale:

Option A: Prevention is one of the principal goals of therapeutic management because there is no
cure for sickle cell disease. Consequently, health education to help lessen sickling crises is key.

Option B: Opioids usually are required for pain control.

Option C: Anticoagulants do not prevent sickling.

Option D: Fluids are encouraged to increase the fluid volume and prevent sickling.

47. Mrs. Santos was instructed by the nurse on foods to encourage her child's diet concerning
the latter's iron deficiency anemia; which of the following if stated by the mother would
indicate the need for further instruction?
A. Fish
B. Lean meats
C. Whole-grain breads
D. Yellow vegetables

Rationale:

Option D: If a parent states that she should stress the intake of yellow vegetables, she needs
additional teaching because yellow vegetables are not good source of iron.

Options A, B, and C: Fish, lean meats, and whole-grain breads are good food sources of iron and
should be supported and encouraged.

48. The long-term complications seen in thalassemia major are associated to which of the
following?
A. Anemia
B. Growth retardation
C. Hemochromatosis
D. Splenomegaly

Option C: Long-term complications arise from hemochromatosis, excessive iron deposits


precipitating in the tissues and causing destruction.

Option A: Anemia is a sign of this disorder.


Options B and D: Cellular damage from hemochromatosis may lead to splenomegaly, growth
retardation, skeletal complications, cardiac problems, gallbladder disease, hepatomegaly, and skin
changes.

49. The patient with anemia has been prescribed an iron supplement. With which beverage
should the nurse encourage the patient to take the supplement?
A) Milk
B) Water only
C) Orange Juice
D) Tea

Answer: C. Vitamin C helps aid in the absorption of iron. Therefore iron supplements should be
taken with a glass or orange juice or a vitamin C tablet.
https://quizlet.com/136060997/med-surg-ch25-patient-with-cancer-study-guide-and-nclex-questions-flash-cards/

https://quizlet.com/233520526/chapter-22-care-of-patients-with-cancer-nclex-questions-flash-cards/

https://quizlet.com/165497530/nclex-breast-cancer-flash-cards/

https://quizlet.com/82571996/intrapartum-flash-cards/

https://quizlet.com/60244633/nclex-questions-grwoth-development-exam-vi-flash-cards/

https://quizlet.com/86408928/ob-complications-flash-cards/
https://quizlet.com/337540526/labor-aq-flash-cards/
https://quizlet.com/390191810/high-risk-intrapartum-practice-questions-flash-cards/
https://quizlet.com/337688235/labordeliveryhigh-risk-flash-cards/

https://quizlet.com/510173298/maternal-and-child-nursing-sample-exam-1-flash-cards/
https://quizlet.com/89886075/nr465-maternal-nursing-exam-2-antepartum-labor-and-delivery-newborn-medications-flash-
cards/
https://quizlet.com/58143354/intrapartum-nclex-questions-flash-cards/

https://quizlet.com/452648766/exam-4-high-risk-newborn-nclex-questions-flash-cards/

https://quizlet.com/210021411/nclex-rn-practice-test-2-flash-cards/
https://quizlet.com/541730854/nur-128-oxygenation-sids-flash-cards/
https://quizlet.com/82298530/nursing-170-final-flashcards-alterations-in-oxygenation-flash-cards/
CARDIO
https://quizlet.com/555654336/final-exam-congenital-heart-defects-coarctation-of-aorta-ventricular-septal-defect-patent-d
uctus-arteriosus-tetralogy-of-fallot-flash-cards/
https://quizlet.com/450870493/cardiovascular-pediatrics-flash-cards/

https://quizlet.com/533107599/practice-questions-week-8-flash-cards/

https://quizlet.com/304686302/developmental-stages-infancy-to-adolescents-flash-cards/#:~:text=The%20maternity%20
nurse%20is%20providing,infant%20to%20signal%20a%20need.
https://quizlet.com/304265546/postpartum-complications-chap-21-saunders-30-ati-20-flash-cards/
HEMATOLOGIC
https://quizlet.com/103438851/hematologic-disorders-nclex-flash-cards/
https://quizlet.com/393885699/hematologic-disorders-practice-questions-flash-cards/
https://quizlet.com/518252663/pediatric-nursing-hematologic-disorders-nclex-practice-quiz-flash-cards/
MUSCULOSKELETAL
https://quizlet.com/80711804/musculoskeletal-nclex-practice-questions-flash-cards/
https://quizlet.com/117001447/chapter-42-musculoskeletal-disorders-nclex-review-flash-cards/

https://quizlet.com/415041636/chapter-25-the-high-risk-newborn-nclex-flash-cards/
https://quizlet.com/262537799/peds-ch-15-nursing-care-of-the-hospitalized-child-flash-cards/
https://quizlet.com/65656822/chapter-40-nursing-considerations-for-the-hospitalized-child-flash-cards/
QUIZ 1: MATERNAL AND CHILD (well)

1. When assessing the adequacy of sperm for conception to occur, which of the following
is the most useful criterion?

A. Sperm count

B. Sperm motility

C. Sperm maturity

D. Semen volume

Rationale: Although all of the factors listed are important, sperm motility is the most
significant criterion when assessing male infertility. Sperm count, sperm maturity, and
semen volume are all significant, but they are not as significant as sperm motility.

2. Which of the following urinary symptoms does the pregnant woman most frequently
experience during the first trimester?

a. Dysuria

b. Frequency

c. Incontinence

d. Burning

Rationale: Pressure and irritation of the bladder by the growing uterus during the first
trimester is responsible for causing urinary frequency. Dysuria, incontinence, and burning
are symptoms associated with urinary tract infections.

3. On which of the following areas would the nurse expect to observe chloasma?

a. Breast, areola, and nipples

b. Chest, neck, arms, and legs

c. Abdomen, breast, and thighs

d. Cheeks, forehead, and nose

Rationale: Chloasma, also called the mask of pregnancy, is an irregular hyperpigmented


area found on the face. It is not seen on the breasts, areola, nipples, chest, neck, arms,
legs, abdomen, or thighs.
4. Cervical softening and uterine souffle are classified as which of the following?

a. Diagnostic signs

b. Presumptive signs

c. Probable signs

d. Positive signs

Rationale: Cervical softening (Goodell sign) and uterine soufflé are two probable signs of
pregnancy. Probable signs are objective findings that strongly suggest pregnancy. Other
probable signs include Hegar sign, which is softening of the lower uterine segment;
Piskacek sign, which is enlargement and softening of the uterus; serum laboratory tests;
changes in skin pigmentation; and ultrasonic evidence of a gestational sac. Presumptive
signs are subjective signs and include amenorrhea; nausea and vomiting; urinary
frequency; breast tenderness and changes; excessive fatigue; uterine enlargement; and
quickening.

5. Which of the following would the nurse identify as a presumptive sign of pregnancy?

a. Hegar sign

b. Nausea and vomiting

c. Skin pigmentation changes

d. Positive serum pregnancy test

Rationale: Presumptive signs of pregnancy are subjective signs. Of the signs listed, only
nausea and vomiting are presumptive signs. Hegar sign,skin pigmentation changes, and a
positive serum pregnancy test are considered probably signs, which are strongly
suggestive of pregnancy.

6. A pregnant client states that she “waddles” when she walks. The nurse’s explanation is
based on which of the following is the cause?

a. The large size of the newborn

b. Pressure on the pelvic muscles

c. Relaxation of the pelvic joints

d. Excessive weight gain

Rationale: During pregnancy, hormonal changes cause relaxation of the pelvic joints,
resulting in the typical "waddling" gait. Changes in posture are related to the growing fetus.
Pressure on the surrounding muscles causing discomfort is due to the growing uterus.
Weight gain has no effect on gait.

7. FHR can be auscultated with a fetoscope as early as which of the following?

a. 5 weeks gestation

b. 10 weeks gestation

c. 15 weeks gestation

d. 20 weeks gestation

Rationale: The FHR can be auscultated with a fetoscope at about 20 week's gestation.
FHR usually is ausculatated at the midline suprapubic region with Doppler ultrasound
transducer at 10 to 12 week's gestation. FHR, cannot be heard any earlier than 10 weeks'
gestation.

8. Which of the following fundal heights indicates less than 12 weeks’ gestation when the
date of the LMP is unknown?

a. Uterus in the pelvis

b. Uterus at the xiphoid

c. Uterus in the abdomen

d. Uterus at the umbilicus

Rationale: When the LMP is unknown, the gestational age of the fetus is estimated by
uterine size or position (fundal height). The presence of the uterus in the pelvis indicates
less than 12 weeks’ gestation. At approximately 12 to 14 weeks, the fundus is out of the
pelvis above the symphysis pubis. The fundus is at the level of the umbilicus at
approximately 20 weeks’ gestation and reaches the xiphoid at term or 40 weeks.

9. Which of the following would cause a false-positive result on a pregnancy test?

a. The test was performed less than 10 days after an abortion

b. The test was performed too early or too late in the pregnancy

c. The urine sample was stored too long at room temperature

d. A spontaneous abortion or a missed abortion is impending

Rationale: A false-positive reaction can occur if the pregnancy test is performed less than
10 days after an abortion. Performing the tests too early or too late in the pregnancy,
storing the urine sample too long at room temperature, or having a spontaneous or missed
abortion impending can all produce false-negative results.

10. Which of the following nursing interventions would the nurse perform during the third
stage of labor?

a. Obtain a urine specimen and other laboratory tests.

b. Assess uterine contractions every 30 minutes.

c. Coach for effective client pushing

d. Promote parent-newborn interaction.

Rationale: During the third stage of labor, which begins with the delivery of the newborn,
the nurse would promote parent-newborn interaction by placing the newborn on the
mother's abdomen and encouraging the parents to touch the newborn. Collecting a urine
specimen and other laboratory tests is done on admission during the first stage of labor.
Assessing uterine contractions every 30 minutes is performed during the latent phase of
the first stage of labor. Coaching the client to push effectively is appropriate during the
second stage of labor.

11. When talking with a pregnant client who is experiencing aching swollen, leg veins, the
nurse would explain that this is most probably the result of which of the following?

a. Thrombophlebitis

b. Pregnancy-induced hypertension

c. Pressure on blood vessels from the enlarging uterus

d. The force of gravity pulling down on the uterus

Rationale: Pressure of the growing uterus on blood vessels results in an increased risk for
venous stasis in the lower extremities. Subsequently, edema and varicose vein formation
may occur. Thrombophlebitis is an inflammation of the veins due to thrombus formation.
Pregnancy-induced hypertension is not associated with these symptoms. Gravity plays
only a minor role with these symptoms.

12. Immediately before expulsion, which of the following cardinal movements occur?

a. Descent

b. Flexion
c. Extension

d. External rotation

Rationale: Immediately before expulsion or birth of the rest of the body, the cardinal
movement of external rotation occurs. Descent flexion, internal rotation, extension, and
restitution (in this order) occur before external rotation.

13. When taking an obstetrical history on a pregnant client who states, “I had a son born at
38 weeks gestation, a daughter born at 30 weeks gestation and I lost a baby at about 8
weeks,” the nurse should record her obstetrical history as which of the following?

a. G2 T2 P0 A0 L2

b. G3 T1 P1 A0 L2

c. G3 T2 P0 A0 L2

d. G4 T1 P1 A1 L2

Rationale: The client has been pregnant four times, including current pregnancy (G). Birth
at 38 weeks’ gestation is considered full term (T), while birth form 20 weeks to 38 weeks is
considered preterm (P). A spontaneous abortion occurred at 8 weeks (A). She has two
living children (L).

14. Which of the following refers to the single cell that reproduces itself after conception?

a. Chromosome

b. Blastocyst

c. Zygote

d. Trophoblast

Rationale: The zygote is the single cell that reproduces itself after conception. The
chromosome is the material that makes up the cell and is gained from each parent.
Blastocyst and trophoblast are later terms for the embryo after zygote.
15. A postpartum client has a temperature of 101.4ºF, with a uterus that is tender when
palpated, remains unusually large, and not descending as normally expected. Which of the
following should the nurse assess next?

a. Lochia

b. Breasts

c. Incision

d. Urine

Rationale: The data suggests an infection of the endometrial lining of the uterus. The
lochia may be decreased or copious, dark brown in appearance, and foul smelling,
providing further evidence of a possible infection.

Option B: All the client’s data indicate a uterine problem, not a breast problem. Typically,
transient fever, usually 101ºF, may be present with breast engorgement. Symptoms of
mastitis include influenza-like manifestations.

Option C: Localized infection of an episiotomy or C-section incision rarely causes systemic


symptoms, and uterine involution would not be affected.

Option D: The client data do not include dysuria, frequency, or urgency, symptoms of
urinary tract infections, which would necessitate assessing the client’s urine.

16. When teaching a client about contraception. Which of the following would the nurse
include as the most effective method for preventing sexually transmitted infections?

a. Spermicides

b. Diaphragm

c. Condoms

d. Vasectomy

Rationale: Condoms, when used correctly and consistently, are the most effective
contraceptive method or barrier against bacterial and viral sexually transmitted infections.
Although spermicides kill sperm, they do not provide reliable protection against the spread
of sexually transmitted infections, especially intracellular organisms such as HIV. Insertion
and removal of the diaphragm along with the use of the spermicides may cause vaginal
irritations, which could place the client at risk for infection transmission. Male sterilization
eliminates spermatozoa from the ejaculate, but it does not eliminate bacterial and/or viral
microorganisms that can cause sexually transmitted infections.

17. When performing a pelvic examination, the nurse observes a red swollen area on the
right side of the vaginal orifice. The nurse would document this as enlargement of which of
the following?

a. Clitoris

b. Parotid gland

c. Skene’s gland

d. Bartholin’s gland

Rationale: Bartholin's glands are the glands on either side of the vaginal orifice. The clitoris
is female erectile tissue found in the perineal area above the urethra. The parotid glands
are open into the mouth. Skene's glands open into the posterior wall of the female urinary
meatus.

18. A nurse is assessing a client in the 4th stage if labor and notes that the fundus is firm
but that bleeding is excessive. The initial nursing action would be which of the following?

a. Massage the fundus

b. Place the mother in the Trendelenburg’s position

c. Notify the physician

d. Record the findings

Rationale: If the bleeding is excessive, the cause may be laceration of the cervix or birth
canal. Massaging the fundus if it is firm will not assist in controlling the bleeding.
Trendelenburg's position is to be avoided because it may interfere with cardiac function.

19. Methergine or pitocin is prescribed for a woman to treat PP hemorrhage. Before


administration of these medications, the priority nursing assessment is to check the:

a. Amount of lochia
b. Blood pressure

c. Deep tendon reflexes

d. Uterine tone

Rationale: Methergine and pitocin are agents that are used to prevent or control
postpartum hemorrhage by contracting the uterus. They cause continuous uterine
contractions and may elevate blood pressure. A priority nursing intervention is to check
blood pressure. The physician should be notified if hypertension is present.

20. Which of the following interventions would be helpful to a breastfeeding mother who is
experiencing engorged breasts?

a. Applying ice

b. Applying a breast binder

c. Teaching how to express her breasts in a warm shower

d. Administering bromocriptine (Parlodel)

Rationale: Teaching the client how to express her breasts in a warm shower aids with
let-down and will give temporary relief. Ice can promote comfort by vasoconstriction,
numbing, and discouraging further letdown of milk.

21. On the first PP night, a client requests that her baby be sent back to the nursery so
she can get some sleep. The client is most likely in which of the following phases?

a. Depression phase

b. Letting-go phase

c. Taking-hold phase

d. Taking-in phase

Rationale:
22. A nurse in the labor room is caring for a client in the active phases of labor. The nurse
is assessing the fetal patterns and notes a late deceleration on the monitor strip. The most
appropriate nursing action is to:

a. Place the mother in the supine position

b. Document the findings and continue to monitor the fetal patterns

c. Administer oxygen via face mask

d. Increase the rate of pitocin IV infusion

Rationale:

23. A laboring client complains of low back pain. The nurse replies that this pain occurs
most when the position of the fetus is

a. Breech

b. Transverse

c. Occiput anterior

d. Occiput posterior

Rationale:

24. A nurse is describing the process of fetal circulation to a client during a prenatal visit.
The nurse accurately tells the client that fetal circulation consists of:

a. Two umbilical veins and one umbilical artery

b. Two umbilical arteries and one umbilical vein

c. Arteries carrying oxygenated blood to the fetus

d. Veins carrying deoxygenated blood to the fetus

Rationale:
25. A nurse is performing an assessment of a primipara who is being evaluated in a clinic
during her second trimester of pregnancy. Which of the following indicates an abnormal
physical finding necessitating further testing?

a. Consistent increase in fundal height

b. Fetal heart rate of 180 BPM

c. Braxton hicks contractions

d. Quickening

Rationale:

26. Before birth, which of the following structures connects the right and left auricles of the
heart?

a. Umbilical vein

b. Foramen ovale

c. Ductus arteriosus

d. Ductus venosus

Rationale:

27. Which of the following is true regarding the fontanels of the newborn?

a. The anterior is triangular shaped; the posterior is diamond shaped.

b. The posterior closes at 18 months; the anterior closes at 8 to 12 weeks.

c. The anterior is large in size when compared to the posterior fontanel.

d. The anterior is bulging; the posterior appears sunken.

Rationale:
28. A nurse prepares to administer a vitamin K injection to a newborn infant. The mother
asks the nurse why her newborn infant needs the injection. The best response by the
nurse would be:

a. “Your infant needs vitamin K to develop immunity.”

b. “The vitamin K will protect your infant from being jaundiced.”

c. “Newborn infants are deficient in vitamin K, and this injection prevents


your infant from abnormal bleeding.”

d. “Newborn infants have sterile bowels, and vitamin K promotes the growth of
bacteria in the bowel.”

Rationale:

29. The primary critical observation for Apgar scoring is the:

a. Heart rate

b. Respiratory rate

c. Presence of meconium

d. Evaluation of the Moro reflex

Rationale:

30. A nurse in a newborn nursery is performing an assessment of a newborn infant. The


nurse is preparing to measure the head circumference of the infant. The nurse would most
appropriately:

a. Wrap the tape measure around the infant’s head and measure just above the eyebrows.

b. Place the tape measure under the infants head at the base of the skull and wrap around
to the front just above the eyes
c. Place the tape measure under the infants head, wrap around the occiput, and
measure just above the eyes

d. Place the tape measure at the back of the infant’s head, wrap around across the ears,
and measure across the infant’s mouth.

Rationale:

31. Vitamin K is prescribed for a neonate. A nurse prepares to administer the medication
in which muscle site?

a. Deltoid

b. Triceps

c. Vastus lateralis

d. Biceps

Rationale:

32. A nursing instructor asks a nursing student to describe the procedure for administering
erythromycin ointment into the eyes if a neonate. The instructor determines that the
student needs to research this procedure further if the student states:

a. “I will cleanse the neonate’s eyes before instilling ointment.”

b. “I will flush the eyes after instilling the ointment.”

c. “I will instill the eye ointment into each of the neonate’s conjunctival sacs within one
hour after birth.”

d. “Administration of the eye ointment may be delayed until an hour or so after birth so that
eye contact and parent-infant attachment and bonding can occur.”
Rationale:

33. A baby is born precipitously in the ER. The nurses initial action should be to:

A. Establish an airway for the baby

B. Ascertain the condition of the fundus

C. Quickly tie and cut the umbilical cord

D. Move mother and baby to the birthing unit

Rationale:

34. The primary critical observation for Apgar scoring is the:

A. Heart rate

B. Respiratory rate

C. Presence of meconium

D. Evaluation of the Moro reflex

Rationale:

35. When performing a newborn assessment, the nurse should measure the vital
signs in the following sequence:
A. Pulse, respirations, temperature

B. Temperature, pulse, respirations

C. Respirations, temperature, pulse

D. Respirations, pulse, temperature

Rationale:

36. Within 3 minutes after birth the normal heart rate of the infant may range between:

A. 100 and 180

B. 130 and 170

C. 120 and 160

D. 100 and 130

Rationale:

37. The nurse is aware that a healthy newborn’s respirations are:

A. Regular, abdominal, 40-50 per minute, deep

B. Irregular, abdominal, 30-60 per minute, shallow

C. Irregular, initiated by chest wall, 30-60 per minute, deep

D. Regular, initiated by the chest wall, 40-60 per minute, shallow

Rationale:
38. A newborn has small, whitish, pinpoint spots over the nose, which the nurse knows are
caused by retained sebaceous secretions. When charting this observation, the nurse
identifies it as:

A. Milia

B. Lanugo

C. Whiteheads

D. Mongolian spots

Rationale:

39. When teaching umbilical cord care to a new mother, the nurse would include which
information?

A. Apply peroxide to the cord with each diaper change

B. Cover the cord with petroleum jelly after bathing

C. Keep the cord dry and open to air

D. Wash the cord with soap and water each day during a tub bath

Rationale:

40. A mother of a term neonate asks what the thick, white, cheesy coating is on his skin.
Which correctly describes this finding?
A. Lanugo

B. Milia

C. Nevus flammeus

D. Vernix

Rationale:

41. Which condition or treatment best ensures lung maturity in an infant?

A. Meconium in the amniotic fluid

B. Glucocorticoid treatment just before delivery

C. Lecithin to sphingomyelin ratio more than 2:1

D. Absence of phosphatidylglycerol in amniotic fluid

Rationale:

42. When performing nursing care for a neonate after a birth, which intervention has the
highest nursing priority?

A. Obtain a dextrostix

B. Give the initial bath

C. Give the vitamin K injection

D. Cover the neonates head with a cap


Rationale:

43. A woman delivers a 3.250 g neonate at 42 weeks’ gestation. Which physical finding is
expected during an examination if this neonate?

A. Abundant lanugo

B. Absence of sole creases

C. Breast bud of 1-2 mm in diameter

D. Leathery, cracked, and wrinkled skin

Rationale:

44. Which age group has the greatest potential to demonstrate regression when they are
sick?

a. Infant

b. Toddler

c. Adolescent

d. Young adult

Rationale: Toddlers are less able to understand and interpret what is happening to them
when ill; therefore, they commonly regress to a previous level of development in an
attempt to reduce anxiety.
45. Which stage of development is most unstable and challenging regarding the
development of personal identity?

a. Adolescence

b. Toddlerhood

c. Middle childhood

d. Young adulthood

Rationale: Adolescents (12 to 20 years—Identity versus Role Confusion) have multiple


and complex physiological (e.g., puberty), psychological (e.g., self identity and
independence), and social (e.g., peer pressure, altered roles, and maturing relationships)
milestones than any other stage of development. The multiplicity

of these stressors can have a major impact on the development of the adolescent's
personal identity and sense of self.

46. A nurse is evaluating the developmental level of a two (2)-year-old. Which of the
following does the nurse expect to observe in this child?

a. Uses a fork to eat

b. Uses a cup to drink

c. Uses a knife in cutting foods

d. Pours own milk into the cup

Rationale: By age 2 years, the child can use a cup and can use a spoon correctly but with
some spilling. By ages 3 to 4, the child begins to use a fork. By the end of the preschool
period, the child should be able to pour milk into a cup and begin to use a knife for cutting.
47. A maternity nurse is providing instruction to a new mother regarding the psychosocial
development of the newborn infant. Using Erikson’s psychosocial development theory, the
nurse would instruct the mother to

a. Allow the newborn infant to signal a need

b. Anticipate all of the needs of the newborn infant

c. Avoid the newborn infant during the first 10 minutes of crying

d. Allow the infant to cry, once lessen, then attend to the infant

Rationale: According to Erikson, the caregiver should not try to always anticipate the
newborn infant’s needs but must allow the newborn infant to signal needs. If a newborn is
not allowed to signal a need, the newborn will not learn how to control the environment.
Erikson believed that a delayed or prolonged response to a newborn's signal would inhibit
the development of trust and lead to mistrust of others.

48. The parents of a two (2)-year-old boy arrive at a hospital for a visit. The child is in the
playroom when the parents arrive. When the parents enter the playroom, the child does
not readily approach the parents. The nurse interprets this behavior as indicating that:

a. The child is withdrawn

b. The child is self-centered

c. The child has adjusted to the hospitalized setting

d. This is a normal pattern


Rationale: The phases through which young children progress when separated from their
parents include protest, despair, and denial or detachment. In the stage of protest, when
the parents return, the child readily goes to them. In the stage of despair, the child may not
approach them readily or may cling to a parent. In denial or detachment, when the parents
return, the child becomes cheerful, interested in the environment and new persons
(seemingly unaware of the lost parents), friendly with the staff, and interested in
developing superficial relationships.

49. A mother of a three (3)-year-old tells a clinic nurse that the child is constantly rebelling
and having temper tantrums. The nurse most appropriately tells the mother to:

a. Punish the child every time the child says “no”, to change the behavior

b. Allow the behavior because this is normal at this age period

c. Set limits on the child’s behavior

d. Ignore the child when this behavior occurs

Rationale: According to Erikson, the child focuses on independence between ages 1 and 3
years. Gaining independence often means that the child has to rebel against the parents'
wishes. Saying things like "no" or "mine" and having temper tantrums are common during
this period of development. Being consistent and setting limits on the child's behavior are
the necessary elements.

50. A clinic nurse assesses the communication patterns of a five (5)-month-old infant. The
nurse determines that the infant is demonstrating the highest level of developmental
achievement expected if the infant:

a. Uses simple words such as “mama”

b. Uses monosyllabic babbling

c. Links syllables together


d. Coos when comforted

Rationale:

Reference/s:

https://quizlet.com/60244633/nclex-questions-grwoth-development-exam-vi-flash-cards/

https://quizlet.com/34517445/fundamentals-of-nursing-adult-lifespan-flash-cards/

1. One of the participants attending a parenting class asks the teacher “what is the leading
cause of death during the first month of life?

A. Congenital Abnormalities

B. Low birth weight

C. SIDS

D. Infection

2. Immediately after birth the nurse notes the following on a male newborn: respirations
78; apical hearth rate 160 BPM, nostril flaring; mild intercostal retractions; and grunting at
the end of expiration. Which of the following should the nurse do?
A. Call the assessment data to the physician’s attention

B. Start oxygen per nasal cannula at 2 L/min.

C. Suction the infant’s mouth and nares

D. Recognize this as normal first period of reactivity

https://quizlet.com/255282850/questions-i-got-wrong-pt-2-flash-cards/

3. A client has just given birth at 42 weeks’ gestation. When assessing the neonate, which
physical finding is expected?

A. A sleepy, lethargic baby

B. Lanugo covering the body

C. Desquamation of the epidermis

D. Vernix caseosa covering the body

https://quizlet.com/24292121/final-flash-cards/

4. After reviewing the client’s maternal history of magnesium sulfate during labor, which
condition would the nurse anticipate as a potential problem in the neonate?

A. Hypoglycemia

B. Jitteriness

C. Respiratory depression

D. Tachycardia

C. Respiratory Depression.Magnesium Sulfate crosses the placenta, and adverse


neonatal effects are respiratory depression, hypotonia, and bradycardia. The serum blood
sugar isn't affected by magnesium sulfate. The neonate would be floppy, not jittery.

https://quizlet.com/127266015/q3-pharm-magnesium-sulfate-quiz-flash-cards/
5. Neonates of mothers with diabetes are at risk for which complication following birth?

A. Atelectasis

B. Microcephaly

C. Pneumothorax

D. Macrosomia

By keeping the nursery temperature warm and wrapping the neonate in blankets, the
nurse is preventing which type of heat loss?

A. Conduction

B. Convection

C. Evaporation

D. Radiation

Convection heat loss is the flow of heat from the body surface to the cooler air.

https://quizlet.com/376855337/ob-newborn-nclex-practice-questions-flash-cards/

A neonate has been diagnosed with caput succedaneum. Which statement is correct
about this condition?

A. It usually resolves in 3-6 weeks

B. It doesn’t cross the cranial suture line

C. It’s a collection of blood between the skull and the periosteum

D. It involves swelling of tissue over the presenting part of the presenting head

Caput succedaneum is the swelling of tissue over the presenting part of the fetal scalp
due to sustained pressure; it resolves in 3-4 days.
https://quizlet.com/24292121/final-flash-cards/

The most common neonatal sepsis and meningitis infections seen within 24 hours after
birth are caused by which organism?

A. Candida albicans

B. Chlamydia trachomatis

C. Escherichia coli

D. Group B beta-hemolytic streptococci

Group B streptococcus (GBS) has been a major cause of neonatal sepsis.

A mother of a term neonate asks what the thick, white, cheesy coating is on his skin.
Which correctly describes this finding?

A. Lanugo

B. Milia

C. Nevus flammeus

D. Vernix

Which condition or treatment best ensures lung maturity in an infant?

A. Meconium in the amniotic fluid

B. Glucocorticoid treatment just before delivery

C. Lecithin to sphingomyelin ratio more than 2:1

D. Absence of phosphatidylglycerol in amniotic fluid


When performing nursing care for a neonate after birth, which intervention has the highest
nursing priority?

A. Obtain a dextrostix

B. Give the initial bath

C. Give the vitamin K injection

D. Cover the neonates head with a cap

A healthy term neonate born by C-section was admitted to the transitional nursery 30
minutes ago and placed under a radiant warmer. The neonate has an axillary temperature
of 97.6ºF, a respiratory rate of 80 breaths/minute, and a heel stick glucose value of 60
mg/dl. Which action should the nurse take?

A. Wrap the neonate warmly and place her in an open crib

B. Administer an oral glucose feeding of 10% dextrose in water

C. Increase the temperature setting on the radiant warmer

D. Obtain an order for IV fluid administration

Assessment findings indicate that the neonate is in respiratory distress—most likely from transient
tachypnea, which is common after cesarean delivery. A neonate with a rate of 80 breaths a minute
shouldn’t be fed but should receive IV fluids until the respiratory rate returns to normal. To allow for
close observation for worsening respiratory distress, the neonate should be kept unclothed in the
radiant warmer.

Which neonatal behavior is most commonly associated with fetal alcohol syndrome (FAS)?

A. Hypoactivity

B. High birth weight

C. Poor wake and sleep patterns

D. High threshold of stimulation

Altered sleep patterns are caused by disturbances in the CNS from alcohol exposure in
utero. Hyperactivity is a characteristic generally noted. Low birth weight is a physical
defect seen in neonates with FAS. Neonates with FAS generally have a low threshold for
stimulation.

A newborn’s mother is alarmed to find small amounts of blood on her infant girl’s diaper.
When the nurse checks the infant’s urine it is straw colored and has no offensive odor.
Which explanation to the newborn’s mother is most appropriate?

A. “It appears your baby has a kidney infection”

B. “Breast-fed babies often experience this type of bleeding problem due to lack of vitamin
C in the breast milk”

C. “The baby probably passed a small kidney stone”

D. “Some infants experience menstruation like bleeding when hormones from the mother
are not available”

An insulin-dependent diabetic delivered a 10-pound male. When the baby is brought to the
nursery, the priority of care is to:

A. clean the umbilical cord with Betadine to prevent infection

B. give the baby a bath

C. call the laboratory to collect a PKU screening test

D. check the baby’s serum glucose level and administer glucose if < 40 mg/dL

Soon after delivery, a neonate is admitted to the central nursery. The nursery nurse begins
the initial assessment by:

A. auscultate bowel sounds.

B. determining chest circumference.

C. inspecting the posture, color, and respiratory effort.

D. checking for identifying birthmarks.


A neonate is admitted to a hospital’s central nursery. The neonate’s vital signs are:
temperature = 96.5 degrees F., heart rate = 120 bpm, and respirations = 40/minute. The
infant is pink with slight acrocyanosis. The priority nursing diagnosis for the neonate is:

A. Ineffective thermoregulation related to fluctuating environmental temperatures.

B. Potential for infection related to lack of immunity.

C. Altered nutrition, less than body requirements related to diminished sucking reflex.

D. Altered elimination pattern related to lack of nourishment.

The nurse hears the mother of a 5-pound neonate telling a friend on the telephone, “As
soon as I get home, I’ll give him some cereal to get him to gain weight.” The nurse
recognizes the need for further instruction about infant feeding and tells her:

A. “If you give the baby cereal, be sure to use Rice to prevent allergy.”

B. “The baby is not able to swallow cereal, because he is too small.”

C. “The infant’s digestive tract cannot handle complex carbohydrates like cereal.”

D. “If you want him to gain weight, just double his daily intake of formula.”

The nurse instructs a primipara about safety considerations for the neonate. The nurse
determines that the client does not understand the instructions when she says:

A. “All neonates should be in an approved car seat when in an automobile.”

B. “It’s acceptable to prop the infant’s bottle once in a while.”

C. “Pillows should not be used in the infant’s crib.”

D. “Infants should never be left unattended on an unguarded surface.”


The nurse is preparing to discharge a multipara 24 hours after a vaginal delivery. The
client is breastfeeding her newborn. The nurse instructs the client that if engorgement
occurs the client should:

A. wear a tight fitting bra or breast binder.

B. apply warm, moist heat to the breasts.

C. contact the nurse-midwife for a lactation suppressant.

D. restrict fluid intake to 1000 ml daily.

While giving nursing care to a hospitalized adolescent, the nurse should be aware that the
MAJOR threat felt by the hospitalized adolescent is

A. Pain management

B. Restricted physical activity

C. Altered body image

D. Separation from family

The nurse is observing children playing in the hospital playroom. She would expect to see
4 year-old children playing

A. Competitive board games with older children

B. With their own toys along side with other children

C. Alone with hand held computer games

D. Cooperatively with other preschoolers

Cooperative play is typical of the preschool period.

Which age group has the greatest potential to demonstrate regression when they are sick?

A. Adolescent
B. Young Adult

C. Toddler

D. Infant

The parents of a 2-year-old arrive at a hospital to visit their child. The child is in the
playroom when the parents arrive. When the parents enter the playroom, the child does
not readily approach the parents. The nurse interprets this behavior as indicating that:

A. The child is withdrawn

B. The child is self-centered

C. The child has adjusted to the hospitalized setting

D. This is a normal pattern

Rationale: The phases through which young children progress when separated from their parents
include protest, despair, and denial or detachment. In the stage of protest, when the parents return, the
child readily goes to them. In the stage of despair, the child may not approach them readily or may cling
to a parent. In denial or detachment, when the parents return, the child becomes cheerful, interested in
the environment and new persons (seemingly unaware of the lost parents), friendly with the staff, and
interested in developing superficial relationships.

A nurse is preparing to care for a 5-year-old who has been placed in traction following a
fracture of the femur. The nurse plans care, knowing that which of the following is the most
appropriate activity for this child?

A. Large picture books

B. A radio

C. Crayons and coloring book

D. A sports video

A 16-year-old is admitted to the hospital for acute appendicitis, and an appendectomy is


performed. Which of the following nursing interventions is most appropriate to facilitate
normal growth and development?
A. Allow the family to bring in the child’s favorite computer games

B. Encourage the parents to room-in with the child

C. Encourage the child to rest and read

D. Allow the child to participate in activities with other individuals in the same age group
when the condition permits

A 16 year old child is hospitalized, according to Erik Erikson, what is an appropriate


intervention?

A. tell the friends to visit the child

B. encourage patient to help child learn lessons missed

C. call the priest to intervene

D. tell the child’s girlfriend to visit the child.

The child is 16 years old, In the stage of IDENTITY VS. ROLE CONFUSION. The most
significant persons in this group are the PEERS. B refers to children in the school age
while C refers to the young adulthood stage of INTIMACY VS. ISOLATION. The child is
not dying and the situation did not even talk about the child's belief therefore, calling the
priest is unnecessary.

The nurse is planning pain control for a child. Which is the advantage of administering pain
medication by the intravenous (IV) bolus route?

A. Less expensive than oral medications

B. Produces a first-pass effect through the liver

C. Does not need to be administered frequently

D. Provides most rapid onset of effect, usually in about 5 minutes

Children as young as age 3 years can use facial scales for discrimination. What are some
suggested anchor words for the preschool age group?
A. “No hurt.”

B. “Red pain.”

C. “Zero hurt.”

D. “Least pain

The nurse is assessing a six-month-old child. Which developmental skills are normal and
should be expected?

Speaks in short sentences.

Sits alone.

Can feed self with a spoon.

Pulling up to a standing position.

While teaching a 10 year-old child about their impending heart surgery, the nurse should

A. Provide a verbal explanation just prior to the surgery

B. Provide the child with a booklet to read about the surgery

C. Introduce the child to another child who had heart surgery three days ago

D. Explain the surgery using a model of the heart

The nurse prepares for a Denver Screening test with a 3 year-old child in the clinic. The
mother asks the nurse to explain the purpose of the test. The BEST response is to tell her
that the test

A. Measures potential intelligence

B. Assesses a child’s development

C. Evaluates psychological responses

D. Diagnoses specific problems


The nurse is planning care for an 18 month-old child. Which of the following should be
included in the child’s care?

A. Hold and cuddle the child often

B. Encourage the child to feed himself finger food

C. Allow the child to walk independently on the nursing unit

D. Engage the child in games with other children

The nurse is assessing a four month-old infant. The nurse would anticipate finding that the
infant would be able to

A. Hold a rattle

B. Bang two blocks

C. Drink from a cup

D. Wave “bye-bye”

The age at which a baby will develop the skill of grasping a toy with help is 4 to 6 months.

Which stage of development is most unstable and challenging regarding development of


personal identity?

A. Adolescence

B. Toddler hood

C. Childhood

D. Infancy

Which age group would have a tendency towards eating disorders?

A. Adolescence
B. Toddler hood

C. Childhood

D. Infancy

A maternity nurse is providing instruction to a new mother regarding the psychosocial


development of the newborn infant. Using Erikson’s psychosocial development theory, the
nurse would instruct the mother to

A. Allow the newborn infant to signal a need

B. Anticipate all of the needs of the newborn infant

C. Avoid the newborn infant during the first 10 minutes of crying

D. Attend to the newborn infant immediately when crying

A mother of a 3-year-old tells a clinic nurse that the child is rebelling constantly and having
temper tantrums. The nurse most appropriately tells the mother to:

A. Punish the child every time the child says “no”, to change the behavior

B. Allow the behavior because this is normal at this age period

C. Set limits on the child’s behavior

D. Ignore the child when this behavior occurs

The mother of a 3-year-old is concerned because her child still is insisting on a bottle at
nap time and at bedtime. Which of the following is the most appropriate suggestion to the
mother?

A. Do not allow the child to have the bottle

B. Allow the bottle during naps but not at bedtime

C. Allow the bottle if it contains juice


D. Allow the bottle if it contains water

A nurse is evaluating the developmental level of a 2-year-old. Which of the following does
the nurse expect to observe in this child?

A. Uses a fork to eat

B. Uses a cup to drink

C. Uses a knife for cutting food

D. Pours own milk into a cup

A clinic nurse assesses the communication patterns of a 5-month-old infant. The nurse
determines that the infant is demonstrating the highest level of developmental
achievement expected if the infant:

A. Uses simple words such as “mama”

B. Uses monosyllabic babbling

C. Links syllables together

D. Coos when comforted

Which finding would the nurse consider abnormal when performing a physical assessment
on a 6-month-old?

A. Anterior fontanel is open.

Beginning signs of tooth eruptionB. .

C. Able to track and follow objects.

D. Posterior fontanel is open.


A 12-month-old boy weighed 8 lb 2 oz at birth. Understanding developmental milestones,
what should the nurse caring for the child expect the current weight to be?

24 lb 6 oz

32 lb 8 oz

20 lb 5 oz

16 lb 4 oz

A 39-year-old at 37 weeks gestation is admitted to the hospital with complaints of vaginal


bleeding following the use of cocaine 1 hour earlier. Which complication is most likely
causing the client’s complaint of vaginal bleeding? *

A. Placenta previa

B. Abruptio placentae

C. Ectopic pregnancy

D. Spontaneous abortion

No correct answers

Which of the following is the most common kind of placental adherence seen in pregnant
women? *

A. Accreta

B. Placenta previa

C. Percreta

D. Increta
A pregnant client is diagnosed with partial placenta previa. In explaining the diagnosis, the
nurse tells the client that the usual treatment for partial placenta previa is which of the
following? *

A. Activity limited to bed rest

B. Platelet infusion

C. Immediate cesarean delivery

D. Labor induction with oxytocin

A nurse in the postpartum unit is caring for a client who has just delivered a newborn infant
following a pregnancy with placenta previa. The nurse reviews the plan of care and
prepares to monitor the client for which of the following risks associated with placenta
previa? *

A. Disseminated intravascular coagulation

B. Chronic hypertension

C. Infection

D. Hemorrhage

Because the placenta is implanted in the lower uterine segment, which does not
contain the same intertwining musculature as the fundus of the uterus, this site is
more prone to bleeding.

Maureen in her third trimester arrives at the emergency room with painless vaginal
bleeding. Which of the following conditions is suspected? *

a. Placenta previa

b. Abruptio placentae

c. Premature labor

d. Sexually transmitted disease


Tina experienced painless vaginal bleeding has just been diagnosed as having a placenta
previa. Which of the following procedures is usually performed to diagnose placenta
previa? *

a. Amniocentesis

b. Digital or speculum examination

c. External fetal monitoring

d. Ultrasound

The common normal site of nidation/implantation in the uterus is: *

A. Upper uterine portion

B. Mid-uterine area

C. Lower uterine segment

D. Lower cervical segment

A 39-year-old at 37 weeks gestation is admitted to the hospital with complaints of vaginal


bleeding following the use of cocaine 1 hour earlier. Which complication is most likely
causing the client’s complaint of vaginal bleeding? *

A. Placenta previa

B. Abruptio placentae

C. Ectopic pregnancy

D. Spontaneous abortion

A pregnant woman arrives at the emergency department with abruptio placentae at 34


weeks’ gestation. She’s at risk for which of the following blood dyscrasias? *
A. Thrombocytopenia.

B. Idiopathic thrombocytopenic purpura (ITP).

C. Disseminated intravascular coagulation (DIC).

D. Heparin-associated thrombosis and thrombocytopenia (HATT).

A nurse is assessing a pregnant client in the 2nd trimester of pregnancy who was admitted
to the maternity unit with a suspected diagnosis of abruptio placentae. Which of the
following assessment findings would the nurse expect to note if this condition is present? *

A. Absence of abdominal pain

B. A soft abdomen

C. Uterine tenderness/pain

D. Painless, bright red vaginal bleeding

An ultrasound is performed on a client at term gestation that is experiencing moderate


vaginal bleeding. The results of the ultrasound indicate that an abruptio placentae is
present. Based on these findings, the nurse would prepare the client for: *

A. Complete bed rest for the remainder of the pregnancy

B. Delivery of the fetus

C. Strict monitoring of intake and output

D. The need for weekly monitoring of coagulation studies until the time of delivery

A client makes a routine visit to the prenatal clinic. Although she’s 14 weeks pregnant, the
size of her uterus approximates that in an 18- to 20-week pregnancy. Dr. Diaz diagnoses
gestational trophoblastic disease and orders ultrasonography. The nurse expects
ultrasonography to reveal: *
an empty gestational sac.

grapelike clusters.

a severely malformed fetus.

an extrauterine pregnancy.

Which of the following signs will require a mother to seek immediate medical attention?

When the first fetal movement is felt

No fetal movement is felt on the 6th month

Mild uterine contraction

Slight dyspnea on the last month of gestation

A client makes a routine visit to the prenatal clinic. Although she’s 14 weeks pregnant, the
size of her uterus approximates that in an 18- to 20-week pregnancy. Dr. Diaz diagnoses
gestational trophoblastic disease and orders ultrasonography. The nurse expects
ultrasonography to reveal: *

an empty gestational sac.

grapelike clusters.

a severely malformed fetus.

an extrauterine pregnancy.

Which of the following cause a “complete” molar pregnancy: Select all that applies: *

placenta grows and produces hCG

some fetal components

sperm fertilizes empty egg

two sperm fertilize one egg


no fetus is formed

A client is being admitted to the antepartum unit for hypovolemia secondary to


hyperemesis gravidarum. Which of the following factors predisposes a client to the
development of this? *

trophoblastic disease

maternal age > 35 y.o.

malnourished or underweight clients

low levels of HCG

A 21-year old client, 6 weeks’ pregnant is diagnosed with hyperemesis gravidarum. This
excessive vomiting during pregnancy will often result in which of the following conditions?
*

Bowel perforation

Electrolyte imbalance

Miscarriage

Pregnancy induced hypertension (PIH)

Which of the following statements best describes hyperemesis gravidarum? *

Severe anemia leading to electrolyte, metabolic, and nutritional imbalances in the absence
of other medical problems.

Severe nausea and vomiting leading to electrolyte, metabolic, and nutritional imbalances
in the absence of other medical problems.

Loss of appetite and continuous vomiting that commonly results in dehydration and
ultimately decreasing maternal nutrients
Severe nausea and diarrhea that can cause gastrointestinal irritation and possibly internal
bleeding

Which of the following is the correct health teaching for pregnant mothers taking ferrous
sulfate *

"I should drink milk when ferrous sulfate every daily"

"I should drink calcium when ferrous sulfate every daily"

"I should drink orange juice when ferrous sulfate every daily"

"I should not take anything when ferrous sulfate every daily"

Which of the following is the common risk of a mother unable to take folic acid at the 1st
trimester? *

Risk giving birth with neural tube defects

Risk giving birth with cleft palate

Risk premature birth

Risk giving birth H-mole

Which of the following is eating disorder that involves persistent eating of non-nutritive
substances such as hair, dirt, and paint chips for a period of at least one month? *

Pseudocyesis

Pica

Both A&B

None of the above

Which of the following is no. 1 complication of H-mole? (CAPITALIZED ALL LETTERS) *


CHORIOCARCINOMA

Which of the following is the manifestation of Placenta Previa? *

Painless, bright red vaginal bleeding

Painful, bright red vaginal bleeding

Painless, dark red vaginal bleeding

Painful, dark red vaginal bleeding

Which of the following is the manifestation of Abruptio of Placenta? *

Painful, dark red vaginal bleeding

Painless, dark red vaginal bleeding

Painful, bright red vaginal bleeding

Painless, bright red vaginal bleeding

An 18 years old presents at the emergency department with complaints of lower


abdominal cramping and spotting at 12 weeks' gestation. The physician performs a pelvic
examination and finds that the cervix is closed. What does the physician suspect is the
cause of the cramps and spotting? *

a) Cervical insufficiency

b) Threatened abortion

c) Habitual abortion

d) Ectopic pregnancy

A 21 years old woman is admitted with a diagnosis of ectopic pregnancy. For which of the
following would you anticipate beginning preparation? *

a) Bed rest for the next 4 weeks.


b) Intravenous administration of a tocolytic.

c) Immediate surgery.

d) Internal uterine monitoring.

Ectopic pregnancy means an embryo has implanted outside the uterus, usually in the
fallopian tube. Surgery is usually necessary to remove the growing structure before the
tube ruptures or repair the tube if rupture has already occurred. Bed rest will not correct
the problem of an ectopic pregnancy. Administering a tocolytic is not indicated, nor is
internal uterine monitoring. This makes options A, B, and D incorrect

A client at 9weeks' gestation experiences pregnancy loss. The client asks the nurse if the
bleeding and cramping that occurred during the miscarriage were caused by working long
hours in a stressful environment. What is the most appropriate response from the nurse? *

a) "I can understand your need to find an answer to what caused this. Let's talk about this
further."

b) "It is hard to know why a woman bleeds during early pregnancy."

c) "Your spontaneous bleeding is not work-related."

d) "Something was wrong with the fetus."

Correct answer

c) "Your spontaneous bleeding is not work-related."

Feedback

Rationale:

Ectopic pregnancy means an embryo has implanted outside the uterus, usually in the
fallopian tube. Surgery is usually necessary to remove the growing structure before the
tube ruptures or repair the tube if rupture has already occurred. Bed rest will not correct
the problem of an ectopic pregnancy. Administering a tocolytic is not indicated, nor is
internal uterine monitoring. This makes options A, B, and D incorrect
RhoGAM is given to Rh-negative women to prevent maternal sensitization. In addition to
pregnancy, Rh-negative women would also receive this medication after which of the
following? *

1/1

a. Therapeutic or spontaneous abortion

b. Head injury from a car accident

c. Blood transfusion after a hemorrhage

d. Unsuccessful artificial insemination procedure

Feedback

Any time there is a pregnancy with the chance of maternal and fetal blood mixing,
RhoGAM is needed to prevent sensitization or antibody production. Head injury resulting
from a car crash is not a situation in which there would be mixing of fetal or maternal
blood. The trauma would cause hemorrhage, but not a sensitization reaction. A blood
transfusion after hemorrhage would require typing and cross-matching of the client's
blood; thus, she would receive blood with her own Rh factor, not one with Rh-positive
blood. Because the artificial insemination procedure was unsuccessful, no pregnancy
occurred and RhoGAM would not be necessary.

A pregnant woman, approximately 10 weeks' gestation, comes to the emergency


department after calling her health care provider's office and reporting moderate vaginal
bleeding. Assessment reveals cervical dilation and moderately strong abdominal cramps.
She reports that she has passed some tissue with the bleeding. The nurse interprets these
findings to suggest which of the following? *

a. Threatened abortion

b. Inevitable abortion

c. Incomplete abortion

d. Missed abortion

Correct answer
b. Inevitable abortion

A nurse in the maternity triage unit is caring for a client with a suspected ectopic
pregnancy. Which nursing intervention should the nurse perform first? *

Assess the client's referral and history

Assess the client's emotional resspnse

Assess the client's vital signs.

Assess the client's family status

A woman with an incomplete abortion is to receive misoprostol (Cytotec). The woman asks
the nurse, "Why am I getting this drug?" The nurse responds to the client, integrating
understanding that this drug achieves which effect? *

ensure rest and enough fluid intake

ensure family support

ensures passage of all the products of conception

ensure normal vital signs

Upon entering the room of a client who has had a spontaneous abortion, the nurse
observes the client crying. Which response by the nurse would be most appropriate? *

"I'm sorry you lost your baby."

"Don't worry, you will have another baby soon"

"You can't imagine the numbers of mothers having abortion"

"I'm sure the physician will talk to you and to your husband
A pregnant patient with a history of premature cervical dilatation undergoes cervical
cerclage. Which outcome indicates that this procedure has been successful? *

The client delivers a full-term fetus at 37 weeks' gestation.

The client delivers a full-term fetus at 36 weeks' gestation.

The client delivers a full-term fetus at 39 weeks' gestation.

The client delivers a full-term fetus at 35 weeks' gestation.

Which medication will the nurse anticipate the health care provider will prescribe as
treatment for an unruptured ectopic pregnancy? *

oxytocin

methotrexate

promethazine

methergine

A nurse is conducting an in-service program for a group of nurses working at the women's
health facility about the causes of spontaneous abortion. The nurse determines that the
teaching was successful when the group identifies which condition as the most common
cause of first trimester abortions? *

uterine fibroids

cervical insufficiency

fetal genetic abnormalities

maternal disease

Which of the following is NOT the first Trimester Complications of Pregnancy? *

Abortion (Miscarriage)
Incompetent cervical OS

Ectopic pregnancy

Placenta previa

Which of the following is inappropriate for High Risk Pregnancy Nursing diagnosis? *

Knowledge deficit related to normal changes of pregnancy versus illness complication

Fear regarding financial capacity related to chronic illness

Health-seeking behaviors related to effects of illness of pregnancy

Situational self-esteem related to illness during pregnancy

None of the above

Which of the following is not a danger sign of pregnancy? *

Headache 3/10 early in the morning

Abdominal & chest pain

Visual changes

Vomiting persistent

Which of the following is categorized as an infection caused by organisms that can cross
the placenta by eating raw foods or poorly cooked , contact w/ feces of infected animals &
soil? *

Cytomegalovirus

Toxoplasmosis

Herpes simplex

Syphilis

Which of the following is the causative agent of syphilis? *


HSV

toxoplasma gondii

Treponema pallidum

All of the above

Antenatal Complications: *

Threatened Abortion

Bleeding with closed cervix

Ectopic Pregnancy

knife like abdominal pain with cullen's sign

Incompetent cervix

pink stained vaginal discharge

Missed Abortion

Intermittent bleeding with absence of uterine growth

You might also like